You are on page 1of 142

PEV 107 Page 1

Table of Contents
Unit 1...................................................................................................... 4-27
Sentence Correction ............................................................................................................ 4-19
Sentence Completion ......................................................................................................... 20-27
Unit 2.................................................................................................... 28-51
Voice & Accent .................................................................................................................. 28-30
Stress & Intonation ............................................................................................................ 31-36
Unit 3.................................................................................................... 52-76
Narration ........................................................................................................................... 52-60
Vocabulary Enrichment...................................................................................................... 61-76
Unit 4.................................................................................................. 77-104
Essay Writing ..................................................................................................................... 77-86
Reading Comprehension .................................................................................................. 87-104

Unit 5.................................................................................................. 88-122


Cover Letter ..................................................................................................................... 88-113
Para Jumbles ................................................................................................................. 114-122
Unit 6................................................................................................ 123-144
Critical Reasoning .......................................................................................................... 123-144

PEV 107 Page 2


Know your topics

• Types of errors
Sentence Correction • Strategies to solve

• Types of questions
Sentence Completion • Strategies to solve

• Introduction to vowels and consonants, Introduction to syllables


Voice & Accent • Stress & Intonation

• Direct Indirect speech


Narration • How to convert sentences

• One word substitution


Vocabulary Enrichment • Commonly misspelled words

• Types of essays
Essay Writing • Tips and tricks to write essays effectively

Reading • Types of passages


Comprehension • Types of questions

• Key elements of cover letter


Cover Letter • Format and sample cover letters

• Types of para umbles


Para Jumbles • Tips and tricks to solve para jumbles

• What is critical reasoning and its concepts


Critical Reasoning • Types of questions and strategies to solve

PEV 107 Page 3


SENTENCE CORRECTION
Introduction

 Words, phrases and sentences are the bases of any language.

 Knowing the common structures and the nuances of the language is crucial

 Sentence correction is a type of question

 Two variants - spotting error and improving the sentence

 Involve errors based on grammar concepts of the language being tested

Eight Types of Errors in Sentence Correction

 Subject-Verb Agreement

 Pronoun Agreement

 Modifiers

 Parallelism

 Comparisons

 Redundancy

 Error of Participles

 Verb Tenses

Subject - Verb Agreement

In the present tense, a verb must agree in number with its subject. That's the basic principle of subject-verb agreement.
It's a simple enough rule, but on certain occasions even experienced writers can slip up on it.

Let's have a look at three of the trickier cases of subject-verb agreement:

CASE #1: Making Subject and Verb Agree When Words Come Between Them

In determining subject-verb agreement, don't let yourself be confused by words that come between the subject and the
verb. Let's compare these two sentences:

 This box belongs in the attic.


 This box of ornaments belongs in the attic.

In both sentences, the verb “belongs” agrees with its subject, “box”. Don't let the prepositional phrase in the second
sentence fool you into thinking that “ornaments” is the subject. It's simply the object of the preposition “of” and does not
affect the agreement of subject and verb.

 One (of my sister's friends) is a pilot.


 The people (who survived the explosion) are in a shelter.
 A man (chasing unicorns) is on the terrace.

PEV 107 Page 4


CASE #2: Reaching Agreement When the Subject Is an Indefinite Pronoun

Remember to add an -s to the end of the verb in the present tense if the subject is one of the indefinite pronouns listed
below:

 anyone (everyone, no one, someone)


 anybody (everybody, somebody, nobody)
 anything (everything, something, nothing)
 each, either, neither

In the following sentences, each subject is an indefinite pronoun and each verb ends in -s:

 Nobody claims to be perfect.


 Everybody plays the fool sometimes.
 Each of the divers has an oxygen tank.

In that last sentence, note that “has” agrees with the subject “each”, not with “divers” (the object of the preposition).

CASE #3: Making Have, Do, and Be Agree With Their Subjects

In particular, many agreement errors result from the misuse of the common verbs “have, do, and be.”

We need to remember that the verb “have” appears as “has” if the subject is a singular noun or a third-person singular
pronoun (he, she, it):

 Dana Barrett has ghosts in her bedroom.

If the subject is a plural noun or the pronoun “I, you, we, or they”, use “have”:

 The Ghostbusters have a new client.

In a nutshell, "She has," but "They have."

Similarly, the verb “do” appears as “does” if the subject is a singular noun or, once again, a third-person singular pronoun
(he, she, it):

 Gus does the housework.

If the subject is a plural noun or the pronoun “I, you, we, or they”, use “do”:

 Gus and Martha do the chores together.

The verb “be” has three forms in the present tense: is, am, are. Use “is” if the subject is a singular noun or a third-person
singular pronoun (he, she, it):

 Dr. Venkman is unhappy.

Use “am” if the subject is the first-person singular pronoun (I):

 I am not the person you think I am.

Finally, if the subject is a plural noun or the pronoun “you, we, or they”, use “are”:

PEV 107 Page 5


 The fans are in the stands, and we are ready to play.

Now, let's take one more look at these three verbs—-but from a different angle.

Sometimes a subject may follow (rather than precede) a form of the verb “have, do, and be”. As shown in the sentences
below, this reversal of the usual order occurs in questions that require a helping verb:

 Where has Egon parked the car?


 What do you do in your free time?
 Are we having a test today?

In all of these sentences, the present forms of “have, do, and be” serve as helping verbs and appear in front of their
subjects. Another case in which a form of the verb comes before the subject is in sentences beginning with the
words “there or here”:

 There is a unicorn in the garden.


 Here are the photocopies.

Let’s Practice 1: The article on the subject of Colombian drug lords published in The Economist this morning neither allude
to nor specifically describe the methods that the police employs in the fight against crime.

A) neither alludes nor specifically describes the methods that the police employ in the fight against crime.
B) neither allude to nor specifically describe the methods that the police employ in the fight against crime.
C) neither alludes to nor specifically describes the methods that the police employs in the fight against crime.
D) neither alludes nor specifically describes the methods that the police employs in the fight against crime.
E) neither alludes to nor specifically describes the methods that the police employ in the fight against crime.

Pronoun Agreement
Correct pronouns match their antecedent (the word they stand in for). For example, in the sentence, “Sara took off her
hat,” “Sara” is the antecedent for “her.” Plural pronouns (we, they, them, us) match with plural antecedents, and
singular pronouns (I, he, she, her, him, it) match with singular antecedents.

Let’s look at one example: I never go to that restaurant because they have moldy cheese.

Who has moldy cheese? We can’t tell from the sentence. “They” is a plural pronoun, and its possible antecedents (“I” and
“restaurant”) are both singular. We would rewrite it like this:

I never go to that restaurant because it has moldy cheese.

Let’s Practice 2: While the Senate of the Roman Republic did not have the power to enact laws, their decrees generally
were obeyed like law, and through them they exercised considerable influence.

(A) their decrees generally were obeyed like law, and through them they
(B) its decrees generally were obeyed as if to be law, and through these it
(C) their decrees, generally obeyed as law, through it they
(D) its decrees generally were obeyed like law, and through them it
(E) their decrees were generally obeyed as was the law, and through it they
Modifiers

PEV 107 Page 6


Consider the following example: Looking out the window, the trees were seen by her.

Look at the part of the sentence before the comma. This is the modifier. It modifies or changes the rest of the sentence in
some way.

 What is the verb in the modifier? The verb is “looking out”.

 What is the subject of the verb? You may notice that in the modifier there is no subject for this verb. This makes the
modifier a dangling modifier. It is not connected to the rest of the sentence properly. The rule is that the correct
subject of the verb must come immediately after the comma.
 To determine what the subject is, you can ask yourself the question: Who or what performed the action? The answer
to that question gives you the subject. The answer is that “she” did the looking. It cannot be the trees looking out the
window. Therefore, “she” must come directly after the comma.

The correctly written sentence would become: Looking out the window, she saw the trees.

In a dangling modifier, the modifying phrase is misplaced, describing the incorrect word or phrase so that the sentence
is illogical. Take a look at the following sentence:

A beautiful red-haired mermaid, the president thought Ariel was highly intelligent.
This sentence contains a dangling modifier: The president is likely not a beautiful red-haired mermaid (though that would
be cool). The modifier “A beautiful red-haired mermaid” belongs next to the noun it modifies, which is Ariel. Let’s rewrite
it to remove the dangling modifier:

The president thought Ariel, a beautiful red-haired mermaid, was highly intelligent.
Let’s Practice 3: Between 1892 and 1893, Claude Monet produced a series of paintings of the Rouen Cathedral, revised in
his studio in 1894, and with the French public receiving it as an emblem of all that was noble about their history and
customs.

(A) produced a series of paintings of the Rouen Cathedral, revised in his studio in 1894, and with the French public receiving
it
(B) produced a series of paintings of the Rouen Cathedral, which he revised in his studio in 1894 and which the French
public received
(C) produced a series of paintings of the Rouen Cathedral, which he revised in his studio in 1894, and that the French public
received it
(D) painted the Rouen Cathedral, which he revised in his studio in 1894, and that the French public received it
(E) painted the Rouen Cathedral, revised in his studio in 1894, and the French public received it

Parallelism
While forming a sentence, the structure of the sentence should be kept parallel. If an infinitive is used, then all the phrases
should have an infinitive.
Example: She likes to cook, dance and play.

Similar rule is used for a gerund.

PEV 107 Page 7


Example: She likes cooking, dancing and playing.

Let’s Practice 4: With companies spending large parts of their advertising budgets online, the market for content such as
feature articles and opinion essays created by a professional writer, a blogger, and by individual users, are
expanding rapidly.

A. the market for content such as feature articles and opinion essays created by a professional writer, a blogger, and by
individual users, are expanding
B. the market for content such as feature articles and opinion essays created by professional writers, bloggers, and by
individual users, are expanding
C. the market for content such as feature articles and opinion essays created by a professional writer, a blogger, and
individual users, is expanding
D. the market for content such as feature articles and opinion essays created by professional writers, bloggers, and
individual users, are expanding
E. the market for content such as feature articles and opinion essays created by professional writers, bloggers, and
individual users, is expanding
Comparisons
The comparisons made should be between two similar things. If we say: - The population of London is greater than any
other city in India.
We are comparing: - (a) The population of London (b) Any other city in India.
While comparison had to be made between the populations of both. So, the correct expression should be:-
The population of London is greater than that of any other city in India.
(a) When comparative degree is used with than, make sure that we exclude the thing compared from the rest of class of
things by using the
Example: He is stronger than any man living.(incorrect).

He is stronger than any other man living.(correct).


Similarly, Solomon was wiser than all other men.
In superlative degree, we must include the thing compared.
Solomon was the wisest of all men.
He is the strongest of all men.
Let’s Practice 5: Like humans, who can recognize another’s knowledge and beliefs—an ability that may not be unique
to mankind—reading others’ intentions and displaying awareness of what others see seem within the capabilities of
some some non-human primates.
A. reading others’ intentions and displaying awareness of what others see seem within the capabilities of some some non-
human primates.
B. the capabilities of some non-human primates include reading others’ intentions and displaying awareness of what
others see.
C. some non-human primates seem capable of reading others’ intentions and displaying awareness of what others see.
D. some non-human primates‘ ability to read others’ intentions and displaying awareness of what others see.
E. non-human primates, reading others’ intentions, seem to have some abilities of displaying awareness of what others
see.

PEV 107 Page 8


Redundancy
This is the error of writing the same thing twice.
Example: a. He returned back from Delhi. B. I hardly have any money to give you.

The correct constructions should be;


a. He came back from Delhi. B. I have no money to give you.
Let’s Practice 6: Many house builders offer rent-to-buy programs that enable a family with insufficient savings for a
conventional down payment to be able to move into new housing and to apply part of the rent to a purchase later.

(A) programs that enable a family with insufficient savings for a conventional down payment to be able to move into new
housing and to apply
(B) programs that enable a family with insufficient savings for a conventional down payment to move into new housing
and to apply
(C) programs, which enable a family with insufficient savings for a conventional down payment to be able to move into new
housing, applying

Error of Participles

(Is the subject or doer close to the participle?)


Function of –ing verbs
What function do the –ing verbs do in the following sentences?

-Smoking is injurious to health. -Swimming is a good exercise. - He has given up smoking.

 They function as the subject or object in the sentence. If you ask the question ‘what is injurious or a good exercise’
or ‘what has he given up’ you will get the –ing words as answer. You should ask ‘what’ questions before the verb
to know the subject.

 While he was smoking he entered the hall.


 He entered the hall when he was smoking.

In the sentences above the words smoking and swimming don’t answer the question ‘what entered or who crossed’ but
tell how he entered or how they crossed.

These are different ways of combining two sentences.

Smoking is a participle here and the main function of a participle is to combine two sentences.

 The two sentences, in this case, are : He was smoking. At that very same time he entered the hall. Or,

He entered the hall. At that very same time he was smoking.

 Consider the other sentence: While swimming, they crossed the flooded river.
 If we split the same, we can say: They swam. This is how they crossed the river.

However, we can’t say the above sentences in the following manner:

Smoking the hall was entered by him. Or, (Incorrect)

Swimming the flooded river was crossed by them. (Incorrect)

PEV 107 Page 9


Consider the following sentences:

Writing a novel, his name became famous.

Having been a very good scientist, they made him the president.

 Ask ‘who wrote the novel his name or some person?’ It was some person. The sentence, therefore, should be
‘Writing a novel, he became famous’.
 In the second sentence, who was a scientist–they or the president? It was the president. Therefore, the sentence
should be ‘Having been a very good scientist, he was made president by them’.
 It is clear from the above discussion that the doer of the participle verb should not be distanced from it or
interrupted by any other subject, noun or verb.

Let’s Practice 7: After she attended the career fair, many more resources were at Ankita’s disposal, including job boards,
new contacts, and numerous books and pamphlets to help her improve her resume and cover letter.

A) many more resources were at Ankita’s disposal


B) at Ankita’s disposal were many more resources
C) there were many more resources at Ankita’s disposal
D) Ankita, at her disposal, had many more resources
E) Ankita had many more resources at her disposal

Verb Tenses
Sequence of Tenses Is the principle in accordance with which the tense of the verb in a subordinate clause follows
the tense of the verb in the Principal clause.

1. When verb in Principal clause is in Present or Future tense, the verb in the dependent clause may be in any
tense (according to sense).

 1. He says that he likes music. Or He will say that he likes music.


 2. He says that he liked music. Or He will say that he liked music.
 3. He says that he will work hard. Or He will say that he will work hard.

2. When the verb in the Principal clause is in the Past Tense, the verb in the Dependent clause must also be in one
or other of the four forms of the past Tense.

1. He said that he was late.


2. You said that you would help him.
3. He worked hard so that he might pass.
4. He said that you were working hard.

EXCEPTIONS:

3. If the Dependent clause states a universal truth or a habitual truth, a Past Tense in the Principal clause is
followed by Present Tense in the Dependent clause.

1. He said that the sun rises in the east.


2. You said that you are a vegetarian.

PEV 107 Page 10


4. A Past Tense in the Principal Clause is followed by any tense in the Adverbial clause beginning with ‘than’ as

 I valued your friendship more than (I value) his.


 He liked you more than I did (like you).
 He liked you more than I do (like you).
 I then saw him oftener than I see him now.

5. A Past tense in the Principal Clause may be followed by any tense in the adjectival clause as:

 I visited the place where he lives.


 I visited the place where he lived.
 I visited the place where he will live

Let’s Practice 8: Shortly after their first expeditions to the region, the Spanish initiated a number of attempts to subjugate
the Maya, but it takes some 170 years before the Spanish established substantive control over all Maya lands.

a) but it takes some 170 years


b) but they would take some 170 years
c) but it took some 170 years
d) but it was going to take some 170 years

Correlative conjunction

When you use correlative pairs of conjunctions in a sentence, make sure that the words or
groups of words immediately following each conjunction are in the same form.

 Incorrect: Either I will attend the show, or they will be attending.


 Correct: Either I will attend the show, or they will. (or)
 Either I will be attending the show, or they will be attending.

Order of personal pronoun

For all good things: You, he and I have achieved remarkable success.
For all bad things: I, you and he are responsible for the failure of the project.

ORDER OF ADVERB

The order of the adverb: MPT = Manner–Place–Time


Manner = slowly, steadily
Place = here, there, up, down
Time = before, ago, now, today

PEV 107 Page 11


Tutorial
EXERCISES: Beginner

1.The three friends, Max included, was supposed to meet for dinner later that night.
A) was supposed to meet
B) was supposed to have met
C) were suppose to be meeting
D) were supposed to meet
E) they were supposed to be meeting

2.The number of students chosen for the prestigious medical internship have more than doubled in the past fifteen years.
A) have more than doubled
B) have been more than doubling
C) has more than doubled
D) has been more than doubling
E) has doubled even more

3.Following intense debate, the faculty has approved the measure to increase class size by 15% over the next four years.

A) the faculty has approved the measure to increase


B) the faculty has approved the measure and increased
C) the faculty have approved the measure to increase
D) the faculty have given their approval to the measure to increase
E) the faculty, having approved the measure to increase

4.Without proper funding and a better campaign strategy, there is no chances that our candidate will be elected to office.

A) there is no chances that


B) there can be no chance for
C) there is no chance that
D) there are no chances for
E) there will be no chances for

5.Some members of the tribe has been protesting the recent passage of hunting laws applying to indigenous populations.

A) members of the tribe has been protesting


B) members of the tribe have been protesting
C) tribe members has been protesting
D) tribe members will have been protesting
E) members of the tribe, having protested

6.The results of the study clearly indicates a reduction in the number of useable pounds that can be salvaged from an average ton of
recyclable goods.

A) indicates a reduction
B) indicates that a reduction
C) indicating a reducing
D) indicate a reducing
E) indicate a reduction

7.The president of Costa Rica, along with two vice presidents, are elected for a four-year term by the people.

PEV 107 Page 12


A) are elected for a four-year term by the people
B) are elected, by the people, for a four-year term
C) is elected for a four-year term by the people
D) are elected for four-year terms by the people
E) is elected for four-year terms by the people

8.Neither of our school’s students nominated for the national spelling bee were able to win the competition

A) Neither of our school’s students nominated for the national spelling bee were
B) Neither of our school’s students nominated for the national spelling bee was
C) Neither of the students from our school nominated for the national spelling bee were
D) Neither of the students nominated for the national spelling bee from our school were
E) Neither one of our school’s students who was nominated for the national spelling bee was

9. His co-workers praised both his determination and the way he paid attention to detail.

A) and the way he paid attention to detail


B) and also praised his attention to detail
C) and his attention to detail
D) they praised the way he paid attention to detail
E) also they praised his attention to detail

10. The art studio is spacious, pleasantly cluttered, and has good lighting.

A) and has good lighting


B) and being well-lit
C) and is lit well
D) and well-lit
E) and the lighting is good

11. The school board requested that a waiver be obtained and that the residency requirements are reviewed.

A) that the residency requirements are reviewed


B) the residency requirements will be reviewed
C) the residency requirements reviewed
D) to review the residency requirements
E) a review of the residency requirements

12. Richard is not only a terrific pianist, but also great at playing hockey.

A) Richard is not only a terrific pianist, but also great at playing hockey.
B) Richard not only is a terrific pianist, but is also great at playing hockey.
C) Not only great at playing hockey, Richard also is a terrific pianist.
D) Richard is not only a terrific pianist, but also a great hockey player.
E) Also great at playing hockey, Richard is a terrific pianist.

13. I don’t remember whom has the most stock in the company.

A) whom has the most stock


B) who have the most stock
C) whom have the more stock
D) who has the most stock
E) whose the most stock

PEV 107 Page 13


14. Anyone who chooses to be part of our coalition to defend homeless people are required to sign several documents.
A) people are required to sign
B) people is required to sign
C) person is required to sign
D) people are required, signing
E) people is required, signing

15. Larry was entertained and enlightened by the scintillating presentation.

A) was entertained and enlightened


B) entertained and was enlightened
C) entertained and enlightened
D) was entertaining and enlightened
E) will entertain and enlighten

EXERCISES: Intermediate

1. The Egyptian Museum in Cairo, which contains 120,000 objects from prehistoric times through the Greco-Roman period, are
home to one of the most impressive collections of ancient Egyptian artifacts.

A) which contains 120,000 objects from prehistoric times through the Greco-Roman period, are home to
B) which contain 120,000 objects from prehistoric times through the Greco-Roman period, are home to
C) containing 120,000 objects from prehistoric times through the Greco-Roman period, are home to
D) which is containing 120,000 objects from prehistoric times through the Greco-Roman period, are home to
E) which contains 120,000 objects from prehistoric times through the Greco-Roman period, is home to

2. The professor's consistent late arrival is offset somewhat by the remarkable quality of his lectures.

A) The professor's consistent late arrival


B) The consistent late arrival of the professor
C) The professor's consistently late arrival
D) Lately, the professor's arriving consistently
E) The professor's consistent late arriving

3. The concerto sounds more sophisticatedly in the 200-year-old concert hall than it did in the practice room, which has decidedly
inferior acoustics.

A) sounds more sophisticatedly


B) sound more sophisticatedly
C) sounds with greater sophistication
D) sounds more sophisticated
E) sound more sophisticated

4. Previously thought to have been extinct, a team of biologists rediscovered the New Caledonia crested gecko in 1994.

a) a team of biologists rediscovered the New Caledonia crested gecko in 1994.


b) a team of biologists, in 1994, rediscovered the New Caledonia crested gecko.
c) in 1994 the New Caledonia crested gecko was rediscovered by a team of biologists.
d) in 1994 a team of biologists rediscovered the New Caledonia crested gecko.
e) the New Caledonia crested gecko was rediscovered by a team of biologists in 1994.

5. Some of the many renovations set for Memorial Field in the coming years include building additional seating, improving
safety, and the construction of a new varsity athletics centre.

PEV 107 Page 14


A) and the construction of a new varsity athletics center
B) and constructing a new varsity athletics center
C) and also the construction of a new varsity athletics center
D) and a new varsity athletics center
E) and a new varsity athletics center under construction

6. Valerie recalls her college years with such nostalgia that she often lost herself in reminiscence.

A) she often lost herself in reminiscence


B) she often had lost herself in reminiscence
C) she often loses herself in reminiscence
D) she often will be losing herself in reminiscence
E) she often will have lost herself in reminiscence

7. Even though he does not like crowds, John still likes New York City more than Akshay.

A) John still likes New York City more than Akshay


B) New York City is still liked more than Akshay by John
C) John is still liking New York City more than is Akshay
D) John still likes New York City more than Akshay does
E) New York City is still more liked by John than Akshay

8. Both candidates oppose increased defence spending. Neither of the two candidates oppose the war in Iraq.
(a) Neither of them oppose the war
(b) Neither of the two candidates are opposed to the war
(c) Neither of the two candidates is opposed to the war
(d) Either of the two candidates are not opposed to the war

9. Of the numerous decisions a student has to take, the question of which career path to choose is for certain the
more confusing.
(a) the question of which career path to choose is certainly the most confusing
(b) it seems certain that the more confusing is the question of which career path to choose
(c) certainly choosing their career path is the most confusing
(d) the question of which career path to choose is certainly the more confusing

10. Recent historians have begun to reevaluate the career of General William Tecumseh Sherman, paint him as a war criminal and a
monstrous villain.

a) paints him as a war criminal and a monstrous villain.


b) paint him as the war criminal and the monstrous villain.
c) paint him as a war criminal and a monstrous villain.
d) paint him for a war criminal and a monstrous villain.
e) painting him as a war criminal and a monstrous villain.

11. The new regulations are largely meant to curtail small-scale corruption, but the loopholes present in the law created a sharp
increase in white collar crime.

a) The new regulations largely meant to curtail small scale corruption


b) The new regulations are largely meaning to curtail small scale corruption
c) The new regulations are largely meant for curtailing small scale corruption
d) The new regulations were largely meant to curtail small scale corruption
e) The new regulations are largely meant to curtail small scale corruption

12. Very few politicians are willing to make a promise to eliminate social safety net programs, which made the congressman a rarity
in Washington.

a) which having made the congressman a rarity in Washington.


b) which had made the congressman a rarity in Washington.
PEV 107 Page 15
c) which made the congressman a rarity in Washington.
d) which makes the congressman a rarity in Washington.
e) which making the congressman a rarity in Washington.

13. The teacher sought new ways of teaching, believing her students would get more from lessons featuring playing, acting, and
creativity.
a) featuring playing, acting, and creating.
b) featuring play, act, and creativity.
c) featuring the playing, acting, and creativity.
d) featuring playing, acting, and creativity.
e) featuring playing, acting, and creative things.

14. The kids took too much time cleaning their rooms, left little time to do outdoor chores.
a) left little time to had done outdoor chores.
b) left little time to do outdoor chores.
c) leaving little time to do outdoor chores.
d) left few times to do outdoor chores.
e) leaving little time to doing outdoor chores.

15. All people choose their own religious denomination when they have been adults.
a) when they have been adults.
b) they have been adults.
c) when adults they have been.
d) when being adults.
e) when they are adults.

16. The United States Navy announced that, beginning next year, they plan to close several of their bases in order to reduce
operating expenses.

A. they plan to close several of their bases


B. they are planning to close several of their bases
C. it plans to close several of its bases
D. they plan several closures of their bases
E. it plans to close several of their bases

17. A good writer can avoid these errors unless they haven’t internalized the rules of grammar.
a) A good writer can avoid these errors unless they haven’t internalized the rules of grammar.
b) A good writer can avoid these errors provided they haven’t internalized the grammar rules.
c) A good writer can avoid these errors unless they have internalized the rules of grammar.
d) A good writer can avoid these errors after having internalized the rules of grammar.
e) A good writer can avoid these errors while not having internalized grammar rules.

18. With no previous experience in politics, Harvey’s campaign is more popular than other candidates.

A. With no previous experience in politics, Harvey’s campaign is more popular than other candidates.
B. With no previous experience in politics, Harvey and his campaign is more popular than other candidates.
C. With no previous experience in politics, Harvey’s campaign more popularly than other candidates.
D. With no previous experience in politics, the campaign is more popular with Harvey than other candidates.
E. With no previous experience in politics, Harvey is more popular than other candidates.

19. Because of a server delay in delivering her reply, he thought she hadn’t received his e-mail.

A. Because of a server delay in delivering her reply, he thought she hadn’t received his e-mail.
B. Because of a server delay in delivering her reply; he thought she hadn’t received his e-mail.
C. Because of a server delay in delivering her reply he thought she hadn’t received his e-mail.
D. Because of a server delay in delivering her reply—he thought she hadn’t received his e-mail.

PEV 107 Page 16


20. Professors operating fairly know that he or she cannot base grades on emotions or personality.

A. Professors operating fairly know that he or she cannot base grades on emotions or personality.
B. Professors operating fairly know that they cannot base grades on emotions or personality.
C. Professors operating fairly know that he cannot base grades on emotions or personality.
D. Professors operating fairly know that she cannot base grades on emotions or personality.

EXERCISES: Advanced

1. Reporting a 90 percent drop in net income during the second quarter, dragged down by restructuring charges and weak sales, the
earnings guidance for the year was withdrawn by Best Buy Co.

a) Reporting a 90 percent drop in net income during the second quarter, dragged down by restructuring charges and weak sales,
the earnings guidance for the year was withdrawn by Best Buy Co.
b) Best Buy Co. is reporting a 90 percent drop in net income during the second quarter, dragged down by restructuring charges and
weak sales; the company also withdrew its earnings guidance for the year.
c) Reporting a 90 percent drop in net income during the second quarter and dragged down by restructuring charges and weak sales,
Best Buy Co. withdrew the earnings guidance for the year.
d) Best Buy Co. has reported a 90 percent drop in net income during the second quarter, dragged down by restructuring charges
and weak sales; the company has also withdrawn its earnings guidance for the year.
e) Best Buy Co. is reporting a 90 percent drop in net income during the second quarter, dragged down by restructuring charges and
weak sales; the company has also withdrawn its earnings guidance for the year.

2. The philosophical doctrine of Incompatibility posits an inherent irreconcilability among the doctrine of Determinism, which holds
that each state of affairs is necessitated by the states of affairs that preceded it, and the existence of free will.

A) among the doctrine of Determinism, which holds that each state of affairs is necessitated by the states of affairs that preceded it,
and the existence of free will
B) between the doctrine of Determinism, holding each state of affairs as necessitated by the states of affairs that preceded it, and free
will existing
C) in the doctrine of Determinism, which holds the idea that each state of affairs is necessitated by the states of affairs preceding, and
the existence of free will
D) between the doctrine of Determinism, which holds that each state of affairs is necessitated by the states of affairs preceding it, and
the existence of free will
E) among the doctrine of Determinism, which holds that each state of affairs may be necessitated by the states of affairs preceding it,
and free will existing

3. Erasmus's tomb lies inside the Basel Munster, located in Switzerland, an architectural monument which having survived medieval
earthquakes, and remains one of Switzerland's most well-known buildings to this day.

A) Erasmus's tomb lies inside the Basel Munster, located in Switzerland, an architectural monument which having survived medieval
earthquakes, and
B) Erasmus's tomb lies inside Switzerland's Basel Munster, an architectural monument that survived medieval earthquakes and
C) Switzerland's Basel Munster, an architectural monument that survived medieval earthquakes, houses Erasmus's tomb,
D) The Basel Munster, in Switzerland, an architectural monument which, having survived medieval earthquakes, is now home to the
tomb of Erasmus and
E) The tomb of Erasmus, being housed inside Switzerland's Basel Munster, is an architectural monument that survived medieval
earthquakes and

4. Simon Bolivar (1783 – 1830) is remembered in that he led the independence revolutions in several South American counties, like
Venezuela and Bolivia, and for instilling the ideals of democracy across the continent.

(A) in that he led the independence revolutions in several South American counties, like Venezuela and Bolivia, and for instilling

(B) to have led the independence revolutions in several South American counties, such as Venezuela and Bolivia, and that he instilled

(C) to have led the independence revolutions in several South American counties, including Venezuela and Bolivia, and having instilled
PEV 107 Page 17
(D) for leading the independence revolutions in several South American counties, like Venezuela and Bolivia, and to have instilled

(E) for leading the independence revolutions in several South American counties, such as Venezuela and Bolivia, and for instilling

5. While larger banks can afford to maintain their own data-processing operations, many smaller regional and community banks are
finding that the cost associated with upgrading data-processing equipment and with the development and maintenance of new
products and technical staff are prohibitive.

(A) cost associated with (B) costs associated with (C) costs arising from
(D) cost of (E) costs of

6. Inertia-gravity waves cause characteristic stripy patterns in the clouds in the lower atmosphere but they are disregarded by
conventional weather forecasts because they are thought to be too small to interact with larger systems such as warm and cold
fronts.

A. they are disregarded by conventional weather forecasts because they are thought to be too small
B. they are disregarded by conventional weather forecasts because these waves are thought to be too small
C. conventional weather forecasts disregard them because they think they are too small
D. conventional weather forecasts disregard these waves because they are thought to be too small
E. conventional weather forecasts think them too small

7. The recent photographs of the giant squid are remarkable because they show these enormous living creatures as moving around
in their natural environment, whereas previous pictures have been of only dead animals.

A. because they show these living creatures as moving around in their


B. in that they show this most enormous of living creatures moving around in its
C. in that they show this enormous living creature moving around in its
D. because these enormous living creatures are shown to be moving around in their
E. because they show this enormous living creature moving around in a

8. It is usual for scientists and social scientists to abandon their theories only if another more attractive theory comes along and not
when they have been proved incorrect.

A. their theories only if another more attractive theory comes along and not when they have been proved incorrect
B. a theory not because it has been proved incorrect, but because another more attractive theory comes along
C. their theories not when they have been proved incorrect, but because other more attractive theories come along
D. a theory only when other more attractive ones come along, rather than when they are proved wrong
E. their theories not if they have been proved wrong, but if another more attractive theory comes along

9. Once a hurricane is identified, it is given a name from a list drawn up by the United States Weather Service, a list that is reused
after a few years, but with the names of the worst hurricanes omitted.

A. but with the names of the worst hurricanes omitted


B. omitting the names of the worst hurricanes
C. the names of the worst hurricanes being omitted
D. after they have omitted the names of the worst hurricanes
E. after omitting the worst hurricane names

10. Dunbar argues that gossip is important in human societies in the maintenance of social cohesion, just as social grooming does
for other primates.

A. gossip is important in human societies in the maintenance of social cohesion, just as social grooming does for other primates
B. gossip is important in the maintenance of social cohesion in human society, just as social grooming does for other primates
C. in human societies gossip is important in social cohesion like social grooming for other primate societies
D. similar to social grooming in primates, human gossip is important in maintaining social cohesion
E. gossip is important in human societies in the maintenance of social cohesion, just as social grooming is in other primate groups

11. Ricks has written extensively on not only major figures in English poetry like Milton and Housman, but also on the lyrics of Bob
Dylan.

PEV 107 Page 18


A. on not only major figures in English poetry like Milton, but also on
B. not only on the poetry of such major figures as Milton and Housman, but also on
C. not only on major figures in English poetry like Milton and Housman, but also on
D. on major figures in English poetry like Milton and Housman, as well as
E. on major figures in English poetry such as Milton and Housman, but also on

12. An analysis of sixteenth century probate inventories in the major English towns show that even some artisans and yeomen
owned silver spoons, cups or salt cellars.

A. show that even some artisans and yeomen owned silver spoons, cups or
B. show that some artisans and yeomen even owned silver spoons, cups or
C. show that even some artisans and yeomen owned silver spoons, cups and
D. shows that some artisans and yeomen owned even silver spoons, cups and
E. shows that even some artisans and yeomen owned silver spoons, cups or

13. Stress-induced amnesia is a rare phenomenon; it strikes the patient apparently without warning and the memory loss can be as
complete as that induced by physical trauma.

A. as complete as that induced by physical trauma


B. as complete as is induced by physical traumas
C. as least as complete as physical trauma induces
D. at least as complete as physical trauma
E. just as complete as those induced by physical trauma

14. It is probable that the prototype cellular motor might be ready for testing around the end of next year.

A. might be ready for testing around the end of next year


B. may be ready for testing about he end of next year
C. might be ready for testing toward next year’s end
D. will be ready for testing toward the end of next year
E. should be ready for testing toward the end of next year

15. Making use of contemporary diaries and letters, Milton’s complexity is brilliantly revealed by Amit.

A. Milton’s complexity is brilliantly revealed by Amit


B. Milton is brilliantly shown in all his complexity by Amit
C. Amit brilliantly reveals Milton’s complexity
D. Amit brilliantly revealed the complexity of Milton
E. Amit’s study has brilliantly revealed the complexity of Milton

PEV 107 Page 19


Sentence Completion
What are Sentence Completion Questions?

• They are nothing but the good old ‘Fill in the Blanks’ type of questions we’ve been handling since primary school!
• The only difference is – the ‘test’ is of a slightly different kind when it comes to your level today; and what companies/
recruiters test in you using this variety of questions.

What do they test?


 Vocabulary
 Not just ‘meanings’ of words, but also their fine usage
 Your ability to mark logical consistency among given elements of a sentence.
 Your ability to grasp how words fit into various contexts
 Sometimes, your mere knowledge of how parts of sentence affect each other can help you determine which option to pick; .You
may not necessarily know the meanings of the options given!!

Question types:
1. Single Blank
2. Double blank
3. Closed Text (Multiple Blanks)

Tips to solve Sentence Completion Questions:

Read the sentence carefully for meaning


You are never going to determine the answer until you understand the question. Think about the sentence means and what part of
speech is necessary to correctly complete the sentence.

Visualize
Before you go to the choices, think of the possible words for the blanks. It will save you from wrong choices. This is better than trying
out the choices to find out "what sounds good." It is faster and less prone to errors.

Structure Words
Look for words like but, rather, although, however, and, while, but, therefore. They reveal the sentence organization and the hint-
blank relationship. They tell you what kinds of words to look for, as they change the thought process in the sentence.

Always read all the answer choices


In sentence completion section, if you are strapped for time and you select “A” because it works without checking the other choices,
it just may be that answer choice “E” was an even better selection.

Use process of elimination


This is especially true of the questions with 2 blanks. This essentially doubles the chances you will know at least one of the 2 words in
the answer choices and that you can eliminate the choice from consideration if the words do not make sense.

Improve Your Vocabulary


Improving your vocabulary and usage can help you do better as the word meanings help you find the right answer.

Working Backwards
The two-blank questions can be easier as you have more opportunities to eliminate wrong choices. If you can eliminate a choice
based on one word, you don't need to know the other word. Often, working backwards i.e. picking the second blank choice first
works better.

Positive/Negative Flow
When you read the sentence, you have to look out for adjectives/adverbs which tell you the idea of the sentence. After finding these
adjectives/adverbs, you need to find out if the idea of the sentence is positive/negative. All the negative ideas may be a "bad
word/bad phrase" or any term which has no/none/not... in it. In simple words, if the flow of the first part of the sentence is positive
and the second part is negative, then the blank must be negative to even the flow of the sentence. This would solve the sentence
completion question without even understanding the question.

PEV 107 Page 20


Punctuation:
Whenever the punctuation "," (comma) appears, followed by a blank in between two sentences, then it means that the synonym of
the phrase/word before "," is the meaning of the blank. In simple words, when you find ',' followed by a blank then find the synonym
of the word before ',' and check the options to match the synonym of the word.
In the same way, when you find ":"( colon) or ";"( semi-colon) in the sentence, they will indicate that the idea coming up is merely an
explanation of the earlier idea. So, simply find the synonym of the word/phrase before the punctuation and fill in the blank with the
synonym from the options given.

Transitional Words
Be alert to transitional words. Transitional words tell you what is coming up. They indicate that the author is now going to draw a
contrast with something stated previously, or support something stated previously.

i. Contrast Indicators:

To contrast two things is to point out how they differ. In this type of sentence completion problem, we look for a word that has the
opposite meaning (an antonym) of some key word or phrase in the sentence.

Following are some of the most common contrast indicators:


But Yet Despite Although However Nevertheless

Example: Although the warring parties had settled a number of disputes, past experience made them _________ to express
optimism that the talks would be a success.

A. rash B. ambivalent C. scornful D. overjoyed E. reticent

"Although" sets up a contrast between what has occurred--success on some issues--and what can be expected to occur--success for
the whole talks. Hence, the parties are reluctant to express optimism. The common word "reluctant" is not offered as an answer-
choice, but a synonym--reticent--is. The answer is (E).

ii. Support Indicators:

Supporting words support or further explain what has already been said. These words often introduce synonyms for words elsewhere
in the sentence.

Following are some common supporting words:


And Also Furthermore Likewise In Addition For

Example: Davis is an opprobrious and ________ speaker, equally caustic towards friend or foe--a true curmudgeon.

A. lofty B. vituperative C. unstinting D. retiring E. laudatory

"And" in the sentence indicates that the missing adjective is similar in meaning to "opprobrious," which is very negative. Now,
vituperative--the only negative word--means "abusive." Hence, the answer is (B).

iii. Cause And Effect Indicators:

These words indicate that one thing causes another to occur. Some of the most common cause and effect indicators are

Because For Thus Hence Therefore If , Then .

Example: Because the House has the votes to override a presidential veto, the President has no choice but to ________.

PEV 107 Page 21


A. object B. abdicate C. abstain D. capitulate E. compromise

Since the House has the votes to pass the bill or motion, the President would be wise to compromise and make the best of the
situation. The answer is (E).

Tutorial
Exercises: Beginner

Complete the sentences with the most appropriate options.

1. Even though the two parties appear united in negotiations, ----.

A) the level of trust between them will always remain, at best, tenuous
B) they weren’t able to agree on the subject
C) they will eventually reach an agreement
D) it is impossible for him to succeed
E) the president unexpectedly rejected it

2. The moment I saw the sad face of my girl friend, ----.

A) I begin to feel depressed


B) I have realized that we will have a discussion about our relationship
C) I had decided to leave the house
D) I found out that something was wrong
E) she has gone out to have a walk

3. You had better take your mobile phone with you ----.

A) so that you can lose it


B) or I couldn’t have called you
C) because I will not be at home during the whole day
D) If you had wanted to go abroad
E) in case you may not find one when you are in need of it

4. ---- when they learned that the chairman would not be able to join the meeting.

A) Hardly had the committee learned the reason of the meeting


B) When they realized why they were all there in that early time of the day
C) It wasn’t until they got a phone call about an urgent meeting the next day
D) However professional they tried to be seen
E) They will have already discussed the most important subjects

5. ----, the Grammys are the highest rated.

A) There are many music awards shows in the US


B) Although Grammys are considered to be highly prestigious
C) Because there is only one big music awards show in the US
D) Much as people are looking forward to next Grammy awards show
E) Of the "big three" music awards shows

Directions: Complete the sentence using the word or set of words for each blank that best fits the meaning of the sentence as a
whole.

1. Although it’s not remotely frightening in the daylight, the estate takes on ______ quality at night.

A. an eerie B. a stately C. a jovial D. an optimistic E. a marvelous

PEV 107 Page 22


2. I have always preferred ______ areas to ______ ones, because I really like the countryside.

A. city … downtown B. rural … urban C. attractive … beautiful D. quiet … tranquil E. magnificent … majestic

3. Though the accident damaged the car pretty severely, mechanics were able to ____ it and make it almost good as new.

A. destroy B. build C. salvage D. hinder E. cure

4. Silvio first arranged the anthology in ______ order based on when the writers published each piece, but he later switched to
______ ordering system based on the spelling of the authors’ names.

A. reverse … a straightforward B. random … an orderly C. mathematical … a geometric


D. chronological … an alphabetical E. original … a superior

5. The other children did not want Alexander to play with them, so they ______ him from their game.

A. exchanged B. exhibited C. exposed D. examined E. excluded

6. Apart from the one small river running through it, the desert is entirely ______.

A. arid B. humid C. remote D. temperate E. moist

7. Eleanor is ______ peanuts: the smallest taste of peanut butter can stop her breathing and put her in the hospital.

A. enamored of B. allergic to C. intrigued by D. interested in E. fatal to

8. The dancer tried to ______ the movements of his teacher, exactly copying every graceful step.

A. plagiarize B. interpret C. mimic D. possess E. refine

9. The professor told her students about the project months in advance so that they would have ______ time to complete their
work.

A. chronological B. constant C. insufficient D. ample E. standard

10. Because the soldier fought so valiantly, he was ______ by the president who awarded him an honorary medal.

A. commended B. rejected C. encountered D. ambushed E. chastised

11. Stalin often purged the Communist Party of his enemies and ______ in order to eliminate ______ and maintain his control over
the party.

A. supporters … dispute B. maniacs … rebellion C. fugitives … power


D. associates … cooperation E. rivals … dissent

12. Northampton High School is ______: students come from 24 different countries and speak 15 languages.

A. diverse B. uniform C. local D. similar E. identical

13. A new rule prohibits students from ______ on campus after school, so, now, all students must leave campus by 4 PM.

A. arriving B. remaining C. dining D. returning E. fighting

14. My mother’s ______ salary never left us with enough money to afford luxuries such as vacations, new cars, or nice clothes.

A. meager B. unlimited C. supportive D. tremendous E. prosperous

15. Trying to teach a dog to speak English is ______ task; the creature will never be able to use language like we do.

PEV 107 Page 23


A. a worthwhile B. an admirable C. a futile D. a respectable E. a fruitful

Exercises: Intermediate

1. Elise always envisioned the monastery as an austere place of worship; however, upon visiting it, she found it surprisingly
______.
i. comfortable
ii. barren
iii. strict
iv. ornate
v. simple

2. Although it is necessary to carry a relatively large number of provisions when traversing the Australian Outback, it is ______
that you keep your pack from becoming too ______.
i. crucial...ponderous
ii. mandatory...insulated
iii. helpful...elongated
iv. imperative...compact
v. important...convoluted

3. After living a life of depravity and transgression, the offender felt so ______ that he declared he would become a priest, and
devote the remainder of his life to ______.
i. melancholy...sadness
ii. tentative...shame
iii. terrible...sin
iv. contrite...atonement
v. stolid...repentance

4. He vowed to embrace a newfound ______ once the trial began; nonetheless the accused resorted to his typical manner of
______ as soon as he took the stand.
i. ingenuousness…naïveté
ii. mendaciousness...deceitfulness
iii. passion...exuberance
iv. candor...duplicity
v. residence...decrepitude

5. Mr. Plainview is a man of secrecy. He deals with the mob and other ______ organizations, and regularly participates in their
______ activities.
i. clandestine...unlawful
ii. anarchistic...fraudulent
iii. amiable...illegitimate
iv. disdainful...scrupulous
v. illegal...exhilarating

6. Even the most ______ gambler stops betting when he or she runs out of money.
i. intelligent
ii. cautious
iii. hazardous
iv. foolhardy
v. circumspect

7. Upon hearing the ______ argument for the opposition, Mr. Algene felt ______ that he would win the debate.
i. dubious...uncertain
ii. substantial...sure
iii. deplorable...convinced
iv. tenuous...confident
v. hardy...positive

PEV 107 Page 24


8. The lifestyle of a monk is ______; one must devote oneself to religious exercise, self discipline, and abstention from material
satisfaction.
i. dull
ii. ascetic
iii. lachrymose
iv. harsh
v. prodigal

9. The orator's speech was too ______; it would have been more succinct if she avoided discussing ______ subjects.
i. complicated...germane
ii. prolix...tangential
iii. resplendent...pertinent
iv. convoluted...complex
v. terse...florid

10. Don was the most ______ individual I had ever met; good fortune eluded him at every turn.
i. contentious
ii. auspicious
iii. venerable
iv. hapless
v. ignominious

11. Due to the workers’ ______ and unremitting work ethic, the bridge was built in under one month.
i. assiduous
ii. laborious
iii. stolid
iv. pedantic
v. jovial

12. It is not a good idea to befriend ______ animals; they are exposed to foreign environments and therefore may carry diseases
that the human immune system has not evolved to combat.
i. domesticated
ii. ponderous
iii. feral
iv. indigenous
v. endangered

13. Although my sister’s ______ with the rock star seemed ridiculous, she actually ______ her dream of going on a date with him
last night.
i. obsession...engaged
ii. fixation...participated in
iii. infatuation...fulfilled
iv. fascination...obliterated
v. affair...succumbed to

14. With such a(n) ______ personality, it is difficult to imagine what made Amanda feel so melancholy.
i. ebullient
ii. exciting
iii. passionate
iv. seditious
v. irascible

15. Being the ______ host that she was, Anika made sure to approach each lodger with ______ disposition.
i. convuvial...an affable
ii. churlish...a reputable
iii. engaging...a specious
iv. wicked...benign
v. amiable...a scrupulous

PEV 107 Page 25


Exercises: Advanced

1) In order to appreciate the (i) ______________ of damaging the ozone layer on earth, it is important to realize
that our planet is built upon relationships between different elements, and that damage to any one component
may result in (ii) ______________ effects to many others.

Blank (i) Blank (ii)


A. repercussions D. distribution
B. factors E. deleterious
C. statistics F. angst

2) Most high quality short stories written before the birth of Christ have traditionally been credited to men rather
than women by most historians; but this dominance has, in modern times, been attributed to (i) ______________
bias rather than a true reflection of (ii) ______________ quality.

Blank (i) Blank (ii)


A. patriotic D. inherent
B. evenhanded E. recognition
C. impeachable F. deterministic

3) In many cases, data is more enjoyable to share than theories, but interestingly, the probability of errors is much
greater when sharing information that supposedly contains (i) ______________ pieces of truth than it is when
(ii) ______________ on a potential model of understanding.

Blank (i) Blank (ii)


A. idealistic D. separating
B. prevaricated E. exchanging
C. invariable F. surmising

4). Artists consider unintentional surrealism in their work to be an abomination rather than a gift; in fact, many
creative souls, by nature, consider themselves to be (i)______________ monitoring their ideas like little children,
and making sure that all of the written work is in line with (ii) ______________ meanings.

Blank (i) Blank (ii)


A. progenitors D. intended
B. tyrants E. potential
C. dissidents F. opportunistic

5) Heald’s latest literary critique (i) ______________ his reputation for brilliance, and begins with a clever
title and attractive book cover, (ii) ______________ the cliche that we should not judge a book by mere
(iii) ______________.

Blank (i) Blank (ii) Blank (iii)


A. underscores D. instructing G. truancy
B. mixes E. demonstrating H. aesthetics
C. obfuscates F. belying I. content

6) Despite legal recourse for nonpayment of taxes being (i) ______________ in governments, the
expense of enforcing these payments is sometimes more costly than the (ii) ______________ itself.

Blank (i) Blank (ii)


A. obsolete D. recompense
B. considered E. proffer
C. ubiquitous F. dissent

PEV 107 Page 26


7). Before the work of Merck, pictures of people could be taken but rarely (i) ______________ natural
beauty; this fact, combined with the (ii) ______________ nature of most photographic processes in the early
1900’s often kept photographers from generating the (iii) ______________ of great work so common today.

Blank (i) Blank (ii) Blank (iii)


A. witnessed D. encumbering G. artlessness
B. captured E. striated H. difficulty
C. encountered F. polemical I. volume

8) Andrew’s caustic temperament and extreme views can turn off the most patient political adviser; he even
angered a board of ______________ moderates.

A. evenhanded
B. didactic
C. equivocal
D. ostentatious
E. polemical

9) Many times the work of a fledgling writer is clearly (i) ______________ even when the work as a whole
is (ii) ______________, since a lack of skill does not always equate to a boring read.

Blank (i) Blank (ii)


A. brilliant D. intriguing
B. incompetent E. ridiculous
C. substantive F. archaic

10) It is hypocritical to be the type of leader who is very (i) ______________ about what you know, especially
when you do not (ii)______________ others this privacy because you are constantly (iii) ______________
information about their personal goals.

Blank (i) Blank (ii) Blank (iii)


A. outspoken D. link G. selling
B. taciturn E. distribute H. fabricating
C. spontaneous F. afford I. soliciting

PEV 107 Page 27


VOICE & ACCENT
What is Accent?

The term accent has various meanings, but in speaking, an accent is an identifiable style of pronunciation, often varying regionally
or even socioeconomically. But, our aim is to neutralize & globalize your accent.

It can be contrasted with a person's dialect, which includes regional vocabulary. "Standard English has nothing to do with
pronunciation," In fact, most people who speak Standard English do so with some sort of regional pronunciation, so that you can tell
where they come from much more by their accent than by their grammar or vocabulary.

An accent is a particular way of pronouncing a language. 'Warsh' for wash in Cajun Louisiana, 'New Yawk' for New York among native
New Yorkers, 'aboot' for about in Canada. The appeal of dialects and accents comes from our appreciation of their
musical intonations, imaginative word choices, and emotive speech rhythms.

Components of Accent
There are 2 main parts to an accent:
Intonation
Pronunciation

Some important points to know about English pronunciation:

When we talk, we use our vocal organs – parts of our mouth and throat, such as our lips, teeth, tongue and larynx (voice box). This
whole area of the body is called the vocal tract.

 To make a consonant sound, we position some of our vocal organs together or close to each other. For example, our lips
come together when we make the first sound in the word pie.

 To make a vowel sound, we position the vocal organs further apart. The vocal tract is more open. For example, the word I is
pronounced as a vowel sound (the same one as the second sound in pie).

PEV 107 Page 28


What are VOWELS?

Speech sounds are broadly categorized as Vowels & Consonants. Vowels are those sounds during the utterance of which the air
escapes freely, without any obstruction. In written English there are 5 vowels i.e. a, e, i, o, u.
But, in spoken English there are 20 distinctive vowel sounds, made up of 12 pure vowels or monophthongs and 8 vowel glides or
diphthongs.

Definition: Vowel sounds are sounds that are produced by the un-interrupted flow of air.

Diphthongs : Diphthongs are 'gliding vowels', where one vowel sound glides into another one, as a result of the lips or tongue
moving. A diphthong is a change in vowel quality, whose sound changes within the same syllable.

Monophthong : A vowel with a single sound quality, such as the middle sound in "rat" or "bit". These sounds are made with one
tongue position.

AE sound

IE sound

OY sound

E-UR sound

A-UR sound

O sound

UA sound

AO Sound

What are CONSONANTS?

Consonant sounds are produced by partial or full interruption of the breath flow. Consonants give clarity and sharpness to the
words. They do the work of making our speech crisp and clear, just like the function of treble in a music system. When you
whisper it is consonants that carry forward the sound and the meaning.
‐ They also convey logic
A-Z is the Alphabet
B C D are called letters of the alphabet

The English Consonant sounds are divided into two broad buckets, which are
 Voiced Consonants-Plosive
 Unvoiced Consonants- Non Plosive

Consonant sounds can be produced either with or without a vibration of the vocal cords.

PEV 107 Page 29


Unvoiced Voiced
• /p/ path • /b/ bath
• /t/ time • /d/ dime
• /k/ came • /g/game
• /f/ fan • /v/ van
• /th/ think • /th/them
• /s/ price • z/prize
• /sh/shoe • /zh/usual
• /ch/chin • /dj/gin

Activity : Try speaking these tongue twisters and differentiate amongst the vowel sounds.

The sheep on
the ship slipped
Gnats are not
on the sheet of
now gnawing
sleet. The keen
on the nuts at
king kissed the
night.
quick queen on
her green ring.

The spoiled boy


He let himself
foiled the coy
be led to the
boy’s joy by
place for the
purloining his
plays.
toy.

PEV 107 Page 30


Syllables

Syllables are sound units that build up the structure of every word. They are a very important part of speech. They give a word its
pronunciation. Without them speech would sound dull, boring and meaningless.

Syllable Stress is very important to understand as using stress on the wrong syllable can affect our pronunciation.

Example: Develop Photographer Industry Technology

 Pronunciation is the key to Global Comprehensibility.


 A majority of the communication lapses are due to incorrect pronunciation.

1. Two-Syllable nouns and adjectives

In most two syllable nouns and adjectives, the first syllable takes on the stress.

Examples:

SAMples CARton PURple RAIny CHIna HAPpy

2. Two-Syllable verbs and prepositions

In most two syllable verbs and prepositions, the stress is on the second syllable.

Examples:

reLAX diRECT aMONG aSIDE betWEEN deCIDE

More about word stress on two-syllable words

 About 80% of two-syllable words get their stress on the first syllable.
 There are, of course, exceptions to this rule, but very few nouns and adjectives get stress on their second syllable.
 Verbs and prepositions usually get stress placed on the second syllable, but there are exceptions to this too.

Note: There are many two-syllable words in English that can be pronounced in two different ways. The stress change also changes the
part of speech of the word.

Examples:

 PREsent = a gift (noun); non past or future (adjective)


 preSENT = to give something to someone (verb)

 OBject = something you can see and touch (noun)


 obJECT = to disagree with something (verb)

3. Three-Syllable words

For three-syllable words, word stress in on 2nd syllable most of the times. However, exceptions are always there.

adEquate amAzement attEntion attrActive banAna

4. Words ending in er, ly

PEV 107 Page 31


For three-syllable words ending with the suffixes er or ly, the stress is placed on the first syllable.

ORderly SIlently LOvingly MAnager GARdener EAsier

5. Words ending in consonants and in y

If there is a word that ends in a consonant or in a y, then the first syllable usually gets the stress.

RARity OPtimal GRAdient GEnorous

6. Words with various endings

Take a good look at the list of suffixes below (suffixes are word endings).

The stress is going to be on the syllable right before the suffix. This applies to words of all syllable lengths.

Examples:

1. able: ADDable, DURable, LAUGHable


2. ial: differENTial, SOcial, fiNANcial
3. cian: muSIcian, phySIcian, cliNIcian
4. ery: BAkery, SCEnery
5. ian: coMEdian, ciVILian, techNIcian
6. ible: reSIstible, imPOSsible, TERRible
7. ic: arCHAic, plaTOnic, characteRIStic
8. ics: diaBEtics, paediAtrics, TOpics
9. ion: classifiCAtion, repoSItion, vegeTAtion
10. ia: MEdia, bacTERia, vicTORia
11. ient: inGREdient, PAtient, ANcient
12. ious: mySTERious, reLIgious, VARious
13. ish: SELfish, ENglish, PUnish
14. osis: hypNOsis, diagNOsis, osMOsis

7. Words ending in ade, ee, ese, que, ette, oon

Words that use the suffix ade, ee, ese, eer, que, ette, or oon have the primary stress actually placed on the suffix.

1. ade: lemoNADE, cruSADE, arCADE


2. ee: aGREE, jamborEE, guaranTEE
3. eer: sightSEER, puppeTEER
4. ese: SiamESE, JapanESE, chEESE
5. ette: cassETTE, CorvETTE, towelETTE
6. que: unIQUE, physIQUE
7. oon: baLOON, afterNOON, carTOON

8. Stress on the second from the end syllable

You put stress on the second syllable from the end of the word with words ending in ic, sion, and tion.

iCONic GRAPHic hyperTENsion teleVIsion nuTRItion

9. Stress on the third from end syllable

PEV 107 Page 32


You put stress on the third from end syllable with words that end in cy, ty, phy, gy and al.

deMOcracy geOGraphy ALlergy CLArity CRItical

10. Word stress for compound words

A. Compound noun

A compound noun is a noun made out of two nouns that form one word. In a compound noun, the most stress is on the stressed
syllable of the first word.

 SEAfood (sea + food)


 ICEland (ice + land)
 TOOTHpaste (tooth + paste)
 FOOTball (foot + ball)
 BAsketball (basket + ball)

B. Compound adjectives

A compound adjective is an adjective made of at least two words.

Often, hyphens are used in compound adjectives. In compound adjectives, the most stress is placed in the stressed syllable of the
second word.

 ten-MEter
 rock-SOlid
 fifteen-MInute
 old-FAshioned

C. Compound verbs

A compound verb is when a subject has two or more verbs. The stress is on the second or on the last part.

 Matilda loves bread but deTESTS butter.


 Sarah baked cookies and ATE them up.
 Dogs love to eat bones and love to DRINK water.

D. Noun + compound nouns

Noun + compound Nouns are two word compound nouns. In noun + compound noun, the stress is on the first word.

 AIRplane mechanic
 PROject manager
 BOARD member

11. Reflexive pronouns

Reflexive pronouns show that the action affects the person who performs the action. For example: I hit mySELF.

The second syllable usually takes the stress.


PEV 107 Page 33
 mySELF
 themSELVES
 ourSELVES

What is Sentence Stress?


Words in a sentence are not all given the same salience in oral English. Some words are picked out and are stressed in contrast to
others. The one that is the most stressed is said to receive the sentence stress. This usually implies differences in meaning.

I did not say you stole my red hat.

At the moment, nothing is particularly stressed. The meaning seems fairly obvious. But what if some stress is placed on the first word -
I:

I did not say you stole my red hat.

Then the meaning contains the idea that someone else said it, not me. Stress the second and third word and you get another shade of
meaning:

I did not say you stole my red hat. (Strong anger and denial of the fact.)
I did not say you stole my red hat. (Strong anger and denial of the fact.)
I did not say you stole my red hat. (But I implied it that you did. Did you?)
I did not say you stole my red hat (I wasn't accusing you. I know it was someone else)
I did not say you stole my red hat. (I said you did something else with it, or maybe borrowed it.)
I did not say you stole my red hat. (I meant that you stole someone else's red hat)
I did not say you stole my red hat. (I said that you stole my blue hat.)
I did not say that you stole my red hat. (I said that you stole my red bat. You misunderstood my pronunciation)

Analyzing this way, you can see how important stress is in English. Now, you need to understand which words we generally stress and
which we do not stress.

Most sentences have two basic types of word:

 Content words
Content words are the key words of a sentence. They are the important words that carry the meaning or sense—the real
content.
 Structure words
Structure words are not very important words. They are small, simple words that make the sentence correct grammatically.
They give the sentence its correct form—its structure.

If you remove the structure words from a sentence, you will probably still understand the sentence. If you remove the content words
from a sentence, you will not understand the sentence. The sentence has no sense or meaning.

Imagine that you receive this telegram message:

This sentence is not complete. It is not a "grammatically correct" sentence. But you probably understand it. These 4 words
communicate very well. Somebody wants you to sell their car for them because they have gone to France. We can add a few words:

But the information is basically the same:


PEV 107 Page 34
VOICE MODULATION (INTONATION)

Intonation

Intonation is the music of the language. In English, we use tone to signal emotion, questioning, and parts of the sentence among many
other things. It's important to recognize the meaning behind the tones used in everyday speech, and to be able to use them so that
there are no misunderstandings between the speaker and the listener. It is generally true that mistakes in pronunciation of sounds can
be overlooked, but mistakes in intonation make a lasting impression.

In other words we can say that, the intonation of a language refers to the patterns of pitch variation, or the tones,
its uses in its utterance.

Falling Intonation (➘)


(The pitch of the voice falls at the end of the sentence.)
Falling intonation is the most common intonation pattern in English.
It is commonly found in statements, commands, wh-questions (information questions),
confirmatory question tags and exclamations.

 Statements
o Nice to meet you.
o I’ll be back in a minute.
o She doesn’t live here anymore.
o Dad wants to change his car.
o Here is the weather forecast.
o Cloudy weather is expected at the end of the week.

 Commands
o Write your name here.
o Show me what you’ve written.
o Leave it on the desk.
o Take that picture down.
o Throw that out.
o Put your books on the table.
o Take your hands out of your pockets.

 Wh- questions (requesting information.)


(questions beginning with 'who', 'what', 'why', 'where', 'when', 'which', and 'how')
o What country do you come from?
o Where do you work?
o Which of them do you prefer?
o When does the shop open?
o How many books have you bought?
o Which coat is yours?
o Whose bag is this?

 Questions Tags that are statements requesting confirmation rather than questions.
Not all tag questions are really questions.
Some of them merely ask for confirmation or invite agreement, in which case we use a falling tone at the end.

o He thinks he’s so clever, doesn’t he?

PEV 107 Page 35


o She's such a nuisance, isn't she?
o I failed the test because I didn't revise, did I?
o It doesn't seem to bother him much, does it?

 Exclamations
o How nice of you!
o That's just what I need!
o You don't say!
o What a beautiful voice!
o That's a surprise!

Rising Intonation (➚)


(The pitch of the voice rises at the end of a sentence.)
Rising intonation invites the speaker to continue talking.
It is normally used with yes/no questions, and question tags that are real questions.

 Yes/no Questions
(Questions that can be answered by 'yes' or 'no'.)
o Do you like your new ➚teacher?
o Have you finished ➚already?
o May I borrow your ➚dictionary?
o Do you have any ➚magazines?

 Questions tags that show uncertainty and require an answer (real questions).
o We've met already, ➚haven't we?
o You like fish, ➚don't you?
o You're a new student ➚aren't you?
o The view is beautiful, ➚isn't it?

We sometimes use a combination of rising and falling intonation in the same sentence.
The combination is called Rise-Fall or Fall-Rise intonation.

Rise-Fall Intonation (➚➘)


(The intonation rises and then falls.)
We use rise-fall intonation for choices, lists, unfinished thoughts and conditional sentences.

 Choices (alternative questions.)


o Are you having ➚soup or ➘salad?
o Is John leaving on ➚Thursday or ➘Friday?
o Does he speak ➚German or ➘French?
o Is your name ➚Ava or ➘Eva?

 Lists (rising, rising, rising, falling)


Intonation falls on the last item to show that the list is finished.
o We've got ➚apples, pears, bananas and ➘oranges
o The sweater comes in ➚blue, white pink and ➘black
o I like ➚football, tennis, basketball and ➘volleyball.

 Unfinished thoughts (partial statements)


In the responses to the following questions, the rise-fall intonation indicates reservation.
The speaker hesitates to fully express his/her thoughts.
o Do you like my new handbag? Well the ➚leather is ➘nice... ( but I don't like it.)
o What was the meal like? Hmm, the ➚fish was ➘good... (but the rest wasn't great).

 Conditional sentences
(The tone rises in the first clause and falls gradually in the second clause.)
o If he ➚calls, ask him to leave a ➘message.
o Unless he ➚insists, I'm not going to ➘go.

PEV 107 Page 36


Tutorial
Exercises:

1. Insert the correct word(s) in the sentences below.


1. If we go __________ the beach, would you like to come __________? (to/ too/ two)
2. I am going __________ you like it or not. (whether/ weather)
3. I shall put __________ parcels over __________. (there / their/ they’re)
4. I like to come __________ because I always __________ the latest gossip. (hear/here)
5. There is __________ paper left in the printer, or did you __________ that already?(no/ know)
6. __________ you like to come to the party with me? (would/ wood)
7. You need to go __________ the door at the end of the corridor to get to the exit.(through/ threw)
8. I must __________ a letter to the bank. (write / right)
9. I need to see if he has cashed the __________. (check/ cheque)

10. He __________ his motorbike along the __________. (road/ rode/ rowed)

2.Cecily Berry Vowels Drill: Speak these words loudly:

OOT OHT AWT AHT AYT EET EHT

OOD OHD AWD AHD AYD EED EHD

OON OHN AWN AHN AYN EEN EHN

OOB OHB AWB AHB AYB EEB EHB


OOP OHP AWP AHP AYP EEP EHP

OOM OHM AWM AHM AYM EEM EHM

OOG OHG AWG AHG AYG EEG EHG

OOF OHF AWF AHF AYF EEF EHF

OOV OHV AWV AHV AYV EEV EHV

OOS OHS AWS AHS AYS EES EHS

OOZ OHZ AWZ AHZ AYZ EEZ EHZ

OOTH OHTH AWTH AHTH AYTH EETH EHTH

CECILY BERRY DRILL-EXERCISE FOR MOUTH MUSCLES


OOKHT OHKHT AWKHT AHKHT AYKHT EEKHT

OOGD OHGD AWGD AHGD AYGD EEGD

OOPT OHPT AWPT AHPT AYPT EEPT

OOBD OHBD AWBD AHBD AYBD EEBD

PEV 107 Page 37


OOMD OHMD AWMD AHMD AYMD EEMD

OOND OHND AWND AHND AYND EEND

OOLT OHLT AWLT AHLT AYLT EELT

OOLD OHLD AWLD AHLD AYLD EELD

OOLZ OHLZ AWLZ AHLZ AYLZ EELZ

OOTHT OHTHT AWTHT AHTHT AYTHT EETHT

OOTHD OHTHD AWTHD AHTHD AYTHD EETHD

OOST OHST AWST AHST AYST EEST

OOSTS OHSTS AWSTS AHSTS AYSTS EESTS

OOZD OHZD AWZD AHZD AYZD EEZD

OOFT OHFT AWFT AHFT AYFT EEFT

OOFTS OHFTS AWFTS AHFTS AYFTS EEFTS

OOVD OHVD AWVD AHVD AYVD EEVD

OOKST OHKST AWKST AHKST AYKST EEKST

OOTTH OHTTH AWTTH AHTTH AYTTH EETTH

3. Fill in the blanks using ‘aw’ sound.


• Niagara _ ___ls is the highest falls in the world.
• The children are playing with the basket __ ____.
• He got a ______ _____ship from his school.
• ____ _____ walk to your left on the road.
• Roses have _ ______.

4.Unscramble the jumbled words


Liob -
Jyeon -
Snioe -
Oyj -
Inoc -
Sybo -
Ayorl -

5.Choose the Correct Response.


• I went to the barber to get a (hair/ hare)cut.
• Sheena is a (fare/ fair) girl.
• On Ridhima’s birthday her father gifted her a teddy(bare/ bear).
• Please don’t (stare/ stair) at the stranger.
• I bought a (pear/ pair) of scissors for the craft classes.
• I will (ware/ wear) a red dress for the party.

6. Use ‘wear’, ‘where’ or ‘we’re’ correctly:

1. Make sure you __________________ a life jacket on the boat.


2. Please put the art supplies back ____________________ they belong.
PEV 107 Page 38
3. ____________________ going on a road trip next Friday.
4. Mom told me to __________________ a coat, but I didn’t listen.
5. I don’t know ____________________ I put those moving boxes.
6. This weekend, __________________ going to the park to play baseball.
7. _____________________ will you hide the gifts?
8. My sister wants to _____________ the sweater she got for her birthday.
9. John invited us to breakfast, but ______________ not going to make it on time.
10.I will ________________ my red hat to school tomorrow.

7. In English, many written words contain consonant letters that are not pronounced. These letters are
referred to as ‘silent’ letters. Match each word with the appropriate meaning below.

Rhyme often listen island calm column talk foreign hour


exhausted bark

1.sixty minutes __________________________


2land surrounded by water __________________________
3 peaceful and quiet __________________________
4 words containing the same sounds __________________________
5 to hear and give attention when someone speaks __________________________
6 to speak __________________________
7 to be very tired and without energy __________________________
8 from another country or another place __________________________
9 a strong, tall piece of stone or wood __________________________
10 used to support a building __________________________

8. Read the sentence. Write the correct word on the line.

1. The __________________ going around the mountain is very scary. (rode, road)
2. _____________________ not going to believe what I saw today. (your, you’re)
3. I don’t know if I would like ___________________ soup or not. (beet, beat)
4. Stanley ran around two ____________________ before they threw him out. (bases, basis)
5. The huge _________________________ sat dangerously close to the edge. (bolder, boulder)
6. The ______________________ in our new house will be ten feet tall. (ceiling, sealing)
7. The ________________________ ripped on the curtain when the cat climbed it. (seem, seam)
8. My neighbor delivers the ______________________ on your ___________________. (mail, male) (rode, road)
9. He said the dog was __________________ but __________ not. (theirs, there’s) (its, it’s)
10. My mom bought us four different kinds of __________________________ to eat. (cereal, serial)
11. An independent _________________________ is a sentence. (clause, claws)
12. We rented a hotel ___________________ when we went on vacation. (suite, sweet)

9. CECILY BERRY DRILL-CONSONANT SOUND PRACTICE

BOO BOH BAW BAH BAY BEE BIE

COO COH CAW CAH CAY CEE CIE

DOO DOH DAW DAH DAY DEE DIE

FOO FOH FAW FAH FAY FEE FIE

GOO GOH GAW GAH GAY GEE GIE

PEV 107 Page 39


HOO HOH HAW HAH HAY HEE HIE

JOO JOH JAW JAH JAY JEE JIE

KOO KOH KAW KAH KAY KEE KIE

LOO LOH LAW LAH LAY LEE LIE

MOO MOH MAW MAH MAY MEE MIE

NOO NOH NAW NAH NAY NEE NIE

POO POH PAW PAH PAY PEE PIE

ROO ROH RAW RAH RAY REE RIE

SOO SOH SAW SAH SAY SEE SIE

TOO TOH TAW TAH TAY TEE TIE

VOO VOH VAW VAH VAY VEE VIE

WOO WOH WAH WAH WAY WEE WIE

YOO YOH YAW YAH YAY YEE YIE

ZOO ZOH ZAW ZAH ZAY ZEE ZIE

10. Write sentences for 2 pairs of words, demonstrating the different word class:

1. Conduct
Noun ……………………………………………………………………………………………………………………………………………
Verb ……………………………………………………………………………………………………………………………………………
2. Digest
Noun ……………………………………………………………………………………………………………………………………………
Verb ……………………………………………………………………………………………………………………………………………
3. Escort
Noun ……………………………………………………………………………………………………………………………………………
Verb ……………………………………………………………………………………………………………………………………………
4. Insult
Noun ……………………………………………………………………………………………………………………………………………
Verb ……………………………………………………………………………………………………………………………………………
5. Produce
Noun ……………………………………………………………………………………………………………………………………………
Verb ……………………………………………………………………………………………………………………………………………
6. Record

PEV 107 Page 40


Noun ……………………………………………………………………………………………………………………………………………
Verb ……………………………………………………………………………………………………………………………………………
7. Access
Noun ……………………………………………………………………………………………………………………………………………
Verb ……………………………………………………………………………………………………………………………………………
8. Address
Noun ……………………………………………………………………………………………………………………………………………
Verb ……………………………………………………………………………………………………………………………………………
9. Auction
Noun ……………………………………………………………………………………………………………………………………………
Verb ……………………………………………………………………………………………………………………………………………
10. Balance
Noun ……………………………………………………………………………………………………………………………………………
Verb ……………………………………………………………………………………………………………………………………………
11. Bargain
Noun ……………………………………………………………………………………………………………………………………………
Verb ……………………………………………………………………………………………………………………………………………
12. Blame
Noun ……………………………………………………………………………………………………………………………………………
Verb ……………………………………………………………………………………………………………………………………………
13. Blast
Noun ……………………………………………………………………………………………………………………………………………
Verb ……………………………………………………………………………………………………………………………………………
14. Catch
Noun ……………………………………………………………………………………………………………………………………………
Verb ……………………………………………………………………………………………………………………………………………
15. Cause
Noun ……………………………………………………………………………………………………………………………………………
Verb ……………………………………………………………………………………………………………………………………………
16. Convict
Noun ……………………………………………………………………………………………………………………………………………
Verb ……………………………………………………………………………………………………………………………………………
17. Complex
Noun ……………………………………………………………………………………………………………………………………………
Verb ……………………………………………………………………………………………………………………………………………
18. Desert
Noun ……………………………………………………………………………………………………………………………………………
Verb ……………………………………………………………………………………………………………………………………………
19. Segment
Noun ……………………………………………………………………………………………………………………………………………

PEV 107 Page 41


Verb ……………………………………………………………………………………………………………………………………………
20. Content
Noun ……………………………………………………………………………………………………………………………………………
Verb ……………………………………………………………………………………………………………………………………………

11. Use underlining to show the correct stress on these compound words which have been given in a sentence to
make the word class obvious:
1. I heard a blackbird singing.
2. Put the seedlings in the greenhouse until they are taller.
3. He is a bad-tempered old man.
4. My grandparents are a little old-fashioned. (2 words)
5. I don’t understand what you mean.
6. The water will overflow and come out through this overflow pipe. (2 words)
7. I need to go to the supermarket before I leave for the airport. (2 words)
8. He is waiting at the bus-stop on the main highway. (2 words)
9. He came straight out of the swimming-pool and into the living-room to answer the telephone. (3 words)
10. This raincoat isn’t waterproof. (2 words)
11. Traffic-lights are confusing because I’m colour-blind. (2 words)
12. What’s he like? Well he’s easy-going, and good-looking, very self-confident and always well-dressed. (4 words)

12. Read the sentence. Write the correct word on the line:

1. ____________________ going to be the first to recite ___________________ poem? (Whose, Who’s) (their, there,
they’re)
2. The window ____________________ needs to be painted white. (pain, pane)
3. The yellow part of an egg is called the _______________________, (yoke, yolk)
4. Cindy makes ____________________ salad every time I eat at her house. (pare, pair, pear)
5. Grandma taught me how to _________________ bread __________________. (need, knead) (doe, dough)
6. I love going to the county _______________________ each year. (fare, fair)
7. My dad always says, “I love you _____________” to my mom. (deer, dear)
8. You need to make up your ___________________ before _________ too late. (mined, mind) (its, it’s)
9. They are going to _____________________ Tuesday with __________________ friends. (meat, meet) (there, their,
they’re)
10. The dress looks much better with a belt around the __________________. (waist, waste)
11. Martha Steward showed us how she ____________________________ her turkey. (trust, trussed)
12. Melvin ____________________ make that mistake again. (want, won’t)

PEV 107 Page 42


13. For each question, the correct choice is the one in which the stressed syllable is capitalized, as in vocabulary:

1. Can you pass me a plastic knife?


a) PLAS-tic
b) plas-TIC

2. I want to be a photographer.

a) PHO-to-graph-er
b) pho-TO-graph-er

3. Which photograph do you like best?


a) PHO-to-graph
b) pho-TO-graph

4. He was born in China.


a) CHI-na
b) Chi-NA

5. Whose computer is this?


a) com-PU-ter
b) com-pu-TER

6. I can't decide which book to borrow.


a) DE-cide
b) de-CIDE

7. Couldn't you understand what she was saying?


a) un-DER-stand
b) un-der-STAND

8. Voting in elections is your most important duty.


a) im-POR-tant
b) im-por-TANT

9. We had a really interesting conversation.


a) con-VER-sa-tion
b) con-ver-SA-tion

10. How do you pronounce this word?


a) PRO-nounce
b) pro-NOUNCE

14. Where is the stress in each of these words? Decide which syllable:
1. reception
st nd
a) 1 b) 2 c) 3rd

2. comparison
st nd rd
a) 1 b) 2 c) 3 d) 4th

3. potato
st nd rd
a) 1 b) 2 c) 3

4. bedroom

PEV 107 Page 43


st
a) 1 b) 2nd

5. fourteen

a) 1st b) 2nd

6. forty

a) 1st b) 2nd

7. delicious
st nd rd
a) 1 b) 2 c) 3
8. playful
st nd
a) 1 b) 2

15. Mark the stress on the following words as shown in the sample:

oO Oo Ooo oOo
today table confident tomorrow

Frightened Embarrassed Relaxed advert

Crowded Enjoy Glasses video

Journalist Describe Envelope photograph

Assistant Invited Appeared rely

Internet Appearance Happen invented

16. How many syllables are there in each word? Choose the correct answer.

1. monkey

a) 1 b) 2 c) 3 d) 4

2. relocation

a) 1 b) 2 c) 3 d) 4

3. magnet

a) 1 b) 2 c) 3 d) 4

PEV 107 Page 44


4. slobs

a) 1 b) 2 c) 3 d) 4

5. characterise

a) 1 b) 2 c) 3 d) 4

6. travelling

a) 1 b) 2 c) 3 d) 4

7. rocket

a) 1 b) 2 c) 3 d) 4

8. trying

a) 1 b) 2 c) 3 d) 4

9. garden

a) 1 b) 2 c) 3 d) 4

10. caterpillar

a) 1 b) 2 c) 3 d) 4

17. Match the following conversations with the correct sentence stress:

A. Where did you get these flowers from? The cemetery? 1. I asked you to buy me a bunch of white roses.

B. Yes....

A. You weren’t supposed to steal them! ________________

A. Here are the flowers Bob asked me to get. 2. I asked you to buy me a bunch of white roses.

B. Eh? Bob didn’t say anything, _______________

A. Why do these roses have your mother’s name on them? 3. I asked you to buy me a bunch of white roses.

B. I got them for my mother, just like you asked.

A. In what world would I ask you to buy your mother flowers on our

anniversary? _______________

PEV 107 Page 45


A. John, why are there yellow roses on the table? _______________ 4. I asked you to buy me a bunch of white roses.

A. Oh, lilies, they’re beautiful, but _____________ 5. I asked you to buy me a bunch of white roses.

A. I was kind of busy today, so my secretary did me the favour of ordering 6. I asked you to buy me a bunch of white roses.

you the flowers you wanted.

B. What?! ____________________ Why do I want roses from your

secretary?

A. Listen, I know a single rose is supposed to be very romantic, but 7. I asked you to buy me a bunch of white roses.

________________

A. Susie, you’re always bossing me around. Ordering me to buy you 8. I asked you to buy me a bunch of white roses.

flowers is the last straw.

B. Fred, honestly, I don’t understand what you’re talking about.

__________________, not order!

18. Use the clues provided to find 8 words in the puzzle that contain the diphthongs ou or ow:

19. Look at each of these words. Decide on which syllable the stress falls?

1. personal

st nd rd
a. 1 b. 2 c. 3

2. personnel

PEV 107 Page 46


st nd rd
a. 1 b. 2 c. 3

3. adjective

st nd rd
a. 1 b. 2 c. 3

4. enhance

st nd
a. 1 b. 2

5. Canadian

st nd rd th
a. 1 b. 2 c. 3 d. 4

6. Japanese

st nd rd
a. 1 b. 2 c. 3

7. psychology

st nd rd th
a. 1 b. 2 c. 3 d. 4

8. politician

st nd rd th
a. 1 b. 2 c. 3 d. 4

20. SVAR LINES


(Read the following lines using proper stress and intonation)
1. He did not settle for less than what he wanted.
2. You can change anything about the house except its location.
3. Technological advancement has shortened the attention span.
4. I find philosophical discussions very interesting.
5. I want clear and definitive answers to all the questions in the test.
6. The terminator is generally referred to as a weapon of mass destruction.
7. Fortunately no one was badly injured.
8. With the advent of technology, things have changed dramatically.
9. Is the current generation determinant enough to fight the corruption?
10. The garden behind our house was my favorite hangout place.
11. One should accept the truth.
12. Frustration increases exponentially and decreases gradually.
13. I’m responsible for what I say and not for what you interpret.

PEV 107 Page 47


14. Money makes some people greedy.
15. The presentation on the nation’s financial situation was enlightening.
16. The elephant is the largest terrestrial mammal.
17. Name any five people who have taught you something worthwhile.
18. Its perfectly reasonable for the kids to be afraid of darkness.
19. Human brain is considered to be a machine.
20. The over speeding automobile crashed into the wall.
21. First impressions are mostly prejudiced.
22. Water can be converted into a boundless source of energy.
23. Why do engineers often opt for management program?
24. People who don’t switch off their phones during meetings are unprofessional.
25. He did not produce enough victories to save his captaincy.

The English Learner’s Guide to UK Slang: 18 Must-know British Words for Casual Use

1. Chuffed-When someone is chuffed, they are very pleased or happy about something.
I’m absolutely chuffed with my birthday present. Thanks!

2. Knackered-means that someone is extremely tired.


I’ve been up half of the night with the baby. I’m totally knackered.

3. Bants-is an abbreviation of “Banter.” means to joke or to exchange witty remarks with others.
I’m going to Nando’s for some bants with the lads.

4.Cheeky-it means that they are being a little rude or disrespectful, but usually in a way that is funny and endearing (cute).
That is a cheeky smile…are you up to something?
Did you just take the last biscuit? That was a bit cheeky!
It can also be used if you are eating, drinking or doing something that you maybe should not or that is not good for
you.
I’m just going to have a cheeky burger on the way home.
Are you coming to the pub tonight? On a Tuesday?! Well OK, just a few cheeky drinks.

5. Fag-In British slang, however, it just means a cigarette.


I’m going outside for a fag.

6.Cuppa-comes from the phrase “cup of.” You only need to make it clear if it is a “cuppa” coffee or a “cuppa” something
other than tea.
Would you like a cuppa? I’d love one. I’ll get the kettle on.

7.Bum-You may already know that this word is the informal word for “bottom.” It also has another meaning. It is used
when somebody uses or gets something from someone else without paying.
Can I bum a fag?
How did you get here? I bummed a lift with Tony.

8.Mate-While in standard English a mate is a life partner, it is commonly used in Britain to mean a friend. It is also often
used to address strangers in informal situations, such as in bars or on public transport. It is particularly used between men.

What are you doing this weekend? Hanging out with some of my mates.
Excuse me, mate, is anyone sitting here?

PEV 107 Page 48


Hey pal, could I get a whiskey and a beer please?

9.Gutted-It also has the meaning of being bitterly disappointed about something.
I was gutted when I failed the exam.

10.Narky-another word for moody or bad-tempered.


She won’t speak to me. She’s been narky with me all day.

11.Fluke-A “fluke” is something caused by chance or luck. Something can also be described as being “flukey,” meaning
that it is particularly lucky or coincidental.
I hit the bulls-eye (the red target on a darts board)!, That was a total fluke! You wouldn’t be able to do it again.
I won 10 pounds on the lotto again!, That is so flukey!

12.Gagging-The original meaning of this word is choking or retching (making movements and sounds like vomiting).
However, it also means to desperately need or want something.
I’m gagging for a cuppa. I haven’t had one all day!

13.Cracking-When something is cracking it is particularly good or excellent. People can also be described as “cracking.” For
example: “He’s a cracking lad.” It can also mean to get started on something (and this meaning is also used in American
speech).

That was a cracking dinner. Compliments to the chef.


I have lots of Christmas presents to wrap. I’d better get cracking!

14.Bloody-In standard English, “bloody” usually refers to something covered in blood. In British slang, though, this is an
intensifier (something that puts stress or importance on another word) and a mild expletive (swear word).

Bloody British English is bloody confusing! Bloody hell! Why do they have so much bloody slang's?

15.Kerfuffle-A skirmish or fight caused by differing views.


He and I got into a kerfuffle over politics.

16.Marbles-Wit, intelligence, or good sense.


Have you lost your marbles?

17.Numpty -An incompetent or unwise person.


You and your Numpty friend should apologize.

18.Skint-Without money, broke, bankrupt.


Sorry I can’t join you this time. I’m skint.

PEV 107 Page 49


Direct Indirect Speech
What is Direct & Indirect Speech?

Direct Speech: the message of the speaker is conveyed or reported in his own actual words without any change. It is also called as
REPORTED SPEECH.
Indirect Speech: the message of the speaker is conveyed or reported in our own words.
Example on Process of Conversion from Direct to Indirect Speech
a) Direct: Radha said, “I am very busy now.”
b) Indirect: Radha said that she was very busy then.
The verb that introduces the Reported Speech is called the Reporting Verb.

In the above sentence, “Said” is the Reporting Verb and “I am very busy now” is the Reported Speech.

 All inverted commas or quotation marks are omitted and the sentence ends with a full stop.
 Conjunction ‘that’ is added before the indirect statement.
 The Pronoun is changed in Person.
 Present Tense is changed to Past.
 The adverb ‘now’ is changed to ‘then’.

Tips on Direct and Indirect Speech

Tip 1: Conversion Rules as per the Reporting Verb


When the reporting or principal verb is in the Past Tense, all Present tenses of the direct are changed into the corresponding Past
Tenses.
a) Direct: He said, “I am unwell.”
b) Indirect: He said (that) he was unwell.
If the reporting verb is in the Present or Future Tense, the tenses of the Direct Speech do not change.
a) Direct: He says/will say, “I am unwell.”
b) Indirect: He says/will say he is unwell.
The Tense in Indirect Speech is NOT CHANGED if the words within the quotation marks talk of a universal truth or habitual action.
a) Direct: They said, “We cannot live without water.”
b) Indirect: They said that we cannot live without water.

Tip 2: Conversion Rules of Present Tense in Direct Speech

PEV 107 Page 50


Simple Present Changes to Simple Past
a) Direct: "I am happy", she said.
b) Indirect: She said that she was happy.
Present Continuous Changes to Past Continuous
a) Direct: "I am reading a book", he explained.
b) Indirect: He explained that he was reading a book.
Present Perfect Changes to Past Perfect
a) Direct: She said, "He has finished his food“.
b) Indirect: She said that he had finished his food.
Present Perfect Continuous Changes to Past Perfect Continuous
a) Direct: "I have been to Gujarat", he told me.
b) Indirect: He told me that he had been to Gujarat.

Tip 3: Conversion Rules of Past & Future Tense

Simple Past Changes to Past Perfect


a) Direct: He said, “Ira arrived on Monday."
b) Indirect: He said that Ira had arrived on Monday.
Past Continuous Changes to Past Perfect Continuous
a) Direct: "We were living in Goa", they told me.
b) Indirect: They told me that they had been living in Goa.
Past Perfect Continuous remains Past Perfect Continuous
a) Direct: Jill said “I had been cooking”.
b) Indirect: Jill said that she had been cooking.
Future Simple Changes to Present Conditional
a) Direct: He said, "I will be in Kolkata tomorrow."
b) Indirect: He said that he would be in Kolkata the next day.
Future Continuous Changes to Conditional Continuous
a) Direct: She said, "I'll be using the car next Friday.”
b) Indirect: She said that she would be using the car next Friday.
Future Perfect Continuous remains Future Perfect Continuous (Will changes to would)
a) Direct: She said, "We will have been watching a movie.”
b) Indirect: She said that they would have been watching a movie.

Tip 4: Changes in Modals

CAN changes into COULD


a) Direct: He said, "I can swim."
b) Indirect: He said that he could swim.

PEV 107 Page 51


MAY changes into MIGHT
a) Direct: He said, "I may buy a house.”
b) Indirect: He said that he might buy a house.
MUST changes into HAD TO/WOULD HAVE TO
a) Direct: He said, "I must work hard.”
b) Indirect: He said that he had to work hard.
Modals that DO NOT Change: Would, Could, Might, Should, Ought to.
a) Direct: He said, "I should face the challenge.”
b) Indirect: He said that he should face the challenge.

Tip 5: Conversion of Interrogative


Reporting Verb like ‘said/ said to’ changes to asked, enquired or demanded
a) Direct: He said to me, “What are you doing?”
b) Indirect: He asked me what I was doing.
If sentence begins with auxiliary verb, the joining clause should be if or whether.
a) Direct: He said, “Will you come for the meeting?”
b) Indirect: He asked them whether they would come for the meeting.
If sentence begins with ‘wh’ questions then no conjunction is used as the "question-word" itself act as joining clause.
a) Direct: “Where do you live?” asked the girl.
b) Indirect: The girl enquired where I lived.
Interrogative sentence in indirect speech ends with a FULL STOP and not a QUESTION MARK.

Tip 6: Command, Request, Exclamation, Wish

Commands and Requests


Indirect Speech is introduced by some verbs like ordered, requested, advised and suggested. Forbid(s)/ forbade is used for the
negative sentences. The imperative mood is changed into the Infinitive.
a) Direct: Rafique said to Ahmed, “Go away.”
b) Indirect: Rafique ordered Ahmed to go away.
c) Direct: He said to her, “Please wait.”
d) Indirect: He requested her to wait.
Exclamations and Wishes
Indirect Speech is introduced by some words like grief, sorrow, happiness, applaud. Exclamatory sentence changes into assertive
sentence and Interjections are removed.
a) Direct: He said, “Alas! I am undone.”
b) Indirect: He exclaimed sadly that he was broke.

Tip 7: Change of Pronouns

The first person of the reported speech changes according to the subject of reporting speech.

PEV 107 Page 52


a) Direct: She said, “I am in ninth class.”
b) Indirect: She says that she was in ninth class.
The second person of reported speech changes according to the object of reporting speech.
a) Direct: He says to them, "You have completed your job.”
b) Indirect: He tells them that they have completed their job.
The third person of the reported speech doesn't change.
a) Direct: He says, "She is in tenth class.”
b) Indirect: He says that she is in tenth class.

Tip 8: Change of Place and Time

Words expressing nearness in time or place in Direct Speech are generally changed into words expressing distance in Indirect
Speech.
Now -- then
Here -- there
Ago -- before
Thus -- so
Today -- that day
Tomorrow -- the next day
This -- that
Yesterday -- the day before
These -- those
Hither-- thither
Come -- go
Hence -- thence
Next week/month -- following week/month
a) Direct: She said, “My father came yesterday.”
b) Indirect: She said that her father had come the day before.
c) Direct: She says/will say, “My father came yesterday.”
Indirect: She says/will say that her father had come yesterday. (Here the reporting verb ‘says’ is in the present tense OR ‘will say’ is
in future tense; hence the time expression ‘yesterday’ won’t change.)

Tip 9: Punctuation

The words that are actually spoken should be enclosed in quotes and begin with a capital letter
Example: He said, “You are right.”
Comma, full stop, question mark, or exclamation mark must be present at the end of reported sentences and are placed inside the
closing inverted comma or commas.
Example: He asked, “Can I come with you?”

PEV 107 Page 53


If direct speech comes after the information about who is speaking, comma is used to introduce the piece of speech, placed before
the first inverted comma.
Example: She shouted, “Stop talking!”
Example: “Thinking back,” she said, “he didn't expect to win.” (Comma is used to separate the two reported speech and no capital
letter to begin the second sentence).

Tip 10: Conversion of Indirect to Direct Speech

1. Use the reporting verb, "say" or "said to" in its correct tense.
2. Remove the conjunctions "that, to, if or whether etc" wherever necessary.
3. Insert quotation marks, question mark, exclamation and full stop, as per the mood of the sentence.
4. Write the first word of the statement with capital letter.
5. Change the past tense into present tense wherever the reporting verb is in the past tense.
Example
a) Indirect: He asked whether he is coming.
b) Direct: He said to him, “Are you coming?”

Tutorial
Exercises: Beginner

Spot the Errors: Each of the following sentences has a mistake. See if you can spot that mistake.

#1: Direct: The boy said, “I’m happy with my results.”

Indirect: The boy said that he is happy with his results. (Incorrect)
Indirect: The boy said that he was happy with his results. (Correct)

#2: Direct: She said, “I have baked a cake.”

Indirect: She said (that) she baked a cake. (Incorrect)


Indirect: She said (that) she had baked a cake. (Correct)

#3: Direct: He said, “All people have equal rights.”

Indirect: He said that all people had equal rights. (Incorrect)


Indirect: He said that all people have equal rights. (Correct)

#4: Direct: Roshni said, “I may meet him here”.

Indirect: Roshni said that she may meet him here. (Incorrect)
Indirect: Roshni said that she might meet him there. (Correct)

#5: Direct: She says, “I will go to school tomorrow.”

Indirect: She says that she would go to school the day after. (Incorrect)

PEV 107 Page 54


Indirect: She says that she will go to school tomorrow. (Correct)

#6: Direct: He said, “She is coming this week to discuss this.”

Indirect: He said that she was coming this week to discuss this. (Incorrect)
Indirect: He said that she was coming that week to discuss it. (Correct)

#7: Direct: He said to them, “Will you come for dinner?”

Indirect: He said to them will they come for dinner? (Incorrect)


Indirect: He asked them whether they would come for dinner.(Correct)

#8: Direct: The teacher said, “Be quiet and listen to my words.”

Indirect: The teacher said them to be quiet and listen to my words. (Incorrect)
Indirect: The teacher urged /ordered them to be quiet and listen to his words. (Correct)

#9: Direct: The old man said, “Ah! I am ruined.”

Indirect: The old man said that Ah he was ruined! (Incorrect)


Indirect: The old man exclaimed with sorrow that he was ruined.

#10: Indirect: The policeman enquired where we were going.

Direct: The policeman enquired where are you going. (Incorrect)


Direct: The policeman said, “Where are you going?” (Correct)

Change the following into indirect speech.

1. The girl said, ‘It gives me great pleasure to be here this evening.’
2. The man said, ‘I must go as soon as possible.’
3. She said, ‘I don’t want to see any of you; go away.’
4. The teacher says, ‘If you work hard, you will pass.’
5. He said, ‘I have won.’
6. He said, ‘Let’s wait for her return.’
7. Alice said, ‘How clever I am!’
8. ‘Which way did she go?’ asked the young man.
9. He said to me, ‘Where are you going?’
10. ‘Please wait here till I return,’ she told him.

Exercises: Intermediate

Complete the sentences in reported speech. Note whether the sentence is a request, a statement or a
question.

1. He said, "I like this song."


2. "Where is your sister?" she asked me.
3. "I don't speak Italian," she said.
4. "Say hello to Jim," they said.
5. "The film began at seven o'clock," he said.
6. "Don't play on the grass, boys," she said.

PEV 107 Page 55


7. "Where have you spent your money?" she asked him.
8. "I never make mistakes," he said.
9. "Does she know Robert?" he wanted to know.
10. "Don't try this at home," the stuntman told the audience.
11. Mike: "Don't shout at Peter!"
12. Yuvraj: "Don't talk to your neighbour!"
13. Jane: "Don't watch the new film!"
14. Teacher: "Don't forget your homework!"
15. Marcel: "Don't sing that song!"

Instructions: Choose the correct answer.

Q1 - I have no idea ____.


 where she's
 where she is
 where is she

Q2 - Could you tell me ____ gone?


 where she's
 where is she
 where has she

Q3 - I asked them where ____ going.


 were they
 they were

Q4 - Can you tell me how much ____ cost?


 will it
 it will

Q5 - Do you have any idea how long ____ me to do it?


 did it take
 it took
 took it

Q6 - Has she told you whether ____ help?


 will she
 she will

Q7 - Can you explain why ____ so expensive?


 is it
 it's

Q8 - She asked me where ____ from.


 do I come
 come I
 I come

Q9 - I was wondering why ____ take the train; it'll be quicker.


 don't we

PEV 107 Page 56


 we don't

Q10 - Can you remember _____?


 what did she say
 what she said

Exercises: Advanced

Read the dialogue and report it.

Mike: "What are you doing here, Liz? I haven't seen you since June."
Liz: "I've just come back from my holiday in Ireland."
Mike: "Did you enjoy it?"
Liz: "I love Ireland. And the Irish people were so friendly."
Mike: "Did you go to the Wicklow Mountains?"
Liz: "It was my first trip. I can show you some pictures. Are you doing anything tomorrow?"
Mike: "I must arrange a couple of things. But I am free tonight."
Liz: "You might come to my place. What time shall we meet?"
Mike: "I'll be there at eight. Is it all right?"
Mike asked Liz _______________ . And he said ______________

Example : what she was doing there; he had not seen her since June

Liz explained that _______________ .

Mike wondered ______________ .

Liz told him that _______________ and that ______________ .

Mike wanted to know ____________________ .

Liz said that _____________ and that she ______________ .

And then she asked him if _____________________ .

Mike explained that _______________ .

But he added that ____________ .

Liz suggested that _________________ and asked him _____________ .

Mike said __________________ .

And then he asked ___________ .

Reported Speech– Change of Pronouns


Helen is in front of the class holding a presentation on London. As Helen is rather shy, she speaks with a very
low voice. Your classmate Gareth does not understand her, so you have to repeat every sentence to him.

Complete the sentences in reported speech (no backshift). Note the change of pronouns and verbs.

PEV 107 Page 57


 Helen: I want to tell you something about my holiday in London.
 Gareth: What does she say?
 You: She says that she wants to tell us something about her holiday in London.(Example)
 Helen: I went to London in July.
 Gareth: What does she say?
 You: She says that _________
 Helen: My parents went with me.
 Gareth: What does she say?
 You: She says that _________
 Helen: We spent three days in London.
 Gareth: What does she say?
 You: She says that _________
 Helen: London is a multicultural place.
 Gareth: What does she say?
 You: She says _________
 Helen: I saw people of all colours.
 Gareth: What does she say?
 You: She says that _________
 Helen: Me and my parents visited the Tower.
 Gareth: What does she say?
 You: She says that _________
 Helen: One evening we went to see a musical.
 Gareth: What does she say?
 You: She says that _________
 Helen: I love London.
 Gareth: What does she say?
 You: She says that _________
 Helen: The people are so nice there.
 Gareth: What does she say?
 You: She says that _________

PEV 107 Page 58


Vocabulary Enrichment
8 Best ways to learn English Vocabulary
1. Keep a list

Words are all around us. Sitting down and looking up words in a dictionary is not the only way to find new vocabulary. You can find
new words by talking with English speakers, watching shows in English, reading the news—anywhere, anytime. To keep track of new
words, keep a vocabulary list.

2. Use new words in conversations

Now that you have a list, it can be easy to forget about these words as you move on to new ones. This is especially true for common
words and words that you’re not sure how to use. The more you use the words, the better you’ll remember them.

3. Play games

Games are a good way to learn because they make learning fun, and they help give you context for your new words. That means
you’re giving the word you learn an extra meaning. For example, you might remember the way that the word was used during the
game. You might remember your friend laughing about how the word was used. You might remember that it was very hard to think of
that word the first time while playing! Having a real-life memory attached to that word makes it much more memorable.

4. Don’t try to remember words alone

As we already mentioned, it is better (and easier) to learn new vocabulary by giving them some context. One way to do this is to
remember words in a sentence. This is a great option because you will not only know the word, but you will also know exactly how to
use it in conversation.

5. Learn word parts

Many words can be broken down into smaller parts. For example, the word “dictionary” comes from the Latin word dictio, which
means “to speak.”

This is called the root of the word. A root word is a base used to create many words. Now that you know the root word dictio, you
might notice it in other words too, like dictate, dictator and contradict. Even if you don’t know what the words mean, you now know
that they have something to do with speaking.

6. Take vocabulary quizzes

Now that you’ve learned a few new words, how well do you remember them? You can have fun learning, but you need to make sure
that you are making good progress.

7. Learn with others

Learning is easier and more fun when you do it with others! Find a group of friends who want to learn English with you, get an online
native speaking buddy or join a website with other learners. Whatever you choose to do, you will benefit greatly from working with
others.

8. Ask “What does that mean?”

Finally, never be afraid to ask questions! If someone uses words you don’t understand, ask them “what does that mean?” Many people
are very patient and understanding if you tell them that you’re learning English. In fact, many will actually enjoy helping you!

PEV 107 Page 59


One Word Substitution
Let’s start with Word Root method:

1. Thei 2. Logy
Thei meaning god/religion Logy means study of something
Ist meaning follower o Theology - Teho means god or religion so theology
Now adding different words means study of religion
o Theist meaning the one who believes in god o Anthropology - Anthrop means human being so
o Atheist meaning the one who don’t believe in god. anthropology means study of human being.
o Monotheist mono+theist mono(only one) so o Morphology - Morph means structure so
monotheist means the one who believes in one morphology means study of structures.
god. o Anthropomorphism - Its antrop+morph+ism
o Polytheist Ploy means many so polytheists means meaning is giving human structure to god.
the one who believe in many gods. o Zoomorphism - Its zo+morph+ism meaning giving
o Pantheist Pan mean all over so pantheist is one animal structure to god.
who believe in all gods across all religions.

3. Phobia 4. Cracy
Cracy means govt /system
Phobia means fear or dislike o Democracy - demo+cracy means govt of people
o Autocracy - auto+cracy means
o Zoophobia - zoo+phobia means fear of animal
dictatorship/kingship (when one person rules the
o Androphobia - Andro means boys/male so
whole government or system)
androphobia means the one who don’t like boys.
o Theocracy - Theo+cracy means when government is
o Gynophobia - Gyno means girl / women so
of religious people like priest, bishops etc.
gynophobia means the one who don’t like girls
o Plutocracy - Pluto means wealthy so plutocracy
o Pedophobia - Pedo means child so pedophobia is
means when government is controlled by wealthy
the one who don’t like children’s.
people like industrialist.
o Demophobia - demo means people so
o Oligocracy - Oligo means selected few so oligocracy
demophobia means the one who fear of public
means when selected few runs the government.
speaking.

5. Somnia 6. Cide
Somnia sleepness
Cide meaning kill/ murder
Insomnia means lack of sleep
o Infanticide - infant+cide meaning to kill child.
Somniferous - ferrous means things causing something so o uxoricide - uxor means wife so uxoricide mean A
somniferous are the things that cause sleep like pills etc. husband who murders his wife.
o Somnambulist - som+amb+ist here som means o Mariticide - marit means husband so maritcide
sleep ambu means walk so somnambulist means mean The murder of a husband by his wife.
the one who walk while sleeping. o Patricide - patri means father so patricide means A
o Ambulant - means things able to walk person who murders their father.
o Somniloquist - loquist means talking so o Matricide - matri mean mother so matricide means
somniloquist means the one who talks while A person who murders their mother.
sleeping.

PEV 107 Page 60


7. Bene 8. Cata

Good, well Down, against completely intensive according to

o Benefactor- person who gives money to a cause o Cataclysm- a flood or other disaster
o Beneficial- producing a good effect o Catalogue- a complete listing
o Benevolent- showing kindness or goodwill o Catastrophe- turning for the worst, a substantial
disaster

9. Cious 10. Anti


Cious means to do something Against, opposite of

o Loquacious - loquis means talking so lozuacious


means the one who talks a lot o Antibody- a substance that destroys micro-
o Voracious - vora means eat so voracious means organisms
the one who eats a lot. o Antiseptic- preventing infection;
o Avaricious - avari means greed so avaricious o Antisocial- opposing social norm
means greedy
o Pugnacious - pug means fight so pugnacious
means the one who fights a lot.

11. Cert 12. Circum

sure Around, about

o Ascertain- to find out something with certainty; o Circumnavigate- to sail around


o Certain- being absolutely sure o Circumscribe- to draw around
o Certify to state that something is true o Circumspect- looking around
o Circumvent- to go around or bypass restrictions;

13. Cline 14. Counter

lean Opposite, contrary, opposing

o Inclination- a leaning toward o Counteract- to oppose the effects of an action


o Incline- a surface that slopes or leans o Countermand- to cancel a previous order
o Recline- to lean back and relax o Counteroffensive- attack against an attack

15. Dia 16. Dict

Through, between, apart, across speak

o Diabetes- disease characterized by excessive thirst o Contradict- to express the opposite of


and discharge of urine o Prediction- a statement foretelling the future
o Diagnosis- understanding a condition by going o Dictate- to speak out loud for another person to
through a detailed review of symptoms write down.
o Dialog- conversation between two people

PEV 107 Page 61


17. Ethno 18. Funct

Race, people Perform, work

o Ethnic- pertaining to a defined group od people o Defunct- no longer working or alive


o Ethnocentric- focusing on the ethnicity of people o Function- to work or perform a role normally
o Ethnology- the science of people and races. o Malfunction- to fail to work correctly.

19. Hetero 20. Inter

Different, other Between, among, jointly

o Heterogeneous- made up of unrelated parts o International- involving two or more countries


o Heteronyms- words with same spelling but o Intersection- place where roads come together
different meanings o Intercept- to stop or interrupt the course of.
o Heterodox- not conforming to traditional beliefs

21. Junct 22. Lab

join work

o Conjunction- a word that joins parts of sentences o Collaborate- to work with a person
o Disjunction- a disconnection o Elaborate- to work out the details
o Junction- a place where two things join. o Laborious- requiring a lot of hard work.

23. Lex 24. Liber

Word, law, reading free

o Lexicology- the study and history of words o Liberate- to set free;


o Alexia- los of the ability to read o Libertine- a person with a free, wild lifestyle
o Illegal- not authorized by the official rules or laws o Liberty- freedom.

25. Loc 26. Macro

place Large, great

o Dislocate- to put something out of its usual place o Macroevolution- large scale evolution
o Location- a place o Macromolecule- a large molecule
o Relocate- to move to a new place. o Macroeconomics- study of the overall forces of
economy

PEV 107 Page 62


27. Mand 28. Mega

To order Great, large, million

o Command- an order or instruction o Megalopolis- an area with many nearby cities


o Demand- a hard-to-ignore orer o Megaphone- a device that projects a loud voice
o Mandate- an official order o Mega structure- huge building or other structure

30. Migr
29. Meta
move
Change, after, beyond, between

o Immigrant- a person who moves to a mew country


o Metaphysics- study of nature and reality to settle
o Metamorphosis- a complete change of form o Migrant- person who moves from place to place
o Metastasis- the transmission of disease to other o Migration- the process of moving
parts of the body

31. Mort 32. Nav

death ship

o Immortal- living forever, unable to die o Circumnavigate- to sail around a place


o Mortal- certain to die o Naval- relating to a navy or warships
o Mortician- an undertaker o Navigate- to sail a ship through a place

33. Neo 34. Osteo

New, recent bone

o Neoclassic- a revival of classic form,


o Neo-colonialism- the indirect (“new”) economic o Osteoarthritis- inflammation caused by
and political control of a region by a more powerful degeneration of the joints
foreign power o Osteopathy- therapy that uses among others
o Neonatal- a newborn child, especially first few manipulation of the skeleton to restore health
weeks o Osteology- the study of bones

35. Pan 36. Poly

All, any, everyone Many, more than one

o Panacea- a cure for all diseases or problems o Polychrome- with many colours
o Panorama- an all- around view o Polyglot- a person fluent in many languages
o Pantheism- the worship of all gods o Polygon- shape with 3 or more straight sides
o Pandemic- affecting all

PEV 107 Page 63


37. San 38. Sol

health alone

o Sane- mentally healthy o Desolate- lonely, dismal, gloomy


o Sanitary- relating to cleanliness and health o Solitary- done alone , by yourself
o Sanitation- maintenance of public health and o Solo- a performance done by one person alone

39. techno 40. Tort

Technique, skill twist

o Technology- the practical application of knowledge o Contortion- a twisted shape or position


o Technocracy – rule of technology o Distort- to alter the shape or condition of
o Technologically- characterized by technology o Retort- reply in a manner that is supposed to
change the effect of something previously said

41. Trans 42. Uni

Across, beyond, through One, single

o Transcontinental- across the continent o Unicycle- a vehicle with one wheel


o Transfer- to move from one place to another o Unilateral- decided by only one person or nation
o Transport- to carry something across a space o Unique- the only one of its kind
o Unison- as one voice

43. Un 44. Vac

Not, opposite of, lacking empty

o Unabridged- not shortened o Evacuate- to empty a dangerous place


o Unfair- opposite of fair o Vacant- empty, not occupied
o Unfriendly- lacking friendliness o vacation- a time without work

45. Verb 46. Urb

word city

o Verbalize- to put into words o Suburb- residential area on the edge of a city
o Adverb- a word relating to a verb o Urban- relating to a city
o Proverb- a short saying that expresses a well- o Urbanology- the study of city life
known truth

PEV 107 Page 64


47. Ultra 48. Tri

Beyond, extreme,more than Three, once in every three, third

o Ultrahigh- extremely high o Triangle- a figure with 3 sides and 3 angles


o Ultramodern- more modern than anything else o Triathlon- an athletic contest with 3 events
o Untrasonic- sound waves beyond human hearing o Tricycle- a 3- wheel vehicle with pedals

49. Tetra 50. Tox

four poison

o Tetrapod- having 4 legs o Detoxification- the process of removing poison


o Tetrarchy- government by 4 rules o Toxic- poisonous
o Tetrose- a monosaccharide with four carbon atoms o Toxicology- the study of poisons
o Intoxicated- influenced by drugs

PEV 107 Page 65


Commonly Misspelled Words

10 Golden Rules on Spelling Correction


Rule 1: When the suffix “full” is added to a word, one “ I” is removed.
Faith + full = faithful
Use + full= useful

Rule 2: If the word to which the suffix “full” is added ends in “ll”, one “I” is removed from the word also.
Skill +full = skilful
Will + full= wilful

Rule 3: Words of two or three syllables ending in single vowel + single consonant double the final consonant if the last syllable is
stressed. e.g.
Permit + ed = permitted
Occur + ing =occurring
Control + ed =controlled
Begin + ing = beginning

Rule 4: Consonant ‘L’ is doubled in the words ending in single vowel + “I” before a suffix beginning with a vowel e.g.
Signal + ing = signalling
Repel + ent = repellent
Quarrel + ed = quarrelled
Travel + er = traveller

Rule 5: Words ending in silent “e”, drop the “e” before a suffix beginning with a vowel e.g.
Hope + ing = hoping
Live + ed = lived
Drive + er = driver
Tire + ing= tiring

Rule 6: If the suffix begins with a consonant “e” is not dropped e.g
Hope + full = hopeful
Sincere + ly= sincerely
But,
True + ly = truly
Nine + th = ninth
Argue + ment = argument

Rule 7: A final “y” following a consonant changes to “i” before a suffix except “ing”. e.g.
Carry + ed = carried
Happy + ly = happily
Marry + age = marriage
Beauty + full = beautiful
But,
Marry + ing = marrying
Carry + ing = carrying

Rule 8: A final “y” following a vowel does not change before a suffix. e.g:
Obey + ed = obeyed
Play + er = player
Pray + ed= prayed

PEV 107 Page 66


Rule 9: When the suffix “ous” is added to a word ending in “ce”, “e” is changed to “i”.
Space + ous= spacious
Vice + ous= vicious
Malice + ous = malicious
Grace + ous= gracious

Rule 10: When the suffix “ing” is added to a word ending in “ie”, “ie” is changed to “y”.
Lie + ing= lying
Die + ing = dying
Tie + ing= tying
Hie + ing= hying
Vie + ing= vying

Correct spelling Spelling advice Common misspelling

accommodate, accommodation two cs, two ms accomodate, accomodation

achieve i before e acheive

across one c accross

aggressive, aggression two gs agressive, agression

apparently -ent not -ant apparantly

appearance ends with -ance appearence

argument no e after the u arguement

assassination two double s’s asassination

basically ends with -ally Basicly

beginning double n before the -ing Begining

believe i before e beleive, belive

business begins with busi- Buisness

chauffeur ends with -eur Choffer

colleague -ea- in the middle Collegue

committee double m, double t, double e Commitee

conscious -sc- in the middle Consious

curiosity -os- in the middle Curiousity

definitely -ite- not –ate- Definately

dilemma -mm- not -mn- Dilema

disappear one s, two ps Dissappear

disappoint one s, two ps Dissappoint

PEV 107 Page 67


ecstasy ends with –sy ecstacy

embarrass two rs, two s’s Embarass

environment n before the m Enviroment

fluorescent begins with fluor- Florescent

foreign e before i Forein

foreseeable begins with fore- Forseeable

forty begins with for- Fourty

friend i before e Freind

further begins with fur- Futher

gist begins with g- Jist

glamorous -mor- in the middle Glamourous

government n before the m Goverment

guard begins with gua- Gard

happened ends with -ened Hapened

harass, harassment one r, two s’s harrass, harrassment

humorous -mor- in the middle Humourous

idiosyncrasy ends with -asy Idiosyncracy

incidentally ends with -ally Incidently

independent ends with -ent Indepedent

interrupt two rs Interupt

irresistible ends with -ible Iresistible

knowledge remember the d Knowlege

millennium, millennia double l, double n millenium, milennia

occasion two cs, one s ocassion, occassion

occurred, occurring two cs, two rs occured, ocurring

occurrence two cs, two rs, -ence not - occurance, ocurrence


ance

pavilion one l Pavillion

persistent ends with -ent Persistant

PEV 107 Page 68


piece i before e Peice

possession two s’s in the middle and Posession


two at the end

preferred, preferring two rs prefered, prefering

propaganda begins with propa- Propganda

publicly ends with –cly Publicaly

receive e before i Recieve

referred, referring two rs refered, refering

resistance ends with -ance Resistence

separate -par- in the middle Seperate

siege i before e Seige

successful two cs, two s’s Sucessful

supersede ends with -sede Supercede

surprise begins with sur- Suprise

tattoo two ts, two os Tatoo

tendency ends with -ency Tendancy

therefore ends with -fore Therefor

threshold one h in the middle Treshold

tomorrow one m, two rs tommorow

tongue begins with ton-, ends with - Tounge


gue

truly no e Trule

unforeseen remember the e after the r Unforseen

unfortunately ends with -ely Unfortunatly

until one l at the end Untill

weird e before i Wierd

wherever one e in the middle Wherver

PEV 107 Page 69


Tutorial
Exercises
1. Group A: To E or Not to E?

Some of the words in parentheses require the addition of the letter e; others are correct as they stand.

1. Gus is (tru-ly) sorry for keeping you awake last night.


2. We were criticized (sever-ly) by the sewing circle.
3. The shed was (complet-ly) demolished.
4. Merdine was (sincer-ly) grateful for the reprieve.
5. The Simpsons are (argu-ing) again.
6. They (argu-d) last night for hours.
7. When is Mr. Wolfe (com-ing) home.
8. Maya is (writ-ing) her autobiography.
9. Mr. White is (judg-ing) the essay contest.
10. Be (car-ful) when you light the furnace.

2. Group B: IE or EI?

Some of the words require the addition of ie; others require ei.

1. Paint the (c--ling) before you paint the walls.


2. Gus has been (rec--ving) threatening e-mails.
3. A (w--rd) noise came out of the attic.
4. I paid almost five dollars for that (p--ce) of pie.
5. I don't (bel--ve) in coincidence.
6. The guests brought (th--r) children to the wedding.
7. (N--ther) of us can help you today.
8. The (n--ghbors) complained about our parents.
9. Linda (w--ghs) less than her collie.
10. We waited for the (fr--ght) train to pass.

3. Group C: A, E, or I?

Complete each word with the letter a, e, or i.


1. I borrowed these flowers from the (cemet-ry).
2. My parakeet eats huge (quant-ties) of bird seed.
3. The tax cuts will (ben-fit) the wealthy.
4. It was a (priv-lege) to meet you.
5. He has a remarkably (unpleas-nt) disposition.
6. We placed the puppies in (sep-rate) rooms.
7. Henry is an (independ-nt) thinker.
8. I found an (excell-nt) excuse to resign from the committee.
9. She chose items from different (cat-gories).
10. Professor Legree made another (irrelev-nt) remark.

4. Group D: Single or Double?


Some of these words require the doubling of a consonant; others are correct as they stand.

1. The sun was (shin-ing) down like honey.


2. The experiment was (control-ed) by a madman.
3. The region is slowly (begin-ing) to recover.
4. Doug (pour-ed) sugar over Yoddy's oatmeal.
5. She keeps (forget-ing) to call me.
6. Gus (admit-ed) his mistake.

PEV 107 Page 70


7. They were (sweat-ing) in the classroom.
8. That idea never (occur-ed) to me.
9. The bunny went (hop-ing) down to the abattoir.
10. His doctor (refer-ed) him to a dimple specialist.

DIRECTIONS for the question 1 - 15: Choose an option, which can be substituted for a given word/sentence/phrase out
of given options.
Question No. : 1 A person who thinks only of himself

a) Egoist
b) eccentric
c) proud
d) boaster

Question No. : 2 Savitri travels by foot. She is a

a) traveller
b) stickler
c) disciplinarian
d) pedestrian

Question No. : 3 In Magadh there was the government by a king or queen

a) democratic
b) monarchy
c) plutocracy
d) autocracy

Question No. : 4 This is a practice of having several wives.

a) polygamy
b) dotage
c) monogamy
d) bigamy

Question No. : 5 A life history written by somebody else

a) biography
b) autobiography
c) anthropology
d) ornithology

Question No. : 6 The act of murder of a human being.

a) matricide
b) patricide
c) homicide
d) suicide

Question No. : 7 Words written on the tomb of a person

a) manuscript
b) inscription
c) Epitaph
d) engrave

PEV 107 Page 71


Question No. : 8 Rohit is greedy for money. His collegues call him

a) avaricious
b) spendthrift
c) splendid
d) cynic

Question No. : 9 A person who hates women

a) cruel
b) misogynist
c) misanthropist
d) philanthropist

Question No. : 10 A person who insists on something

a) Disciplinarian
b) Stickler
c) Instantaneous
d) Boaster

One Word Substitution


1) A bicycle for two or more people

A) Barrow
B) Toboggan
C) Tricycle
D) Tandem

2) A body of persons appointed to hear evidence or judge and give their verdict (decision)

A) Committee
B) Audience
C) Lawyer
D) Jury

3) A brief or a short stay at a place

A) Halt
B) Sojourn
C) Intermission
D) Interlude

4) A broad road bordered with trees

A) Sanctuary
B) Facade
C) Boulevard
D) Boudoir

5) A building in which aircraft are housed

A) Granary
B) Garage
C) Hanger

PEV 107 Page 72


D) Dockyard

6) A building where an audience sits

A) Stadium
B) Gymnasium
C) Auditorium
D) Assembly

7) A child born after death of his father

A) Orphan
B) Posthumous
C) Bastard
D) Progenitor

8) A collection of slaves

A) Crew
B) Company
C) Cortege
D) Coffle

9) A decorative ring of flowers and leaves

A) Wreathe
B) Wreth
C) Wrath
D) Wreath

10) A den for small animals

A) Dyke
B) Shed
C) Hutch
D) Cave

11) A disease that affects a large number of people in an area at the same time

A) Academic
B) Incorrigible
C) Contagious
D) Epidemic

12) A disease which is spread by direct contact

A) Incurable
B) Infectious
C) Fatal
D) Contagious

13) A doctor who specializes in the eye diseases of the eyes

A) Geologist
B) Ophthalmologist
C) Usurer

PEV 107 Page 73


D) Oncologist

14) A doctor who treats children

A) Gynaecologist
B) Physiologist
C) Cardiologist
D) Paediatrician

15) A dramatic performance

A) Mascot
B) Masque
C) Mask
D) Mosque

16) A drawing on transparent paper

A) Red print
B) Transparency
C) Blue print
D) Manuscript

17) A figure of speech by which a thing is spoken of as being that which it only resembles

A) Alliteration
B) Simile
C) Metaphor
D) Personification

18) A fixed orbit in space in relation to earth

A) Geo-stationary
B) Geo-centric
C) Geo-synchronous
D) Polar

19) A former student of a school, college or university

A) Genius
B) Learner
C) Scholar
D) Alumnus

20) A four footed animal

A) Scullery
B) Chaperon
C) Quadruped
D) Matron

PEV 107 Page 74


ESSAY WRITING
What is writing?

This concise, supplemental text helps students understand the intricacies of grammar in order to improve their writing skills. A
variety of well-known literary quotes and excerpts contextualize grammar principles and are accompanied by a range of simple to
complex writing exercises. The author uses the method of first breaking down the quotation into its grammatical elements, and then
rebuilding it to examine how the elements and devices create or enhance meaning, demonstrating how the featured writer has used
a sophisticated knowledge of language to communicate effectively.

The major types of essays:

1) Narrative Essays: Telling a Story


2) Descriptive Essays: Painting a Picture
3) Expository Essays: Just the Facts
4) Persuasive Essays: Convince Me
5) Argumentative Essays: Logic/ Argument

Narrative Essays: Telling a Story

In a narrative essay, the writer tells a story about a real-life experience. While telling a story may sound easy to do, the narrative
essay challenges students to think and write about themselves. When writing a narrative essay, writers should try to involve the
reader by making the story as vivid as possible. The fact that narrative essays are usually written in the first person helps engage the
reader. “I” sentences give readers a feeling of being part of the story. A well-crafted narrative essay will also build towards drawing a
conclusion or making a personal statement.

Descriptive Essays: Painting a Picture

A descriptive essay paints a picture with words. A writer might describe a person, place, object, or even memory of special
significance. However, this type of essay is not description for description’s sake. The descriptive essay strives to communicate a
deeper meaning through the description. In a descriptive essay, the writer should show, not tell, through the use of colorful
words and sensory details. The best descriptive essays appeal to the reader’s emotions, with a result that is highly evocative.

Expository Essays: Just the Facts

The expository essay is an informative piece of writing that presents a balanced analysis of a topic. In an expository essay, the writer
explains or defines a topic, using facts, statistics, and examples. Expository writing encompasses a wide range of essay variations,
such as the comparison and contrast essay, the cause and effect essay, and the “how to” or process essay.
Because expository essays are based on facts and not personal feelings, writers don’t reveal their emotions or write in the first
person.

Persuasive Essays: Convince Me

While like an expository essay in its presentation of facts, the goal of the persuasive essay is to convince the reader to accept the
writer’s point of view or recommendation. The writer must build a case using facts and logic, as well as examples, expert opinion,
and sound reasoning. The writer should present all sides of the argument, but must be able to communicate clearly and
without equivocation why a certain position is correct.

Argumentative Essays: Logic/ Argument

This is the type of essay where you prove that your opinion, theory or hypothesis about an issue is correct or more truthful than
those of others. In short, it is very similar to the persuasive essay (see above), but the difference is that you are arguing for your
opinion as opposed to others, rather than directly trying to persuade someone to adopt your point of view.
PEV 107 Page 75
 Formal Writing Style
 Complex – Longer sentences are likely to be more prevalent in formal writing. You need to be as thorough as possible with your
approach to each topic when you are using a formal style. Each main point needs to be introduced, elaborated and concluded.

 Objective – State main points confidently and offer full support arguments. A formal writing style shows a limited range of
emotions and avoids emotive punctuation such as exclamation points, ellipsis, etc., unless they are being cited from another
source.

 Full Words – No contractions should be used to simplify words (in other words use "It is" rather than "It's"). Abbreviations must
be spelt out in full when first used, the only exceptions being when the acronym is better known than the full name (BBC, ITV or
NATO for example).

 Third Person – Formal writing is not a personal writing style. The formal writer is disconnected from the topic and does not use
the first person point of view (I or we) or second person (you).

Do’s of Essay Writing:

Descriptive Elements:

1. Think about the dominant impression you want to give; don’t attempt to describe every single thing.
2.Use imagery to put your reader right there with you.
3. Order events in time and space in such a way that the reader can imagine the location and time easily.
4. USE ADJECTIVES AND ADVERBS! Instead of: It was hot. In the afternoon, we rested on a log before hiking further into the woods.

Try: Once the afternoon sun began to blaze down upon us, exhaustion set in, and we collapsed on a nearby moss-covered log,
groaning under the weight of its ancient branches. Our hike into the heart of the forest would soon resume.

5. USE A THESAURUS! Instead of: old , Try: elderly, aged, long in the tooth clean try: spotless, hygienic, sterile

Instead of good, Try: marvelous, wonderful, incomparable, unrivaled


Instead of bad, Try: dreadful, horrific, appalling, ghastly.
Don’ts of Essay Writing:

1. Using contractions (for example ‘I don’t think’ or ‘We can’t say’ ) instead of the full form (‘I do not think’ or ‘we cannot say.’) ever
use contractions in academic writing.

PEV 107 Page 76


2. Writing too many words. The examiner is paid to mark on an ‘essay per hour basis,’ and so will not read the end of an essay if it
exceeds the minimum word count by more than about 100 words. This means he or she will not see the end of your argument,
and your score will reduce considerably.
3. Using informal words (for example ‘a nice idea’ or ‘a silly thing to do’) instead of academic words (for example ‘a positive idea’ or
‘a regrettable thing to do.’) Remember that academic vocabulary is different from the language you would use in English when
talking to friends.
4. Giving personal opinion in an IDEAS type Task. Check if the Task is asking for your opinion or not. The first question you should
ask yourself is ‘Is this an OPINION or an IDEAS Task?’
5. Telling stories about your personal history, friends or family. The Task tells you to use ‘examples from your own experience,’ but
this does not mean describing stories from your life or people you know! It means describing examples of things in the world that
you know about, have studied or have learned about in the media.
6. Giving evidence which is too detailed or specific to a subject. You may be an expert in a particular social or scientific field, but the
examiner probably has a different specialty. You need to make your ideas and examples accessible to a general reader. For
example, if the Task topic is about money and you are an accountant, do not use specialized accounting terms.
7. Being emotional or too dramatic when giving your opinion in an OPINION Task. You may feel strongly about issues such as
animals or crime, but academic writing must be unemotional. So avoid phrases such as ‘a disgusting idea’ or ‘I detest this concept.’
It is much better to say ‘an unacceptable idea’ or ‘I disapprove of this concept,’ which is more impersonal and
8. Academic; similar to the type of writing that people use in business reports or university essays. Not following the basic structures
presented in this book. The examiners want to see a clear, well-structured essay that is easy to read. They are accustomed to
seeing the structures we have presented in this book, and they will feel more positive about your essay if they can recognize these
structures in what you write.
9. Avoid ‘lazy’ expressions (“and so on”, “etc”).
10. Avoid SMS Language: We all are typing SMS messages, chatting on Skype and the like, and there is a bunch of shorter ways to
write longer words. We type “u” instead of “you”, “c” instead of “see”, “IMHO” instead of “in my humble opinion”. None of these
can appear in your IELTS exam, unless you are specifically trying to mess up and get a lower score than you deserve. You need to
write the full word and spell it correctly, period.

The essay has the following standardized format:

1) Introduction

2) Elaboration

3) Conclusion

PEV 107 Page 77


Something more about writing Introduction:

An attention-grabbing "hook"( To introduce topic. Example: write down mobile and its usage. instead of directly starting from
usage of mobile phones, start it with Technology and scenario.)

A thesis statement( demand of the essay, what should be written in body paragraphs and your opinion too if required.) Something
more about Thesis statement: A very basic thesis statement is one or two sentences at the end of the first paragraph that tells the
reader the main idea of your essay.

The Functions of a Thesis Statement:

• To announce the topic to the reader


• To reflect a judgment about the topic
• To provide the reader with a blueprint
• of what is to come in the paper

Sample:

As people live longer and longer, the idea of cloning human beings in order to provide spare parts is becoming a reality.
The idea horrifies most people, yet it is no longer mere science fiction.

Model Introduction for Human Cloning Essay Topic or rephrasing

The cloning of animals has been occurring for a number of years now, and this has now opened up the possibility of cloning
humans too. Although there are clear benefits to humankind of cloning to provide spare body parts, I believe it raises a number
of worrying ethical issues.
Thesis
Body Paragraph 1: Topic Sentence:
Main Points: 1.
2.
3.

Body Paragraph 2: Topic Sentence:


Main Points:1.
2.
3.

PEV 107 Page 78


Sample Body Paragraphs: Due to breakthroughs in medical science and improved diets, people are living much longer than in the
past. This, though, has brought with it problems. As people age, their organs can fail so they need replacing. If humans were cloned,
their organs could then be used to replace those of sick people. It is currently the case that there are often not enough organ donors
around to fulfil this need, so cloning humans would overcome the issue as there would then be a ready supply.

However, for good reasons, many people view this as a worrying development. Firstly, there are religious arguments against it. It
would involve creating another human and then eventually killing it in order to use its organs, which it could be argued is murder.
This is obviously a sin according to religious texts. Also, dilemmas would arise over what rights these people have, as surely they
would be humans just like the rest of us. Furthermore, if we have the ability to clone humans, it has to be questioned where this
cloning will end. Is it then acceptable for people to start cloning relatives or family members who have died?

Conclusion: To conclude, I do not agree with this procedure due to the ethical issues and dilemmas it would create. Cloning
animals has been a positive development, but this is where it should end.

Another Sample: Popular events like the Football World Cup and other international sporting occasions are essential in easing
international tension and releasing patriotic emotions in a safe way.

Every four years, the whole world stops to watch international sporting events such as the Olympics and the Football World Cup in
which athletes show their best performance to make their country proud. These sporting occasions have proved to be helpful in
easing international tension in difficult times when powerful leaders were trying to control the world’s economy and other
governments were fighting over the land.

The Olympic Games are one of the best examples which prove how sporting events can bring nations together, at least temporarily.
From the ancient History, when Greeks and Romans would interrupt battles to participate in the games, to the more recent
international disputes, when athletes from Palestine and Israel would forget their differences, compete peacefully and even
embrace each other after an event. Moreover, these popular events have called the world’s attention to the terrible consequences
of wars; thus some leaders have tried to reach agreements to end their disputes and live peacefully.

Similarly, international sporting events show benefits in some developing countries which live in a daily internal civil war. For
example, Brazil has a high rate of unemployment, lack of education, hunger, crime, poverty and corruption which leads to an
immense embarrassment of being Brazilian and a low self-esteem. However, when the Football World Cup starts, the Brazilian
squad, which is considered the best team in the world, provokes an amazing feeling of pride in their country. Most people seem to
forget all their problems and even the criminal activity decreases. They paint roads with the national colors, wear the Brazilian team

PEV 107 Page 79


shirts and buy national flags. Moreover, the competition brings families and neighbors together and even rival gangs watch the
games and celebrate peacefully.

In conclusion, popular sporting events play an important role in decreasing international tensions and liberating patriotic feelings as
history has shown.

Useful Phrases for essay writing:

 Introducing the topic


 Currently / These days …
 These days, …
 Over the few past decades…
 Issues related to … are frequently discussed these days
 This essay will discuss … / will examine the arguments for and against / will examine the reasons why people … / will consider the
positive and negative impacts of this and discuss ways to…
 In my opinion, the potential dangers from this are greater than the benefits we receive.
 However, … is a contentious issue
 Although many people believe that…, others think that
 Although there are arguments on both sides, I strongly agree that…
 While early technologies certainly changed the lives of normal people, I believe that …

 Stating an opinion (thesis statement), general phrases


 I believe that… / It is evident that / It has been claimed that… / It is widely believed that… / It is certainly true to say that… / It is
accepted that… / Many people consider that…
 There is no doubt that…
 Not only … but it could also…
 First and foremost, …
 The principal reason that people … is because
 Smth has increasingly … / people are increasingly using
 Smth take advantage of (natural light)
 There are many advantages (benefits)to (using the Internet)
 Smth may be endangered
 Smth aim to (teach young people moral values)

PEV 107 Page 80


 Government should introduce …
 Smth is at risk of …
 … a responsibility towards (the older people)

 Giving reasons
 One reason for this is that…
 This can be explained by…
 This can be attributed to the fact that…
 It should also be stated that…

 Refuting opinions
 I would dispute this, however.
 However, this is not necessarily the case.
 There are some faults with this reasoning, however.
 On the other hand, …

 Concluding and summarising


 In conclusion, To conclude…
 To sum up…
 Overall, … / On the whole, …
 I strongly believe that…
 In conclusion, it is clear that
 The advantages (benefits)of … outweigh the disadvantages (drawbacks).

 Linking phrases
Nevertheless, however, although, in addition, therefore, furthermore, moreover, also, despite this, finally, in fact, indeed,
consequently, eventually, thereafter … (see speaking brief file)

Templates for Essay:


Introduction

Trend description

- It is certainly true that / There is no doubt that X is increasingly being used to Y nowadays.
- It is certainly true that [populations in developed countries] are increasingly [overweight nowadays].
- Nowadays more and more [specialists] are Z to Y
- Over the past few decades, Z has improved dramatically, [allowing Y to X] [introducing innovative X], which has changed many
aspects of our everyday life
- Over the decades/centuries, [generations of] scientists have been struggling to find answers to the most complicated questions
about X. However, some of them have not been resolved yet. OR been struggling to understand the nature of X
- Nowadays many people increasingly prefer X rather than Y
- Nowadays parents/people can choose whether to Z or X
Problem

- It is certainly true that people are increasingly [producing more garbage], which has had a significant negative effect on the
environment.
- While many people believe that the situation may be improved by Y, I do not consider this as an effective measure and think that
other approaches can be more useful.

People think

PEV 107 Page 81


- It is certainly true that issues/questions related to Z, [such as Y], are frequently discussed these days/nowadays since/due to the
fact that... In particular, many people complain that…
- It is often argued that …
- It is currently a contentious argument over whether…
- It is certainly true that X is a contentious topic for many adults/people.
- It is certainly true that the attitude towards [higher education] is different all over the world.
- It is certainly true that X is a contentious topic which raises many ethical questions.
- There are different opinions in regard to the question of [who should teach X about Z]
- It is certainly true to say that such phenomenon as X is frequently discussed nowadays.
- This still remains a controversial topic for many people.
- These factors have led to a phenomenon known as Z, which is criticized by many people.
- Some people consider this phenomenon as negative, whereas others believe that they benefit from Z / whereas I believe that
society can benefit from Z.
- A commonly held belief is that...

Body
Body paragraph opinion essay starting

- The main reason why I am convinced that Z is because Y.


- Another reason why I support the idea that Y is that X.
Estimation of good / bad

- Z can be beneficial in many ways / have a positive (favorable) effect


- Z increases the quality of everyday life.
- Z can have an adverse effect on [youths] / can affect [youths] adversely.
- These factors can have a negative effect on Z
- X can pose a threat to [healthy relationships]
- The benefits / the potential positive effects [of X] are outweighed by a number of drawbacks.
- There are several obvious shortcomings related to X.
- The downsite (singular!) of ... is that
Agree/disagree

- Somebody subscribe to the view that...


- Somebody concur that (agree without hesitation)
Arguing

- Although Y, this is not necessarily the case nowadays.


- Another significant reason why X is …
- Smth can produce noticeable increase in the quality of life.
- Many factors may be more important than X
- Thus, it has subsequently become apparent that …
- From this it becomes evident that …
- Thus, it is clear that …
Problems

- [The increase in X] has been caused by a variety of factors

PEV 107 Page 82


- Something stems from (there is a direct link)
- The effects of this are extremely serious.
- Y may not be the best way to tackle the issue for several reasons.
- Alternatively, a number of comprehensive measures can be taken to solve X problems.
- The best way to solve the problem would be to …
- By doing this, they would [feel better generally]
- It is important that ...
Info and data support

- Z is a key part of modern business.


- From my knowledge, some recent stories in the media support this belief/point of view.
- Some recent stories in the media support the belief that … (stories – better not use in opinion essay)
- This is yet another example of how ...
- This is/was true of somebody, who ...

Conclusion
Restating estimation of good / bad

- There are a number of advantages and disadvantages in regard to both approaches.


- The benefits of Z are outweighed by a number of disadvantages / by several drawbacks.
- This may have either a positive or a negative effect, depending on X
Stating approaches

- I think that Z should supplement Y with the use/means of N.


- Otherwise, potential dangers may overweight possible benefits.
- People should be aware of risks related to X and [use modern means of communication] wisely.
- [Every adult] should think thoroughly/deliberately [prior to encouraging their children to do so].
- There is no reason why [x and y] cannot exist together.
Problems

- I am convinced that problems related to X should be tackled not only by Y, but by the implementation of other measures as well.
This can include…
- In conclusion, X is clearly alarming, and Z should tackle the issue by the implementation of a set of comprehensive measures.
- As a result, … which will have a significant positive effect on [the environment].
- I am convinced that such a comprehensive approach is the best way to eliminate the root cause of many [social] problems.
- Unless these actions are taken, the devastating effect on [the environment] may be irreversible.

Tutorial
EXERCISE: Identify the type of essay in each statement.

1.According to some people, elderly drivers should be required to reapply for their driving licenses because with age comes
diminished vision, hearing, and reaction time. How do you feel about this issue? Explain what you think should be done and why.

2.Have you ever traveled to a place that you found very meaningful and rewarding? Write an essay that persuades others to visit this
important place.

PEV 107 Page 83


3.If it were up to you to choose one item from the twenty-first century to place in a time capsule for future generations, what would
you choose? Use specific reasons and examples to support your choice, explaining both the item’s significance and the reasons why
it embodies the culture of the early twenty first century.

4.Explain the causes and effects of not voting in elections.

5.Many people spend a great deal of time with animals. Write about the relationships that people have with animals.

6.People often say “Don’t judge a book by its cover.” Describe a time when you misjudged someone based on his or her appearance
or when someone misjudged you.

7. Carpooling, recycling, and planting trees are all activities that are good for the environment. Write an Essay convincing readers to
actively participate in one of these activities.

EXERCISE 2: Write an essay in 300-350 words

1. Right To Privacy
2. Concept of new India
3. Demonitisation and its Impact on Indian Economy
4. Information Technology is a developing Technology
5. Adverts are trying to allure consumers these days and showing hidden facts about products. What do you think?
6. Are we happier than our forefathers?
7. The era of Coalition
8. Challenges to Indian Democracy
9. Cashless Economy
10. Has the Budget 2018 lost its excitement, with the introduction of GST?

PEV 107 Page 84


Reading Comprehension
Reading comprehension is the act of understanding what you are reading. Reading comprehension is an intentional, active, interactive
process that occurs before, during and after a person reads a particular piece of writing. Reading comprehension is one of the pillars of
the act of reading. There are two elements that make up the process of reading comprehension: vocabulary knowledge and text
comprehension. In order to understand a text the reader must be able to comprehend the vocabulary used in the piece of writing. If
the individual words don’t make the sense then the overall story will not either.

General Strategies for Reading Comprehension


There are six main types of comprehension strategies (Harvey and Goudvis; 2000):
1. Make Connections—Readers connect the topic or information to what they already know about themselves, about other texts,
and about the world.
2. Ask Questions—Readers ask themselves questions about the text, their reactions to it, and the author's purpose for writing it.
3. Visualize—Readers make the printed word real and concrete by creating a “movie” of the text in their minds.
4. Determine Text Importance—Readers (a) distinguish between what's essential versus what's interesting, (b) distinguish between
fact and opinion, (c) determine cause-and-effect relationships, (d) compare and contrast ideas or information, (e) discern themes,
opinions, or perspectives, (f) pinpoint problems and solutions, (g) name steps in a process, (h) locate information that answers
specific questions, or (i) summarize.

PEV 107 Page 85


5. Make Inferences—Readers merge text clues with their prior knowledge and determine answers to questions that lead to
conclusions about underlying themes or ideas.
6. Synthesize—Readers combine new information with existing knowledge to form original ideas, new lines of thinking, or new
creations.

8 Types of Reading Comprehension Questions

1. Universal or Main Idea Questions – This question-type asks about the big picture, the passage as a whole.
 “Which of the following best summarizes the passage?”
 “What is the author attempting to illustrate through this passage?”
 “What is the thematic highlight of this passage?”
 Which of the following best states the central idea of the passage?
 The gist of the passage is:
 Which of the following is the principal topic of the passage?
 “Which of the following best describes one of the main ideas ____?”

2. Specific – This type asks about details from the passage. The correct answer is often a paraphrase of something directly stated
in the passage.
 “Which _____ has not been cited as ______?”
 “According to the author, what is ______?”
 “By a _______, the author means…”
 “According to the passage, _______?”
 “Which factor has not been cited _______?”

3. Vocab-in-Context – This is a type of Specific question which asks about the use of a particular word or phrase.
 “In the passage, the phrase _____ refers to”
 “In the sentence, _______, what is the meaning of ‘___’?”

4. Function – This type of question asks about the logical structure of a passage.
 “The author cites ______, in order to”
 “The _____ in the passage has been used by the author to”
 “Which of the following best describes the reason the author ____?”

5. Inference – Inference questions require you to understand what is implied by but not necessarily stated in the passage. The
correct answer may rely on subtle phrases from the passage and be hard to find/less obvious than Specific questions.
 “The passage uses _____ to imply that ___”
 “Which of the following cannot be inferred from the passage?”
 “What does the author mean by _____?”
 “What can be inferred when the author states____?”
 The sentence, ‘______’, implies that”

6. Application – This is a slightly more specific type of inference question, where you’re asking to choose an answer which
mimics a process or exemplifies a situation described in the passage.
 Which of the following could be used to replace_______?
 “A _____, as conceptualized in the passage, can best be described as____”
 “Which of the following best illustrates the situation ____”

7. Tone – This question type is also a specific type of inference, requiring you to infer the author’s point of view and position on
certain statements.
 “The author of the passage is most likely to agree with ____?”
 “Which of the following views does the author most likely support__?”

8. Context outside passage - Like inference questions, applying information questions deal with topics that are not mentioned
explicitly in the passage. This type of question asks you to take the information given in the passage and apply it logically to a
context outside of the passage. An applying information question could look like:

PEV 107 Page 86


 Which of the following situations is most similar to the situation described in the paragraph?
 Identify a hypothetical situation that is comparable to a situation presented in the passage?
 Select an example that is similar to an example provided in the passage?

Main Idea 1
Rock, Paper, Scissors is sometimes used to reach a decision. Two players count to three as they pound one fist into the other,
and then each extends a hand in the shape of one of three symbols: scissor (twofinger “V”), paper (flat hand), or rock (closed
fist). These rules determine the winner: 1) scissor cuts paper; 2) paper covers rock; 3) rock breaks scissor.
A. To explain how to play Rock, Paper, Scissors
B. To tell people how to reach a decision
C. To entertain the reader with an exciting way to reach a decision
D. To describe the hand symbols in Rock, Paper, Scissors

Main Idea 2
George Washington is best known for being our first president. He laid the foundation for the great country we live in today.
Before he took on this important role, he was a leader in many other ways. He was an officer in the French and Indian war, in
1775 he became the commander-in-chief of the Continental Army, and he presided over the Constitutional Convention. All of
these important experiences helped mold Washington into the man he became as our president.
A. Washington was a president, leader of the army, and organizer for the Constitutional Convention
B. Washington was more than just our president
C. The United States would not be what it is today without George Washington
D. Washington led our country in many different roles

Main Idea 3
The attitudes of Americans toward gambling are amazingly contradictory. You may find, for example, that horse racing is legal
in your state, but that you cannot legally play poker for money on your front porch; bookies may be prosecuted by state law,
but they are supposed to purchase a federal license nonetheless; one church condemns gambling, while another raises money
by sponsoring Bingo games. Gambling laws are inconsistent from state to state or even from town to town and are very difficult
to enforce.
A. Americans have negative attitudes toward gambling.
B. Gambling laws are difficult to enforce
C. Gambling laws are inconsistent from community to community.
D. Churches do not have uniform ideas about gambling.

Specific - 1
Naval architects never claim that a ship is unsinkable, but the sinking of the passenger-and-car ferry Estonia in the Baltic surely should
have never have happened. It was well designed and carefully maintained. It carried the proper number of lifeboats. It had been
thoroughly inspected the day of its fatal voyage. Yet hours later, the Estonia rolled over and sank in a cold, stormy night. It went down
so quickly that most of those on board, caught in their dark, flooding cabins, had no chance to save themselves: Of those who
managed to scramble overboard, only 139 survived. The rest died of hypothermia before the rescuers could pluck them from the cold
sea. The final death toll amounted to 912 souls. However, there were an unpleasant number of questions about why the Estonia sank
and why so many survivors were men in the prime of life, while most of the dead were women, children and the elderly.
According to the passage, when the Estonia sank, ----.

A) there were only 139 passengers on board


B) few of the passengers were asleep
C) there were enough lifeboats for the number of people on board
D) faster reaction by the crew could have increased the Estonia's chances of survival
E) all the passengers had already moved out into the open decks

Vocab in context - 1
Her paintings are now very famous and, although the dark colours may not be attractive to some people, they evoke the beauty
and mystery of the deep woods and the skill of a great artist.
The word "Evoke"in the passage means:
a) reason
b) discover
c) make you feel
d) disclose

PEV 107 Page 87


Function – 1

Seeking a competitive advantage, some professional service firms (for example, firms providing advertising, accounting, or health care
services) have considered offering unconditional guarantees of satisfaction. Such guarantees specify what clients can expect and what
the firm will do if it fails to fulfill these expectations. Particularly with first-time clients, an unconditional guarantee can be an effective
marketing tool if the client is very cautious, the firm’s fees are high, the negative consequences of bad service are grave, or business is
difficult to obtain through referrals and word-of-mouth.
However, an unconditional guarantee can sometimes hinder marketing efforts. With its implication that failure is possible, the
guarantee may, paradoxically, cause clients to doubt the service firm’s ability to deliver the promised level of service. It may conflict
with a firm’s desire to appear sophisticated, or may even suggest that a firm is begging for business. In legal and health care services,
it may mislead clients by suggesting that lawsuits or medical procedures will have guaranteed outcomes. Indeed, professional service
firms with outstanding reputations and performance to match have little to gain from offering unconditional guarantees. And any
firm that implements an unconditional guarantee without undertaking a commensurate commitment to quality of service is merely
employing a potentially costly marketing gimmick.

Which of the following is cited in the passage as a goal of some professional service firms in offering unconditional guarantees of
satisfaction?

A A limit on the firm's liability


B Successful competition against other firms
C Ability to justify fee increases
D Attainment of an outstanding reputation in a field

Inference – 1
Jessie Street is sometimes called the Australian Eleanor Roosevelt. Like Roosevelt, Street lived a life of privilege, but she devoted her
efforts to working for the rights of the disenfranchised, including workers, women, refugees, and Aborigines. In addition, she gained
international fame when she was the only woman on the Australian delegation to the conference that founded the United Nations,
just as Eleanor Roosevelt was for the United States.

Which of the following inferences may be drawn from the information presented in the passage?
a. Eleanor Roosevelt and Jessie Street worked together to include women in the United Nations Charter.
b. Usually, people who live lives of privilege do not spend much time participating in political activities.
c. Discrimination in Australia is much worse than it ever was in the United States.
d. At the time of the formation of the United Nations, few women were involved in international affairs.

Application – 1
The piranha is a much-maligned fish. Most people think that this is a deadly creature that swarms through rivers and creeks of the
Amazon rainforest looking for victims to tear apart. And woe betide anyone unlucky enough to be in the same water as a shoal of
piranhas. It takes only a few minutes for the vicious piranhas to reduce someone to a mere skeleton. The truth is that the piranha is
really a much more nuanced animal than the mindless killer depicted in the media. In fact, piranhas are a group made up of
approximately twelve different species. Each piranha species occupies its own ecological niche. One type of piranha takes chunks out
of the fins of other fish. Another type eats fruit falling from trees into the river. Each piranha species plays a unique role in the
ecology of the rainforest floodplains. So what should you do next time you hear someone talking about the “deadly piranha”? You
can remind them that the piranha is not always the notorious killer fish that the tough, muscular heroes of popular nature television
shows would have us believe.

In paragraph 1, the author uses hyperbole—characterized by the use of exaggeration for effect—to describe the piranha. The
author most likely uses hyperbole to
A. frame an argument that is supported in a later paragraph
B. create ambiguity so the reader cannot be sure which position the writer supports
C. juxtapose the myth of the piranha with the truth about the fish
D. evoke vivid images of nature television shows in the reader’s mind

Identify the tone:

1. He shot the man.


2. He shot the man with his Beretta 91.
3. The question is, did that man deserve to get shot?
4. Evil doers deserve evil things, such as getting murdered.
PEV 107 Page 88
5. Funny how The gun collector was murdered by a knife.
6. His dead body is worth more than my entire property.

Tone :

Some of the common tones used by author are listed below:

 Critical : denotes negative or fault finding attitude of the author. Sometimes, the word critical is also used to denote deep
analysis of the issue with a neutral outlook.
 Laudatory/Eulogistic : To shower high praise on something or somebody.
 Cynical : A higher degree of pessimism coupled with a sense of skepticism and helplessness.
 Satirical : To use humour as a tool for healthy criticism.
 Sarcastic : To use the words opposite to what you mean to taunt or make fun of somebody.
 Didactic : When the author tries to teach or instruct through his writing.
 Nostalgic : Coveys a sense of longing for the past.
Context outside passage – 1
One may react to someone’s sarcastic comment with anger on one occasion but with amusement on another depending on current
emotional status. How responsive one is to another’s needs can vary significantly if, for example, an employee has just been
reprimanded by a superior or praised for excellence in customer relations, or, if someone a professor perceives as an excellent
student hands in sub-standard work. Perceptions are different for one who just learned of the death of a favorite relative or has been
informed that they have won the lottery.
-From Visual Literacy by David Moore and Francis Dwyer

Based on the passage, which statement about perceptions would the authors of the passage most likely agree?
A. One who is both perceptive and emotionally reactive most likely lacks mental stability.
B. Perceptions are wrong and should never be trusted.
C. Present feelings affect one’s perception.
D. One should react without being swayed by one’s perceptions.

PEV 107 Page 89


Solutions:
Main Idea 1 - A Main Idea 2 - D Main Idea 3 - C Specific 1 - C
Vocab in Function 1 - B Inference 1 - D Application 1 – C
Context 1 – C
Identify the tone:

1. Detached / Formal
This is a general highlight of an event. Not too descriptive, not too analytical, Hence, formal.
2. Analytical
This gives a few more details, and now we are analysing the given details, hence, analytical.
3. Critical
Critically questions the aspect, hence critical.
4. Disapproving / Cynical
There is a disapproval of the person, the evil doer.
5. Ironic
Self explanatory.
6. Mocking/ Sarcastic
A mocking statement, that even if he's (the evil doer) dead, the people analyzing the after effects (cops, or the Good
Guys), have little or no value, comparatively.

Tutorial
Exercises: Main Idea, Specific, Vocab, Function

1. Select the Main Idea: Someday we will all have robots that will be our personal servants. They will look and behave much like
real humans. We will be able to talk to these mechanical helpers and they will be able to respond in kind. Amazingly, the robots of
the future will be able to learn from experience. They will be smart, strong, and untiring workers whose only goal will be to make
our lives easier.

A. Someday we will all have robots that will be our personal servants.
B. We will be able to talk to these mechanical helpers and they will be able to respond in kind.
C. They will look and behave much like real humans.
D. Amazingly, the robots of the future will be able to learn from experience.

2. Read and select the main idea: Housework is the first thing to fall by the wayside in the time-pressured lives of professional
women. In a survey by Family Circle magazine, over 80 percent of full-time working women said the only way they could balance a
job, home, and family was to sacrifice high cleaning standards. The next thing to get put on the back burner was sex. Almost 75
percent said that by the end of the day, they or their spouses were too tired for anything but TV. Fifty percent reported that
friends and social events were third in line to get squeezed off the calendar.

A. According to a Family Circle survey, full-time working women have to cut back on several customary activities.
B. Women who work full-time say they must sacrifice high cleaning standards.
C. According to a Family Circle survey, full-time working women would be better off if they took on part-time jobs instead.
D. Half of all full-time working women report that working full-time has meant less time for getting together with friends

3. Read and Select the Main Idea: Newspapers had been carrying reports on the famine in Ethiopia for months. Although stories
described how people were starving, there was little response from the public. All that changed in October 1984, when NBC-TV
showed a five-minute report on the famine. As soon as the pictures of starving children were shown, the network's switchboard lit
up. This was the beginning of a huge worldwide effort to send aid to Ethiopia. Similarly, the television coverage of the Vietnam
War contributed to the growing anti-war movement.

A. The first network TV report on the Ethiopian famine was shown in October 1984.
PEV 107 Page 90
B. People who read newspaper reports are not moved by them.
C. Pictures can be more powerful than words alone in moving people into action.
D. TV viewers are more interested in international news than in local news.

4. Patients should always be aware of a prescription drug's possible side effects. Unexpected side effects, such as nausea or
dizziness, can be frightening and even dangerous. The consumer should ask, if he or she can take the medication along with other
drugs he or she is taking. Some combination of drugs can be lethal. Finally, medication should always be stored in its own labeled
bottle, never transferred to another. Accidental mix-ups of drugs can have tragic results. The main idea of this article is:

A. Consumers may use drugs carelessly.


B. Drugs can be dangerous.
C. There are several guidelines for taking prescription medications safely.
D. To avoid tragic mix-ups, medications should always be stored in their own labeled bottles.

5. There are more than 350 different kinds of sharks. Some are as long as a fire truck. Some are so small you could hold one in
your hand. Some sharks have dull teeth. Others have teeth so sharp they can take a bite out of a turtle's shell. Did you know that
some sharks even live in rivers? While other sharks hide on the ocean bottom camouflaging themselves so they can catch fish.
There are even some sharks that glow in the dark! Many people don't know how diverse the species of sharks’ are. The main idea
of this article is:

A. There are a lot of different types of sharks.


B. There are many different places a shark lives. They live in both rivers and oceans.
C. Sharks have different ways of eating and protecting themselves.
D. People don't know that much about the shark species.

6. Read the following passage: Something strange is happening to frogs. Many are growing extra legs, and some aren't growing
legs at all. Some frogs grow legs that are deformed. That means they aren't the right shape. When the frogs grow their legs
sometimes they split into two. Some scientists have found frogs with up to 9 legs. They think that fertilizer is changing the way
the frogs develop, or grow. What results is some very odd looking frogs. What would be the best title for this section? (Use what
you think the main idea to be to figure out a title)

A. DOUBLE TROUBLE
B. STOP FERTILIZING!
C. FREAKY FROGS
D. A 9 LEGGED WONDER

7. Read the following passage and answer the questions: In the 1800's, white Australians wanted to hunt. So, they brought
rabbits to eastern Australia. It was a bad idea. Soon, there was an overabundance, or way too many, rabbits. Rabbits started to
invade, or take over, farms in the western part of Australia. They ate up all the grass and the ground was left bare. They ate
people's crops, which they used for food. They ate everything they could and caused a lot of damage. They were also pests and
got into people's homes. After about a hundred years, the government couldn't deal with the pests any longer and they decided
to kill the rabbits. The main idea of this article is:

A. Rabbits in Australia ate people's crops and became pests in their homes.
B. The government came up with a solution and decided to kill the rabbits in Australia.
C. Rabbits are annoying animals.
D. In Australia, there were too many rabbits and it caused a lot of problems.

8. In the 1800's, white Australians wanted to hunt. So, they brought rabbits to eastern Australia. It was a bad idea. Soon, there
was an overabundance, or way too many, rabbits. Rabbits started to invade, or take over, farms in the western part of Australia.
They ate up all the grass and the ground was left bare. They ate people's crops, which they used for food. They ate everything
they could and caused a lot of damage. They were also pests and got into people's homes. After about a hundred years, the
government couldn't deal with the pests any longer and they decided to kill the rabbits. All of the following statements support
the idea that rabbits caused problems to Australia except:

PEV 107 Page 91


A. They invaded people's farms.
B. The white Australians brought the rabbits to eastern Australia so they could hunt. They had too many opportunities to hunt.
C. Rabbits ate people's crops.
D. They became pests in people's homes.

Passage 1: Montaigne’s pursuit of the character he called Myself—“bashful, insolent; chaste, lustful; prating, silent; laborious,
delicate; knowing, ignorant”—lasted for twenty years and produced more than a thousands pages of observation and revision.
When he died, he was still revising and, apparently, not at all surprised, since Myself was a protean creature, impossible to
anticipate but also, being always at hand, impossible to ignore. I like to think of the essays as a kind of thriller; with Myself, the
elusive prey, and Montaigne, the sleuth, locked in a battle of equals who were too close for dissimulation and too smart for
satisfaction. And it may be that Montaigne did, too, because he often warned his readers that nothing he wrote about myself was
likely to apply for much longer than it took the ink he used, writing it, to dry.

1. The words “prating” and “laborious,” in context, most likely mean what respectively?

(A) bold and relaxed


(B) rough and harsh
(C) awkward and talkative
(D) garrulous and quiet
(E) chatty and ponderous

2. In context, the word “protean” most nearly means:

(A) unfortunate
(B) threatening
(C) unwavering
(D) constantly changing
(E) difficult to describe

3. As used in the passage, the word “dissimulation” connotes a sense of:

(A) deliberate fraud


(B) outright audacity
(C) hidden deception
(D) unfeigned delight
(E) implied criticism

Directions: Here are three vocabulary-in-context questions. As you read each partial passage, use the context to help you determine
the correct answer.

The British barons who signed the Magna Carta irrevocably committed themselves to supporting the idea of limited monarchy.

1. In line 2, "committed" most likely means

a. confined
b. withheld
c. pledged
d. released from
e. performed

Impressionist paintings are now among the most popular words of art in the world. However, when they first appeared, art critics
denounced the paintings, showing no appreciation for the Impressionist's bold new techniques.

2. In line 3, "appreciation" most nearly means

a. increase in value
b. understanding
c. scientific interest

PEV 107 Page 92


d. anxiety about
e. gratitude for

I am supposing, or perhaps hoping, that our future may be sound in the past's fugitive moments of compassion rather than in its solid
centuries of warfare.

3. In line 3, "solid" most nearly means

a. unbroken
b. rugged
c. firm
d. reliable
e. unanimous

Passage 2: The victory of the small Greek democracy of Athens over the mighty Persian empire in 490 B. C. is one of the most
famous events in history. Darius, king of the Persian empire, was furious because Athens had interceded for the other Greek city-
states in revolt against Persian domination. In anger the king sent an enormous army to defeat Athens. He thought it would take
drastic steps to pacify the rebellious part of the empire. Persia was ruled by one man. In Athens, however, all citizens helped to rule.
Ennobled by this participation, Athenians were prepared to die for their city-state. Perhaps this was the secret of the remarkable
victory at Marathon, which freed them from Persian rule. On their way to Marathon, the Persians tried to fool some Greek city-states
by claiming to have come in peace. The frightened citizens of Delos refused to believe this. Not wanting to abet the conquest of
Greece, they fled from their city and did not return until the Persians had left. They were wise, for the Persians next conquered the
city of Etria and captured its people. Tiny Athens stood alone against Persia. The Athenian people went to their sanctuaries. There
they prayed for deliverance. They asked their gods to expedite their victory. The Athenians refurbished their weapons and moved to
the plain of Marathon, where their little band would meet the Persians. At the last moment, soldiers from Plataea reinforced the
Athenian troops. The Athenian army attacked, and Greek citizens fought bravely. The power of the mighty Persians was offset by the
love that the Athenians had for their city. Athenians defeated the Persians in archery and hand combat. Greek soldiers seized Persian
ships and burned them, and the Persians fled in terror. Herodotus, a famous historian, reports that 6400 Persians died, compared
with only 192 Athenians.

1. Athens had ____the other Greek city-states against the Persians.

A. refused help to B. intervened on behalf of C. wanted to fight D. given orders for all to fight

2. Darius took drastic steps to ___ the rebellious Athenians.

A. weaken B. destroy C. calm D. placate

3. Their participation___to the Athenians.

A. gave comfort B. gave honor C. gave strength D. gave fear

4. The people of Delos did not want to ___ the conquest of Greece.

A. end B. encourage C. think about D. daydream about

5. The Athenians were ___by some soldiers who arrived from Plataea.

A. welcomed B. strengthened C. held D. captured

Passage 3: Many great inventions are greeted with ridicule and disbelief. The invention of the airplane was no exception. Although
many people who heard about the first powered flight on December 17,1903, were excited and impressed, others reacted with peals
of laughter. The idea of flying an aircraft was repulsive to some people. Such people called Wilbur and Orville Wright, the inventors of
the first flying machine, impulsive fools. Negative reactions, however, did not stop the Wrights. Impelled by their desire to succeed,
they continued their experiments in aviation. Orville and Wilbur Wright had always had a compelling interest in aeronautics and
mechanics. As young boys they earned money by making and selling kites and mechanical toys. Later, they designed a newspaper-
folding machine, built a printing press, and operated a bicycle-repair shop. In 1896, when they read about the death of Otto Lilienthal,

PEV 107 Page 93


the brother's interest in flight grew into a compulsion. Lilienthal, a pioneer in hang-gliding, had controlled his gliders by shifting his
body in the desired direction. This idea was repellent to the Wright brothers, however, and they searched for more efficient methods
to control the balance of airborne vehicles. In 1900 and 1901, the Wrights tested numerous gliders and developed control techniques.
The brothers' inability to obtain enough lift power for the gliders almost led them to abandon their efforts. After further study, the
Wright brothers concluded that the published tables of air pressure on curved surfaces must be wrong. They set up a wind tunnel and
began a series of experiments with model wings. Because of their efforts, the old tables were repealed in time and replaced by the
first reliable figures for air pressure on curved surfaces. This work, in turn, made it possible for them to design a machine that would
fly. In 1903 the Wrights built their first airplane, which cost less than one thousand dollars. They even designed and built their own
source of propulsiona lightweight gasoline engine. When they started the engine on December 17, the airplane pulsated wildly before
taking off. The plane managed to stay aloft for twelve seconds, however, and it flew one hundred twenty feet. By 1905 the Wrights
had perfected the first airplane that could turn, circle, and remain airborne for half an hour at a time. Others had flown in balloons or
in hang gliders, but the Wright brothers were the first to build a full-size machine that could fly under its own power. As the
contributors of one of the most outstanding engineering achievements in history, the Wright brothers are accurately called the
fathers of aviation.

1. The idea of flying an aircraft was ___to some people.

A. boring B. distasteful C. exciting D. needless

2. People thought that the Wright brothers had ____.

A. acted without thinking B. been negatively influenced

C. been too cautious D. had not given enough thought

3. The Wright's interest in flight grew into a ____.

A. financial empire B. plan C. need to act D. foolish thought

4. Lilenthal's idea about controlling airborne vehicles was ___the Wrights.

A. proven wrong by B. opposite to the ideas of C. disliked by D. accepted by

5. The old tables were __ and replaced by the first reliable figures for air pressure on curved surfaces.

A. destroyed B. canceled C. multiplied D. discarded

6. The Wrights designed and built their own source of ____.

A. force for moving forward B. force for turning around

C. turning D. force to going backward

Passage 3: Marie Curie was one of the most accomplished scientists in history. Together with her husband, Pierre, she discovered
radium, an element widely used for treating cancer, and studied uranium and other radioactive substances. Pierre and Marie's
amicable collaboration later helped to unlock the secrets of the atom. Marie was born in 1867 in Warsaw, Poland, where her father
was a professor of physics. At the early age, she displayed a brilliant mind and a blithe personality. Her great exuberance for learning
prompted her to continue with her studies after high school. She became disgruntled, however, when she learned that the university
in Warsaw was closed to women. Determined to receive a higher education, she defiantly left Poland and in 1891 entered the
Sorbonne, a French university, where she earned her master's degree and doctorate in physics. Marie was fortunate to have studied
at the Sorbonne with some of the greatest scientists of her day, one of whom was Pierre Curie. Marie and Pierre were married in
1895 and spent many productive years working together in the physics laboratory. A short time after they discovered radium, Pierre
was killed by a horse-drawn wagon in 1906. Marie was stunned by this horrible misfortune and endured heartbreaking anguish.
Despondently she recalled their close relationship and the joy that they had shared in scientific research. The fact that she had two
young daughters to raise by herself greatly increased her distress. Curie's feeling of desolation finally began to fade when she was
asked to succeed her husband as a physics professor at the Sorbonne. She was the first woman to be given a professorship at the

PEV 107 Page 94


world-famous university. In 1911 she received the Nobel Prize in chemistry for isolating radium. Although Marie Curie eventually
suffered a fatal illness from her long exposure to radium, she never became disillusioned about her work. Regardless of the
consequences, she had dedicated herself to science and to revealing the mysteries of the physical world.

1. The Curies' ____ collaboration helped to unlock the secrets of the atom.

A. friendly B. competitive C. courteous D. industrious E. chemistry

2. Marie had a bright mind and a __personality.

A. strong B. lighthearted C. humorous D. strange E. envious

3. When she learned that she could not attend the university in Warsaw, she felt___.

A. hopeless B. annoyed C. depressed D. worried E. none of the above

4. Marie ___ by leaving Poland and traveling to France to enter the Sorbonne.

A. challenged authority B. showed intelligence C. behaved D. was distressed E. None

5. _____she remembered their joy together.

A. Dejectedly B. Worried C. Tearfully D. Happily E. Sorrowfully

6. Her ____ began to fade when she returned to the Sorbonne to succeed her husband.

A. misfortune B. anger C. wretchedness D. disappointment E. ambition

7. Even though she became fatally ill from working with radium, Marie Curie was never ____.

A. troubled B. worried C. disappointed D. sorrowful E. disturbed

Exercises : Inference, Application, Tone, Out of Context

DIRECTIONS for the question 1- 4 : Read the passage and answer the question based on it.
On August 3, 1492 , Christopher Columbus set sail from Palos , Spain , with less than a hundred crew members to discover a new
route to Asia. After spending a difficult time at sea, the party sighted land early on the morning ofOctober 12, 1492. They set foot on
an island in the Bahamas which they named Al Salvador. Columbus presumed that the indigenous people were Native Indians as he
was under the mistaken belief that he had set foot on Indian soil. Probably some 10 million American Indians were natives to the land
before the large-scale inhabitation by Europeans and subsequent annihilation of Native Americans started.
However, it took more than a hundred years after Columbus discovered America for the Europeans to finally take the momentous
decision to make the New World their home.
The Native Americans actually welcomed the pale-skinned visitors primarily out of curiosity than anything else. They were fascinated
by the steel knives and swords, fire spewing cannons, brass and copper utensils, etc. that these visitors brought with them.
Eventually, cultural differences erupted. The natives could not stomach the arrogance of the newcomers and the scant respect they
paid to nature. The European settlers viewed every resource — plants, animals, and people as something to be commercially
exploited.
The native Indians were vastly outnumbered in the wars that ensued. The resistance they put up never proved enough to stop the
European settlers. The nomadic lifestyle of the Indians, the relatively unsophisticated weapons at their disposal, the unwillingness of
some of their own people to defend themselves, and the diseases of the white men — all contributed to the virtual elimination of
their race. Some of the diseases brought by Europeans from their overcrowded cities that decimated the natives were: small pox,
plague, measles, cholera, typhoid, and malaria. These deadly diseases, to which most natives had developed no resistance,
devastated many tribes between 1775 and 1850.

PEV 107 Page 95


America was named after an Italian navigator, Amerigo Vespucci, who explored the Northern parts of South America in 1499 and
1500 and later announced to the world about the discovery of a new continent.

Question No. : 1 The primary purpose of the passage is to


A. Disprove the notion that America was named after Columbus
B. Provide a snapshot of the discovery of America and the early years of settlements.
C. Explain how the Europeans eliminated the native Americans in their own land
D. Discuss how the process of colonization of America started.
E. To tell about the Americans Indians.

Question No. : 2 From the passage we can infer that in comparison to the Europeans, Native Americans were
A. Careless about their environment
B. A very unhealthy lot
C. More respectful of nature
D. Ignorant about sanitation
E. Afraid of outsiders

Question No. : 3 What can be inferred from the third paragraph?


A. The native Americans did not have any weapons with which to defend themselves.
B. The native Americans probably attached a lot of importance to and respected nature.
C. The native Americans did not know how to use natural resources.
D. The early settlers became arrogant as they could commercially exploit resources.
E. The native Americans did not know the art of making utensils.

Question No. : 4 Based on the information in the passage which of the following cannot be inferred?
I. Alien diseases wiped out a large proportion of certain Native American tribes
II. The early settlers totally eliminated the Native Americans
III. To the early settlers, even people were resources to be exploited commercially
A. Only I
B. Only II
C. I & II
D. Only III
E. None of these

Passage 2: An increase in the median income of the middle class does not cause average levels of education for the middle
class to go up. If they did, then countries with the highest median income of the middle class would also have the highest levels
of education for this class. In fact, when the median income of the middle class is made suitably comparable for different
countries (accounting for inflation, currency fluctuations and purchasing power parity), there is no such co-relation.
Which of the following can be correctly inferred from the statements above?
a) Countries with the highest median income of the middle class do not have the highest levels of education for the middle class
b) It is difficult to reliably compare the education and income levels of one country with another
c) A reduction in the median income of the middle class of the country will not necessarily lower the average level of education for
the said class
d) Countries with low levels of education for the middle class have comparatively higher levels of median income for the middle
class

Passage 3: An increase in the level of serotonin levels in the human body is known to significantly enhance the mood of the
person and in some cases, help people overcome depression. Serotonin taken orally does not pass into the pathways of the
central nervous system, because it does not cross the blood–brain barrier. However, tryptophan and its metabolite 5-
hydroxytryptophan (5- HTP), from which serotonin is synthesized, does cross the blood–brain barrier. These agents are available
as dietary supplements, and may be effective serotonergic agents.
Which of the following can be correctly inferred from the statements above?

A. Individuals who do not consume enough tryptophan can develop depressive tendencies
B. Individuals who consume tryptophan can metabolize enough serotonin and do not run the risk of depression
C. Consumption of serotonin laced foods can help cure depression in some cases
D. Tryptophan is an effective agent that can help cure depression in some cases

PEV 107 Page 96


Passage 4: The need for knowledge of the constitution and functions, in the concrete, of human nature is great just because the
teacher's attitude to subject matter is so different from that of the pupil. The teacher presents in actuality what the pupil
represents only in posse. That is, the teacher already knows the things which the student is only learning. Hence the problem of
the two is radically unlike. When engaged in the direct act of teaching, the instructor needs to have subject matter at his fingers'
ends; his attention should be upon the attitude and response of the pupil. To understand the latter in its interplay with subject
matter is his task, while the pupil's mind, naturally, should be not on itself but on the topic in hand. Or to state the same point in
a somewhat different manner: the teacher should be occupied not with subject matter in itself but in its interaction with the
pupils' present needs and capacities. Hence simple scholarship is not enough.
It can be inferred from the paragraph that:

A. simple scholarship is lop-sided in its focus


B. simple scholarship might overlook the subject matter and focus purely on the student
C. simple scholarship, with a narrow focus, might not place enough emphasis on the student’s needs
D. simple scholarship will always be lopsided in nature because of its ambiguous stand on problems

Passage 5: All registered cyclists have passed an examination covering traffic rules and safety. Those interested in the new
BikeShare Program must register their bicycles and provide a major credit card to ensure payment. Call 555-1212 for more
information.
(A) argumentative (B) sentimental (C) idealistic (D) informative (E) caustic
Passage 6: On hot summer nights his family drove to the local ice cream stand, where, for just a few minutes, nothing mattered but
the cold, sweet treat. No dessert, before or since, was as delicious as a double-dip cone eaten while he balanced on a log behind the
parking lot.
(A) nostalgic (B) biting (C) irreverent (D) cynical (E) satirical
Passage 7: Iguacu Falls, which sit on the border between Argentina and Brazil, are said to make Niagara look like a leaky faucet. The
great cataracts stretch for two and a half miles across lushly foliaged rocky outcroppings before plunging a staggering two hundred
and thirty feet into the river below.The falls region is densely forested, and is home to a wide variety of plants and animals, including
a number of endangered ones. It is a paradise where parrots dive and swoop through the spray,butterflies cavort among the tropical
plants and coatis, and giant otters and anteaters amble through the trees. The foliage itself varies between tropical and deciduous
with orchids blushing in the shade of pines and ferns nodding gracefully in the shadow of fruit trees.

The author’s attitude toward Iguacu Falls can best be described as which of the following?

A. Overweening pride. B. Positive appreciation. C. Mild acceptance.

D. Apathetic objectivity. E. Cautious optimism.

Passage 8: Opera refers to a dramatic art form, originating in Europe, in which the emotional content is conveyed to the audience
as much through music, both vocal and instrumental, as it is through the lyrics. By contrast, in musical theater an actor's dramatic
performance is primary, and the music plays a lesser role. The drama in opera is presented using the primary elements of theater
such as scenery, costumes, and acting. However, the words of the opera, or libretto, are sung rather than spoken. The singers are
accompanied by a musical ensemble ranging from a small instrumental ensemble to a full symphonic orchestra.

1. It is pointed out in the reading that opera ----.

A)has developed under the influence of musical theater


B) is a drama sung with the accompaniment of an orchestra
C) is not a high-budget production
D) is often performed in Europe

PEV 107 Page 97


E) is the most complex of all the performing arts

2. We can understand from the reading that ----.

F) people are captivated more by opera than musical theater


G) drama in opera is more important than the music
H) orchestras in operas can vary considerably in size
I) musical theater relies above all on music
J) there is argument over whether the music is important or the words in opera

3. It is stated in the reading that ----.

K) acting and costumes are secondary to music in musical theater


L) many people find musical theater more captivating than opera
M) music in musical theater is not as important as it is in opera
N) an opera requires a huge orchestra as well as a large choir
O) opera doesn't have any properties in common with musical theater

Passage 9: Dolphins are regarded as the friendliest creatures in the sea and stories of them helping drowning sailors have been
common since Roman times. The more we learn about dolphins, the more we realize that their society is more complex than
people previously imagined. They look after other dolphins when they are ill, care for pregnant mothers and protect the weakest
in the community, as we do. Some scientists have suggested that dolphins have a language but it is much more probable that they
communicate with each other without needing words. Could any of these mammals be more intelligent than man? Certainly the
most common argument in favor of man's superiority over them that we can kill them more easily than they can kill us is the least
satisfactory. On the contrary, the more we discover about these remarkable creatures, the less we appear superior when we
destroy them.

4. It is clear from the passage that dolphins ----.

P) don't want to be with us as much as we want to be with them


Q) are proven to be less intelligent than once thought
R) have a reputation for being friendly to humans
S) are the most powerful creatures that live in the oceans
T) are capable of learning a language and communicating with humans

5. The fact that the writer of the passage thinks that we can kill dolphins more easily than they can kill us ----.

U) means that they are better adapted to their environment than we are
V) shows that dolphins have a very sophisticated form of communication
W) proves that dolphins are not the most intelligent species at sea
X) does not mean that we are superior to them
Y) proves that Dolphins have linguistic skills far beyond what we previously thought

6. One can infer from the reading that ----.

Z) dolphins are quite abundant in some areas of the world


AA) communication is the most fascinating aspect of the dolphins
BB) dolphins have skills that no other living creatures have such as the ability to think
CC) it is not usual for dolphins to communicate with each other
DD) dolphins have some social traits that are similar to those of humans

Exercises : Mixed

RC #1

In the United States the per capita costs of schooling have risen almost as fast as the cost of medical treatment. But increased
treatment by both doctors and teachers has shown steadily declining results. Medical expenses concentrated on those above forty-

PEV 107 Page 98


five have doubled several times over a period of forty years with a resulting 3 percent increase in the life expectancy of men. The
increase in educational expenditures has produced even stranger results; otherwise President Nixon could not have been moved this
spring to promise that every child shall soon have the “Right to Read” before leaving school.

In the United States it would take eighty billion dollars per year to provide what educators regard as equal treatment for all in
grammar and high school. This is well over twice the $36 billion now being spent. Independent cost projections prepared at HEW and
at the University of Florida indicate that by 1974 the comparable figures will be $107 billion as against the $45 billion now projected,
and these figures wholly omit the enormous costs of what is called “higher education,” for which demand is growing even faster. The
United States, which spent nearly eighty billion dollars in 1969 for “defense,” including its deployment in Vietnam, is obviously too
poor to provide equal schooling. The President’s committee for the study of school finance should ask not how to support or how to
trim such increasing costs, but how they can be avoided.

Equal obligatory schooling must be recognized as at least economically unfeasible. In Latin America the amount of public money
spent on each graduate student is between 350 and 1,500 times the amount spent on the median citizen (that is, the citizen who
holds the middle ground between the poorest and the richest). In the United States the discrepancy is smaller, but the discrimination
is keener. The richest parents, some 10 percent, can afford private education for their children and help them to benefit from
foundation grants. But in addition they obtain ten times the per capita amount of public funds if this is compared with the per capita
expenditure made on the children of the 10 percent who are poorest. The principal reasons for this are that rich children stay longer
in school, that a year in a university is disproportionately more expensive than a year in high school, and that most private universities
depend—at least indirectly—on tax- derived finances.

Obligatory schooling inevitably polarizes a society; it also grades the nations of the world according to an international caste system.
Countries are rated like castes whose educational dignity is determined by the average years of schooling of its citizens, a rating
which is closely related to per capita gross national product, and much more painful.

1. Which one of the following best expresses the main idea of the passage?
(A) The educational shortcomings of the United States, in contrast to those of Latin America, are merely the result of poor allocation
of available resources.
(B) Both education and medical care are severely underfunded.
(C) Defense spending is sapping funds which would be better spent in education.
(D) Obligatory schooling must be scrapped if the goal of educational equality is to be realized.
(E) Obligatory education does not and cannot provide equal education.

2. Consider each of the three choices and select all that apply.
The author most likely would agree with which one of the following solutions to the problems presented by obligatory education?
(A) Education should not be obligatory at all.
(B) Education should not be obligatory for those who cannot afford it.
(C) More money should be diverted to education for the poorest.

3. According to the passage, education is like health care in all of the following ways EXCEPT:
(A) It has reached a point of diminishing returns, increased spending no longer results in significant improvement.
(B) It has an inappropriate “more is better” philosophy.
(C) It is unfairly distributed between rich and poor.
(D) The amount of money being spent on older students is increasing.
(E) Its cost has increased nearly as fast.

4. Why does the author consider the results from increased educational expenditures to be “even stranger” than those from
increased medical expenditures?
(A) The aging of the population should have had an impact only on medical care, not on education.
(B) The “Right to Read” should be a bare minimum, not a Presidential ideal.
(C) Educational spending has shown even poorer results than spending on health care, despite greater increases.
(D) Education has become even more discriminatory than health care.
(E) It inevitably polarizes society.

PEV 107 Page 99


5. Which one of the following most accurately characterizes the author’s attitude with respect to obligatory schooling?
(A) qualified admiration
(B) critical
(C) neutral
(D) ambivalent
(E) resentful

RC #2

The premise with which the multiculturalists begin is unexceptional: that it is important to recognize and to celebrate the wide range
of cultures that exist in the United States. In what sounds like a reflection of traditional American pluralism, the multiculturalists
argue that we must recognize difference, that difference is legitimate; in its kindlier versions, multiculturalism represents the
discovery on the part of minority groups that they can play a part in molding the larger culture even as they are molded by it. And on
the campus multiculturalism, defined more locally as the need to recognize cultural variations among students, has tried with some
success to talk about how a racially and ethnically diverse student body can enrich everyone’s education.

Phillip Green, a political scientist at Smith and a thoughtful proponent of multiculturalism, notes that for a significant portion of the
students the politics of identity is all-consuming. Students he says “are unhappy with the thin gruel of rationalism. They require a
therapeutic curriculum to overcome not straightforward racism but ignorant stereotyping.”

But multiculturalism’s hard-liners, who seem to make up the majority of the movement, damn as racism any attempt to draw the
myriad of American groups into a common American culture. For these multiculturalists, differences are absolute, irreducible,
intractable—occasions not for understanding but for separation. The multiculturalist, it turns out, is not especially interested in the
great American hyphen, in the syncretistic (and therefore naturally tolerant) identities that allow Americans to belong to more than a
single culture, to be both particularists and universalists.

The time-honored American mixture of assimilation and traditional allegiance is denounced as a danger to racial and gender
authenticity. This is an extraordinary reversal of the traditional liberal commitment to a “truth” that transcends parochialisms. In the
new race/class/gender formation, universality is replaced by, among other things, feminist science Nubian numerals (as part of an
Afro- centric science), and what Marilyn Frankenstein of the University of Massachusetts-Boston describes as “ethno-mathematics,”
in which the cultural basis of counting comes to the fore.

The multiculturalists insist on seeing all perspectives as tainted by the perceiver’s particular point of view. Impartial knowledge, they
argue, is not possible, because ideas are simply the expression of individual identity, or of the unspoken but inescapable assumptions
that are inscribed in a culture or a language. The problem, however, with this warmed-over Nietzscheanism is that it threatens to
leave no ground for anybody to stand on. So the multi-culturalists make a leap, necessary for their own intellectual survival, and
proceed to argue that there are some categories, such as race and gender, that do in fact embody an unmistakable knowledge of
oppression. Victims are at least epistemologically lucky. Objectivity is a mask for oppression. And so an appalled former 1960s radical
complained to me that self- proclaimed witches were teaching classes on witchcraft. “They’re not teaching students how to think,”
she said, “they’re telling them what to believe.”

1. Which one of the following ideas would a multiculturalist NOT believe?


(A) That we should recognize and celebrate the differences among the many cultures in the United States.
(B) That we can never know the “truth” because “truth” is always shaped by one’s culture.
(C) That “difference” is more important than “sameness.”
(D) That a school curriculum should be constructed to compensate for institutionalized racism.
(E) That different cultures should work to assimilate themselves into the mainstream culture so that eventually there will be no
excuse for racism.

2. According to a hard-line multiculturalist, which one of the following groups is most likely to know the “truth” about political
reality?
(A) Educated people who have learned how to see reality from many different perspectives.
(B) A minority group that has suffered oppression at the hands of the majority.
(C) High government officials who have privileged access to secret information.

PEV 107 Page 100


(D) Minorities who through their education have risen above the socioeconomic position occupied by most members of their ethnic
group.
(E) Political scientists who have thoroughly studied the problem.

3. The author states that in a “kindlier version” of multiculturalism, minorities discover “that they can play a part in molding the
larger culture even as they are molded by it.” If no new ethnic groups were incorporated into the American culture for many
centuries to come, which one of the following would be the most probable outcome of this “kindlier version”?
(A) At some point in the future, there would be only one culture with no observable ethnic differences.
(B) Eventually the dominant culture would overwhelm the minority cultures, who would then lose their ethnic identities.
(C) The multiplicity of ethnic groups would remain but the characteristics of the different ethnic groups would change.
(D) The smaller ethnic groups would remain, and they would retain their ethnic heritage.
(E) The minority cultures would eventually overwhelm the dominant culture, which would then lose its identity.

4. The author speaks about the “politics of identity” that Phillip Green, a political scientist at Smith, notes is all-consuming for
many of the students. Considering the subject of the passage, which one of the following best describes what the author means by
“the politics of identity”?
(A) The attempt to discover individual identities through political action
(B) The political agenda that aspires to create a new pride of identity for Americans
(C) The current obsession for therapy groups that help individuals discover their inner selves
(D) The trend among minority students to discover their identities in their ethnic groups rather than in their individuality
(E) The increased political activism of minorities on college campuses

5. Which one of the following best describes the attitude of the writer toward the multicultural movement?
(A) Tolerant. It may have some faults, but it is well-meaning overall.
(B) Critical. A formerly admirable movement has been taken over by radical intellectuals.
(C) Disinterested. He seems to be presenting an objective report.
(D) Enthusiastic. The author embraces the multiculturalist movement and is trying to present it in a favorable light.
(E) Ambivalent. Like a moth to a flame he is simultaneously attracted and repulsed by the movement.

6. “Multiculturalist relativism” is the notion that there is no such thing as impartial or objective knowledge. The author seems to
be grounding his criticism of this notion on
(A) the clear evidence that science has indeed discovered “truths” that have been independent of both language and culture.
(B) the conclusion that relativism leaves one with no clear notions of any one thing that is true.
(C) the absurdity of claiming that knowledge of oppression is more valid than knowledge of scientific facts.
(D) the agreement among peoples of all cultures as to certain undeniable truths—e.g., when the sky is clear, day is warmer than
night.
(E) the fact that “truth” is not finitely definable and therefore that any discussion of impartial or objective truth is moot.

RC #3

According to usage and conventions which are at last being questioned but have by no means been overcome, the social presence of
a woman is different in kind from that of a man. A man’s presence is dependent upon the promise of power which he embodies. If
the promise is large and credible his presence is striking. If it is small or incredible, he is found to have little presence. The promised
power may be moral, physical, temperamental, economic, social, sexual—but its object is always exterior to the man. A man’s
presence suggests what he is capable of doing to you or for you. His presence may be fabricated, in the sense that he pretends to be
capable of what he is not. But the pretense is always toward a power which he exercises on others.

By contrast, a woman’s presence expresses her own attitude to herself, and defines what can and cannot be done to her. Her
presence is manifest in her gestures, voices, opinions, expressions, clothes, chosen surroundings, taste—indeed there is nothing she
can do which does not contribute to her presence. Presence for a woman is so intrinsic to her person that men tend to think of it as
an almost physical emanation, a kind of heat or smell or aura.

To be born a woman has been to be born, within an allotted and confined space, into the keeping of men. The social presence of
women has developed as a result of their ingenuity in living under such tutelage within such a limited space. But this has been at the

PEV 107 Page 101


cost of a woman’s self being split into two. A woman must continually watch herself. Whilst she is walking across a room or whilst
she is weeping at the death of her father, she can scarcely avoid envisaging herself walking or weeping. From earliest childhood she
has been taught and persuaded to survey herself continually.

And so she comes to consider the surveyor and the surveyed within her as the two constituent yet always distinct elements of her
identity as a woman.

She has to survey everything she is and everything she does because how she appears to others, and ultimately how she appears to
men, is of crucial importance for what is normally thought of as the success of her life. Her own sense of being in herself is
supplanted by a sense of being appreciated as herself by another. Men survey women before treating them. Consequently how a
woman appears to a man can determine how she will be treated. To acquire some control over this process, women must contain it
and internalize it. That part of a woman’s self which is the surveyor treats the part which is the surveyed so as to demonstrate to
others how her whole self would like to be treated. And this exemplary treatment of herself by herself constitutes her presence.
Every woman’s presence regulates what is and is not “permissible” within her presence. Every one of her actions—whatever its
direct purpose or motivation—is also read as an indication of how she would like to be treated. If a woman throws a glass on the
floor, this is an example of how she treats her own emotion of anger and so of how she would wish to be treated by others. If a man
does the same, his action is only read as an expression of his anger. If a woman makes a good joke this is an example of how she
treats the joker in herself and accordingly of how she as joker-woman would like to be treated by others. Only a man can make a
good joke for its own sake.

1. According to “usage and conventions,” appearance is NECESSARILY a part of reality for


(A) men
(B) women
(C) both men and women
(D) neither men nor women
(E) men always and women occasionally

2. In analyzing a woman’s customary “social presence,” the author hopes to


(A) justify and reinforce it.
(B) understand and explain it.
(C) expose and discredit it.
(D) demonstrate and criticize it.
(E) sanction and promote it.

3. It can be inferred from the passage that a woman with a Ph.D. in psychology who gives a lecture to a group of students is
probably MOST concerned with
(A) whether her students learn the material.
(B) what the males in the audience think of her.
(C) how she comes off as a speaker in psychology.
(D) finding a husband.
(E) whether a man challenges her.

4. The primary purpose of the passage is to


(A) compare and contrast woman’s presence and place in society with that of man’s.
(B) discuss a woman’s presence and place in society and to contrast it with a man’s presence and place.
(C) illustrate how a woman is oppressed by society.
(D) explain why men are better than women at telling jokes.
(E) illustrate how both men and women are hurt by sexism.

5. Consider each choice, and select all that apply.


Which of the following is implied by the passage?
(A) A man is defined by what he does, whereas a woman is defined by how she appears.
(B) Men are not image-conscious.
(C) Good looks are more important to women than to men.
PEV 107 Page 102
COVER LETTER
The purpose of a cover letter, which should always accompany your resume, is to introduce yourself and compel an employer to learn
more about you through your resume.

PERSONALIZE

The statement “To Whom it May Concern” is concerning! Always address your letter to a specific person. If you do not know who
should receive your letter, research the organization’s Web site or call the main number and ask for the appropriate person’s name
and title.
RESEARCH

In addition to determining the recipient’s name and title, research the organization so that you can convey an appreciation for its
mission and an understanding of the duties and qualifications of the position for which you are applying.
CONVINCE

Articulate how your skills and experiences uniquely qualify you for the position and demonstrate why you are a good fit for the
organization. Be clear and concise and limit your cover letter to one page. Additionally, maintain a professional tone while providing
insight into your personality so that your letter leaves an employer with a sense of wanting to learn more about you.
PROOFREAD

A poorly written or error-laden letter is a surefire way to end your candidacy. If you want an employer to spend additional time
reviewing your resume, you must make time to proofread your letter and ensure that your grammar and spelling are perfect.

COVER LETTER FORMAT


Date

Ms./Mr./Dr. recipient’s first and last name


Title
Organization name
Street address
City, state, zip code

Dear Ms./Mr./Dr. Last Name:

Paragraph 1: State why you are writing, how you learned of the organization or position, and basic information about yourself. If you
are writing at the suggestion of someone who knows the recipient, say so.

Paragraph 2: Demonstrate your knowledge and interest in the organization, and use specific examples to show how your background
and skills qualify you for the position.

Paragraph 3: This paragraph is optional and is recommended for elaborating on particularly relevant or impressive details included in
your resume.

Paragraph 4: Indicate that your resume and other supporting materials are enclosed. Reiterate your interest in the position and your
desire to meet for an interview. State your plans to follow up via e-mail or phone within a certain period of time and invite the
employer to contact you to request additional information. Lastly, thank the employer for his/her consideration.

Sincerely,

Your name

PEV 107 Page 103


SAMPLE 1: Computer science cover letter

October 4, 2017
Ms. Lori Hopkins
VP of Information Technology
ABC Corporation
245 Maple St.
Sometown, NJ 55555

Dear Ms. Hopkins:

I found your posting on Shine.com for a network administrator and I wanted to apply immediately. ABC Corporation’s reputation for
responsive service and proactive support differentiates your company from the rest, and I would like to be part of your dynamic team.

Since graduating from Lovely Professional University with a BS in computer science, I have worked for three years as a network
administrator for DEF Company. In this role, I am valued as a solutions-focused manager of networks, IT systems, user support
functions and technology projects. I have experience in all areas covered in your job ad, including:

 Network and Server Design and Administration


 LAN, WAN, VPN, SAN and VoIP Infrastructure
 Network Analysis and Optimization
 System Security, Disaster Recovery and Business Continuity Plans
 Cloud Computing and Data Storage
 Enterprise System and Software Implementations

Known as a keen technical trouble-shooter, I am dedicated to providing cost-effective and expedient solutions optimizing network
stability and security; improving performance of systems and equipment; and resolving issues such as network crashes, system
slowdowns and virus outbreaks.

At DEF Company, I provide world-class service technical support in an enterprise environment, and I am confident I would quickly
become a productive network administrator at ABC Corporation. If you agree, please call me at (555) 555-5555 to set up an interview.

I look forward to your response.

Sincerely,

Barbara Holt

SAMPLE 2 : Computer science cover letter


[5TH May,2018]
Ms. Elisa Thompson
XYZ Company
Hartford, CT 56789
Dear Ms. Thompson
When I discovered the software engineering internship with XYZ Company on shine.com, I was excited by the opportunity to develop
test cases, open bug reports, and triage test case failures. As a sophomore majoring in Computer Science at University College, I enjoy
developing and improving testing technologies. My coursework, employment, and campus involvement have prepared me well for this
position.

PEV 107 Page 104


 Coursework. After completing courses in software development methods, programming languages, network systems, and
embedded system design, my Major GPA is 3.8.
 IT Help Desk employment. As a student employee with IT services, I troubleshoot a wide variety of computing issues and
communicate technical information to non-technical customers.
 Campus involvement. As a member of the Association for Women in Computing, I attend Tech Talks and conferences to expand
my industry knowledge and network with full-time professionals.

I have experience with Java, Python, JavaScript, and SQL. This knowledge will be an asset to this position. I would be thrilled to learn
more alongside your cross-functional team of engineers and developers.
My enclosed resume expands on my coursework, employment, and campus involvement. As I prepare for a career in computer
science, I am dedicated to gaining practical experience to complement my coursework. Thank you for your time and consideration. I
look forward to hearing from you soon.
Sincerely,
Peyton Java

SAMPLE 3: ELECTRICAL ENGINEERING

[Application Date]
Mr. Justin Shaw
Robotics Startup
Chicago, IL 56789
Dear Mr. Shaw
When I discovered the electrical engineering internship with Robotics Startup on internships.com, I was excited by the opportunity to
develop electrical and electronic design for autonomous robots and mobility platforms. As a junior majoring in Electrical Engineering at
University College, I enjoy designing circuits and experimenting with new technologies. My academics, campus involvement, and
volunteer work have prepared me well for this position.

 Academics. After completing courses in Computer Aided Measurement and Controls, Circuit Analysis, Calculus, Physics, and
Electronics, my Major GPA is 3.9.
 Campus involvement. As a member of the Robotics Club, I have gained experience with hardware, data processing, sensors,
microcontroller programming, and communication protocols.
 Volunteer work. For the past two years, I have mentored a high school robotics team. In preparation for a Robotics Competition,
the students build and program an industrial robot.

I have experience creating circuit schematics and PCB layouts. These skills will be an asset to this position and I would be thrilled to
learn alongside your interdisciplinary team of innovators.
My enclosed resume expands on my academics, campus involvement, and volunteer work. As I prepare for a career in electrical
engineering, I am dedicated to gaining practical experience to complement my coursework. Thank you for your time and
consideration. I look forward to hearing from you soon.
Sincerely,
Adrian Ware

SAMPLE 4: COVER LETTER FOR MECHANICAL ENGINEERING BRANCH

[Date]

[Name of HR Manager]
[Position Title]

PEV 107 Page 105


[Company Name]
[Company Address]
[City, State, Zip]

Dear Mr./Ms. [...]:

Please accept this letter and the accompanying resume as an expression of my interest in a position with your organization.

As my resume indicates, in June 2006, I expect to receive a Bachelor of Science degree in mechanical engineering from California
Polytechnic State University. I would then like to begin employment with an organization in which which my effective performance will
be met with recognition and growth.

I have acquired a sound overall knowledge of leading edge engineering principles, tools, and practices, with emphasis on designing,
building and testing of mechanical systems. I am proficient in the use of various automated solutions including current releases of
AutoCAD and SolidWorks.

I have applied classroom learning to innovative and successful projects in which I served as sole or principal designer. Additionally, I
have been a responsible leader in a family owned small business.

My personal attributes include leadership and sound judgment as well as creativity, analytical and troubleshooting skills. I interact
productively with people from diverse backgrounds. I have a career history of achieving employment goals. I have a history of quality
work carried to timely completion.

I am certain I could make significant contributions to your organization, and I would welcome the opportunity to meet with you to
discuss how my education and abilities might best be employed by your organization.

Thank you for your consideration. I look forward to your response.

Sincerely,

Mark Gunlogson

SAMPLE LETTER 5: MECHANICAL ENGINEERING


[5TH May,2018]
Ms. Elisa Thompson
XYZ Company
Hartford, CT 56789
Dear Ms. Thompson

Please accept this letter and the accompanying resume as an expression of my interest for the Mechanical Engineer position offered by
your organization.

I Graduated as a masters in mechanical engineering from university of Alabama in Huntsville. As a mechanical engineer, I have a strong
background in the basic principles of engineering, methods and practices in mechanical system design, testing and troubleshooting
mechanical equipment. In addition, I am proficient in using FEA software’s & advanced CAD designing tools.

PEV 107 Page 106


My professional work experience as graduate trainee engineer at manufacturing industry and my years of academic background have
given me not only a strong technical foundation but also domain knowledge and exposure to Work environment. My key strengths
include effective communication, strong troubleshooting skills, Quick problem solving ability and passionate to learn and adopt new
technologies and skills.

As my years of education and experience make me feel confident that I can excel and live up to the expectations in performing the
duties assigned. My enclosed resume provides more details on my qualifications, skills and work experience.

I would welcome the opportunity to meet with you to discuss how my education and experience might best be employed to your
organization. Please feel free to contact me if you have any further questions. I will be available as soon as possible to take up the
responsibilities if you find me as an potential fit for this position.

Sincerely,

Mark Federick

SAMPLE 6: CIVIL ENGINEERING

November 8, 2017
Ms. Diane Smith
HR Manager
ABC Company
55 Circle Point
Sometown, AZ 55555

Dear Ms. Smith:

Your civil engineer trainee position is an exciting opportunity. ABC Company’s stellar reputation for providing quality service and
project support to your clients is well-known, and I am motivated to join your team.

Highlights of my credentials:

 BS in civil engineering and Engineer in Training (EIT) certification from NCEES.


 Two years of experience as a civil engineer trainee and intern for premier design firms.
 Hands-on experience providing project design and site civil engineering leadership for water, wastewater, roadway and other
municipal infrastructure projects in Sometown, AZ.
 Delivery of on-schedule, on-budget and high-quality completion of infrastructure projects with varying degrees of complexity
and budgets ranging from $500K to $6.8M.
 Proficient in preparing drawings and technical specs using AutoCAD Civil 3D and MicroStation.

In addition, I have worked closely with licensed PEs to save our clients and employers tens of thousands of dollars while maintaining
compliance with regulatory requirements. I have provided creative, cost-effective design solutions to issues including erosion,
corrosive groundwater, insufficient water inflow/drainage, soil instability, steep-slope grading and other engineering challenges.

If my credentials and strong understanding of municipal water and wastewater infrastructure systems are well matched to your needs,
please contact me at (555) 555-5555 or email lm@somedomain.com to schedule an interview. Thanks very much.

Sincerely,

PEV 107 Page 107


Lena Marks, EIT

SAMPLE : CIVIL ENGINEERING

Date: 22 October 2010

Mr Philip Smith
Hiring Manager
Richmond Engineering Services
London
E12 5DD

Dear Mr Smith

I wish to apply for the post of Civil Engineer as advertised on the Dayjob.com web site. The position seems an ideal opportunity for me
as it matches my experience, knowledge and career aspirations.

As you can see from my CV, I have amassed over 7 years of significant, progressive experience in civil engineering projects within the
construction and gas industries. During my career I have participated in more than 20 civil engineering projects, all of them were
successful.

My progressive engineering experience has provided me with the opportunity to develop strong client relation building skills and an
ability to lead multi-disciplinary teams. I am experienced at applying the principles of civil engineering when planning, designing, and
overseeing the construction and maintenance of structures and facilities.

I enclose my resume for your review and look forward to meeting with you in the near future. Should you have any questions or
require clarification on any information please contact me at the above telephone number.

I would welcome the opportunity to talk to you and I hope that you will invite me for a interview. I thank you for your time and I look
forward to hearing from you.

Your sincerely

Richard Daniels
444 Hanley Road
Birmingham
B18 6NF
Mobile: 0121 638 0026
Email: richad.d@dayjob.com

COVER LETTER FOR SCIENCE GRADUATES

[date]

Mr. Rover Turlin


HR Manager
Nestle Ghana Limited
Tema, Greater Accra

Dear Mr. Turlin

Please accept my application for the position of Food Scientist as advertised in the Daily Graphic.

PEV 107 Page 108


I am a first degree holder in Agricultural Science from the Kwame Nkrumah University of Science and Technology. During my National
Service at Accra Brewery, I worked as an Assistant Food Production Scientist. My job experience is in the area of molecular
manufacturing technology and the use of nanotechnology in food sensor development.

I possess top research skills with the ability to liaise effectively in a team.

My attention to detail and excellent problem-solving skills ensure that every project I work on is done accurately and to the highest
possible standard. My verbal communication skills and positive attitude make me an effective communicator with all clients I work
with.

I look forward to the opportunity of discussing my application with you further and how I can significantly contribute to the ongoing
success of your company.

Yours sincerely

Michael Lemodo

SAMPLE LETTER FOR SCIENCE:

Date

Ms. Joy Franklin


California State University
1442 Dark Hollow Road
Camden, NJ 8102

Dear Ms. Joy Franklin,

Highly motivated individual seeking employment in the Agriculture industry where I can put my skills and training to good use by
helping to solve some of the problems the world faces today.

I have a real desire to learn more about wildlife and a concern for the health and well-being of the different species located all over the
globe. The animals and plants play a major role in our future. It is my hope to help find ways for humans and wildlife to exist together
without major consequences by reducing the impact humans have on the animals as our habitats and technology continues to grow.

I earned a bachelor’s degree in wildlife management from California State University and completed an internship to gain hands-on
experience in this profession. My training focused on conservation, environmental management and wildlife ecology and
management.

I have the ability to conduct research to identify problems associated with the fisheries and wildlife and to find suitable solutions for
these problems. My abilities include excellent verbal and written skills that help me to convey the need for concern and to explain why
changes are needed to protect our future. I also have the ability to compile reports that show the results of my research, along with
possible solutions that can help solve some of these problems.

Other skills that make me suitable for a position in this industry includes having lots of patience along with excellent time management
and multitasking skills and a real desire to make a difference.
PEV 107 Page 109
Please call (111)-494-6970 if you have a position available.

Respectfully,

Signature

William Mitchell

Tutorial
Exercises:

QUESTIONS FOR COMPUTER SCIENCE

1. Write a cover letter for applying to a job in Tata Consultancy service, considering yourself as a fresher.
2. Write a cover letter for the post of Software Engineer, giving the details of the past experience and achievements.
3. Write a cover letter for the post of Data Analyst in Adobe in Banglore. Inform Ms. Lori Hopkins ,VP of Information
Technology ,Adobe,245 Maple St.,Banglore.
4. Write a cover letter for the post of Hardware Engineer, giving the details of the past experience and achievements.
5. Write a cover letter to the Robotics Pvt Ltd ,to apply for the internship in the Artificial intelligence and neurological
networks. Also, mention your achievements and subjects of interest.

QUESTIONS FOR MECHANICAL ENGINEERING

1. Write a cover letter for the post of Design Engineer in XYZ Pvt Ltd, Bangalore. Write the letter to Mr.PR Rattan who is the
Manager of the company . Also, mention about the software you have worked on for designing.
2. Write a cover letter to the Robotics Pvt Ltd ,to apply for the internship in the field of Industrial engineering and testing. Also,
mention about the project which you were handling for testing the machines and various projects.
3. Write a cover letter to Ms. Elisa Thompson from XYZ Company at Hartford, regarding the vacancy for Automobile engineer.
Also, mention about your research work , achievements and projects done till yet.

QUESTIONS FOR CIVIL ENGINEERING

1. Write a cover letter to Ms. Anna Thompson from XYZ Company at Hartford, regarding the vacancy for civil engineer. Also,
mention about your research work , achievements and projects done till yet.

2. Write a cover letter for the post of Design Engineer in XYZ Pvt Ltd, Bangalore. Write the letter to Mr.PR Rattan who is the
Manager of the company . Also, mention about the softwares and tools you have knowledge and use for designing.

3. Write a cover letter for applying to a job in Anand Construction company, considering yourself as a fresher.

PEV 107 Page 110


QUESTIONS FOR ELECTRONICS ENGINEERING.

1. Write a cover letter for the post of Junior embedded software engineer in the ABC Pvt ltd, Pune. Write about the experience,
achievements you have attained and also about the projects done till yet.

2. Write a cover letter to the Robotics Pvt Ltd ,to apply for the internship in the Artificial intelligence and neurological
networks. Also, mention your achievements and subjects of interest.

3. Write a cover letter to HBL Power Systems Limited, for the post of electronic engineer. Write this letter as a fresher and do
mention the projects you did in your bachelor degree.

QUESTIONS FOR SCIENCES

1. Write a cover letter for the post of Farm manager in the Imagine People Solutions Pvt Ltd in Gwalior. Mention your
experience ,projects and internship.

2. Write a cover letter for the post of Soil Scientist in the Reeracoen India Pvt Ltd, Bangalore. Write this letter as a fresher and
do mention about the projects and research papers.

3. Write to the Dharitri Rural Agrotech Pvt Ltd for the post of Plant breeder . Write your previous experience , reason behind
leaving the previous industry.

PEV 107 Page 111


Para Jumbles
What are Para jumbles?

Para jumbles are jumbled paragraphs. Basically, you are given a paragraph - but the sentences are not in the right order. It's up to
you to untie this knot and rearrange the sentences so that they logically make sense.

Types of Para jumbles

1. In some Para jumbles, the candidates are given the introductory or opening sentence of the Para jumble and they’re required to
un-jumble the remaining sentences.

2. In some, the closing sentence is provided and the candidates are required to use this to rearrange the remaining sentences.

3. In some both the opening and closing (concluding) sentences are given. These are the easiest Para jumbles to solve.

4. In most cases, neither the opening nor the closing sentences are given. The candidate has to figure that out on his/her own. These
are the most challenging Para jumbles to solve.

How to approach Para jumbles?


Step 1: Read the Para jumble
Give the Para jumble a quick scan to get a ‘feel’ of what the passage is about. Find out the central theme of the Para jumble.
Understanding the central theme/dialogue helps determine the flow of the story which goes a long way in helping you piece the
paragraph together in the right order.

Step 2: Look for the opening sentence


In a Paragraph, the opening sentence usually introduces a person/place/concept/premise/committee and establishes a scene.
Whereas a closing sentence is the one that has a conclusion. Closing sentences/conclusions start with words like ‘therefore’, ‘thus’,
‘hence’, etc. and contain advises/recommendations/suggestions/summaries.
It’s a good idea to find both opening and closing sentences. Once these two are determined, finding the other sentences becomes
easy.
Example:
a. She was waiting for her train
b. Geeta was at the railway station
c. She was going back home on a vacation

In the above example, clearly, the second option (b) is the introductory/opening sentence. The passage talks about train, railway
station, vacation etc. The first sentence must essentially introduce the person who’s catching the train to go on a vacation.
Sentences ‘a’ and ‘c’ refer to the person as ‘she’, which is an indication that these aren’t the opening sentences. Therefore, option
‘b’ is the opening sentence; it introduces the person ‘Geeta’ and the place ‘Railway station’.
Hence, the right order of the above example would be: b, a, c.

Step 3: Weave the sentences together


Once you’re done finding the opening (introductory) and closing (concluding) sentences, start weaving the other sentences together
based on factors like activities/time/chronology/any other sequence.

Activities: Some Para jumbles talk about activities. Determining the order of the activities will help you solve the question. Start by
finding out the initial activity. This will enable you to unfold a sequence that follows the flow of the story, thereby helping you put
the sentences in the right sequence.

Example:
a. She sells cupcakes throughout the day and returns home at 6 in the evening.
b. She has her dinner at 8 and goes to sleep at 11.
c. Maria has a cupcake business.
d. She makes the cupcakes at home in the morning.
Central theme: Activities of a person who owns a cupcake business.

PEV 107 Page 112


Activity tracking:
Sentence ‘c’ is clearly the opening sentence, since it introduces the person ‘Maria’ and her ‘Cupcake business’.
The Parajumble talks about her routine, i.e. from morning to evening. Therefore, sentence ‘d’ comes second as it talks about her
morning activity (‘in the morning’ is an indicator).
This is followed by sentence ‘a’, which talks about her activity throughout the day.
The concluding sentence is sentence ‘b’. It talks about Marias activities after coming back home (‘dinner’ and ‘goes to sleep at 11’
are indicators of a conclusion).
Therefore, the right order of the Parajumble is: c, d, a, b.

Abbreviations and full-forms: Some Para jumbles contain a sentence with a full-form and other sentence(s) with its abbreviations.
Here the sentence with the full-form comes first, followed by the sentence with the abbreviations. Abbreviations are always
introduced with its full-form in preceding sentences.

Example:
a. The CPU carries out the instructions given by the computer program.
b. The Central Processing Unit (CPU) is an important part of a computer.
c. CPUs are considered the heart and brains of a computer.

In the above example, sentence ‘a’ and ‘c’ contain the abbreviations ‘CPU’ and sentence ‘b’ contains the full-form of CPU. Therefore,
sentence ‘b’ comes before sentence ‘a’ and ‘c’. Right order: b, a, c.

Ideas and examples: Ideas always precede the examples. Some Para jumbles contain ideas, and examples of those ideas. The
examples always follow the ideas.

Example:
a. Like, the continental shelf, continental slope, abyssal plain and oceanic trenches.
b. The Ocean floor is divided into many parts.

In the above example, sentence ‘a’ contains examples and sentence ‘b’ introduces an idea. Hence sentence ‘b’ comes before
sentence ‘a’.

Connectives and Transition words: Connectives and transition words are logical connectors of different sentences. Connectives are
words that connect two sentences together. Some examples of connectives are: After, When, Because, Alternatively, Although,
Though, Yet, Until, Since, Etc.
Transition words, are words used by the author to shift one idea in a sentence to another (ushering a change). Some examples of
transition words are: However, Besides, Nevertheless, etc.
Sentences that start with a connective or a transition word are almost never introductory sentences. They always refer to
activities/events/people mentioned/introduced in the preceding sentences.

Articles: The articles ‘The’, ‘A’ and ‘An’ too help in finding out the order of the sentences. ‘The’ is a definite article which is used
before something/someone specific or when something/someone has already been introduced in the previous sentences.
Whereas, ‘A’ and ‘An’ articles are used when something is being introduced for the first time and are also used when stating general
facts. A sentence containing ‘A’ and ‘An’ could be an introductory sentence.

Example:
a. The girl had unusually long hair.
b. There was a girl, living in a tall tower.
c. And the tower was the tallest in town and hidden behind a dense foliage.

In the above example, article ‘A’ is used to introduce the girl and the tower and article ‘The’ is used while mentioning the girl and the
tower in options ‘a’ and ‘c’ respectively. Hence, option ‘b’ comes before opening ‘a’ and option ‘c’. Option ‘c’ contains the
connective ‘And’ which is used to connect two sentences, hence ‘c’ comes after ‘a’. Right order: b, a, c.

Pronouns: Pronouns (He, she, it, him, her, their, etc.) are used in place of a person/place/thing that has already been introduced in
one of the preceding sentences. So, if you find a pronoun in a sentence, it probably isn’t an opening sentence.

PEV 107 Page 113


Example:
a. Ajay is a good singer.
b. He has learnt vocal music for 12 years.

In the above example, option ‘b’ contains the pronoun ‘he’. Whoever ‘he’ is, should essentially be introduced in one of the preceding
sentences. Option ‘a’ introduces a person ‘Ajay’, hence, ‘he’ mentioned in option ‘b’ refers to ‘Ajay’. Option ‘a’ comes before option
‘b’.

Adjectives: Sometimes, adjectives can help solve Para jumbles too, especially comparative adjectives like better, worse, taller,
shorter, etc.
Example:
a. Rahul’s performance was good.
b. Rahul and Ashok danced on the same song.
c. Ashok’s performance was better.

In the above example, the adjectives ‘good’ and ‘better’ are used. ‘Better’ always comes after good (Good -> Better -> Best), hence,
the option containing ‘good’ comes before the option containing ‘better. Right order: b, a, c.

Time sequence approach: Sometimes Para jumbles contain a time sequence, i.e., words indicating a time sequence, such as, dates,
years, or words like, before, later, after, when, etc. Notice these words and be aware of them, as they can help rearrange the
sentences according to the right time sequence.

Example:
a. Hemant eats his dinner at 8pm sharp.
b. Post-homework, he is allowed to watch TV for half an hour.
c. After that, he does his homework.

In the above example, the words ‘after’ and ‘post’ denote time sequence. Hence, options ‘c’ and ‘b’ come after option ‘a’. Right
order: a, c, b.

Elimination technique: If you’re running out of time, you can use the elimination technique to arrive at the right option quickly.
After finding the opening and closing sentences, you can eliminate options which contain the wrong order of the opening and
closing sentences.

Example:
a. He sells newspapers in the morning.
b. Ramesh is a hardworking person.
c. He takes tuition for primary school children in the evening.
d. He then goes to work as a personal assistant to a businessman.

In the above examples, option ‘b’ is the opening sentence and option ‘c’ is the closing sentence. Therefore, any option that doesn’t
contain option ‘b’ in the beginning and option ‘c’ in the end can be eliminated.

Conclusions: Conclusions generally start with words like, thus, therefore, hence, in conclusion, etc. and are
usually advises/summaries/recommendations/suggestions. Finding the opening and closing sentences makes it easier to connect
them with the other options.

Step 4: Re-read the sentence to ensure continuity


Once you have weaved the sentences in the right order, re-read the passage to make sure that it makes sense grammatically and
logically. The right order should have a continuity in the flow of the sentences, and should also make the sentence meaningful.

PEV 107 Page 114


Tutorial
Exercises : Beginner

Q1. Rearrange the following sentences into a coherent paragraph:


1. 1971 war changed the political geography of the subcontinent
2. Despite the significance of the event. There has been no serious book about the conflict
3. Surrender at Dacca aims to fill this gap
4. It also profoundly altered the geo-strategic situation in South-East Asia

A. 1324 B. 3142 C. 2143 D. 1423

Q2. Rearrange the following sentences into a coherent paragraph:

1. Nonetheless, Tocqueville was only one of the first of a long line of thinkers to worry whether such rough equality could survive in the
face of a growing factory system that threatened to create divisions between industrial workers and a new business elite.
2. "The government of democracy brings the nation of political rights to the level of the humblest citizens. He wrote ," Just as the
dissemination of wealth brings the notion of property within the reach of all the members of the community".
3. Tocqueville was far too shrewd an observer to be uncritical about the US, but his verdict was fundamentally positive.
4. No visitor to the US left a more enduring record of his travels and observations than the French writer and political theorist Alexis de
Tocqueville, whose ‘Democracy in America’, first published in 1835, remains one of the most trenchant and insightful analyses of
American social and political practises.

A. 1324 B. 4321 C. 2143 D. 1423

Q3. Rearrange the following sentences into a coherent paragraph:

1). The potential exchanges between the officials of IBBF and the Maharashtra Body-Building Association has all the trappings of a
drama we are accustomed to.

2). In the case of sports persons, there is room for some sympathy, but the apathy of the administrators, which has even led to
sanctions from international bodies, is unpardonable.

3). A case in the point is the hefty penalty of US $10,000 slapped on the Indian Body-Building Federation for not fulfilling its
commitment for holding the Asian Championships in Mumbai in October.

4). It is a matter of deep regret and concern that the sports administrators often cause more harm to the image of the country than
sportsmen and sportswomen do through their dismal performances.
A. 1324 B. 4231 C. 2413 D. 3421

Q4. Rearrange the following sentences into a coherent paragraph:


1). Over the years, I have had the opportunities to observe and understand the thought processes behind the ads that have been
flooding both the print and the TV media.
2). Although there is a huge shift in the quality of ads that we come across on a daily basis-- thanks essentially to improvement in
technology--I somehow can't help but feel that the quality of communication of the message has become diluted.
3). Proportionally, the number of ads that lack in quality, have gone up exponentially as well!!
4). There is an increasing attempt by most companies to be seen as cool and funky.
5). Another reason could be the burgeoning number of companies, which means an exponential increase in the number of ads that
are being made.
A. 12453 B. 45231 C. 24135 D. 35421

PEV 107 Page 115


Q5. Rearrange the following sentences into a coherent paragraph:
1). His political career came to an abrupt end with China's military operation.
2). He attracted as as repelled.
3). He was responsible for the debacle.
4). A man of paradoxes, Menon remained an enigma.
A. 1324 B. 3142 C. 2143 D. 4213

Q6. Rearrange the following sentences into a coherent paragraph:


1). Thus begins the search for relief: painkillers, ice, yoga, herbs, even surgery
2). Most computer users develop disorders because they ignore warnings like tingling fingers, a numb hand or a sore shoulder
3). They keep pointing and dragging until tendons chafe and scar tissue forms, along with bad habits that are almost impossible to
change
4). But cures are elusive , because repetitive stree injuries present a bag of ills that often defy easy diagnosis.

A. 2413 B. 2143 C. 2314 D. 1234

Q7. Rearrange the following sentences into a coherent paragraph:


1). Let us take a look at the manner in which the traditional bank adds value to the customer.
2). The ability to retain deposits, in itself, is not enough to ensure long-term survival and growth.
3). The ability to deploy invested funds into productive economic activity at a higher rate of return, hence contributing to the
prosperity of both the economy and the institution, is the other loop in the banking cycle.
4). Further, as only a small portion of the actual deposit base is retained with the bank in a liquid form, the very survival of the bank
lies in building enough trust with its clientele so as to prevent the occurrence of a sizeable chunk of simultaneous customer
withdrawal (a run on the bank).
5). The bank's basic job is risk absorption- it takes money, which has a lot of attached risk, and provides the customer an assured
rate of return.
A. 12453 B. 45231 C. 15423 D. 35421

Q8. Rearrange the following sentences into a coherent paragraph:


1). What came out was very large garland made out of currency notes.
2). The unsuspecting governor opened the box in full view of the gathering
3). When the RBI governor came to inaugurate the new printing press, the local unit of the BJP handed him a gift wrapped box
4). There was a twist – the notes were all as tattered as notes could get

A. 2413 B. 3124 C. 2314 D. 3214

Q9. Rearrange the following sentences into a coherent paragraph:


1). Otherwise the Congress would not have opposed PSU disinvestment today.
2). It is clear that there is not consensus on economic reform.
3). Nor would allies of ruling NDA opposes privatisation.
4). All this would stop India from becoming the next superpower.

A. 1324 B. 4321 C. 2143 D. 2134

Q10. Rearrange the following sentences into a coherent paragraph:


1). I suggested that Ford should buy up a company called NCP, which owned most of the car parks in the city centres throughout the
UK.
2). We were discussing competing in the European market.
3). If NCP became a Ford company, a notice could be placed at the entrance to all city centres car parks indicating that only Ford cars
could use them
4). At one time I was giving a seminar for the British marketing department of Ford, the biggest Ford operation outside of Detroit.
A. 1324 B. 4213 C. 2413 D. 4321
PEV 107 Page 116
Exercises : Intermediate
Directions (Ques 1-5) : In these questions, each passage consists of six sentences. The first and the sixth sentence are given in the
beginning. The middle four sentences in each have been jumbled up. These are labelled P,Q,R and S. You are required to find out the
proper sequence of the four sentences from the given alternatives (a), (b), (c) and (d).

Ques 1. S1 : In 1945, America faced two powerful enemies in the world war.
S6 : This was the weapon that ended the second world war.
P : America found conventional weapons insufficient to crush them.
Q : These were Germany and Japan who posed strong opposition to America.
R : The result of this was the production of the atom bomb.
S : The government ordered the scientists to conduct research and produce a new deadly weapon.
(a) QPSR (b) PQRS (c) QPRS (d) PQSR

Ques 2. S1 : Advertising is also advantageous to the consumers, if it increases the sale of goods, industry prospers and prices may
be reduced.
S6 : Advertising of this particular kind is planned to stimulate new wants or to induce the buyers to change their habits.
P : There is no obvious connection, for example, between a picture of a smiling girl and a certain brand sweets.
Q : The advertiser’s assumption is that by looking at such pictures, the consumer would be influenced to buy his products.
R : On the other hand, much of the canvassing, of which the consumer is the object, does not convey information but endeavours
merely to draw the public attention to certain products.
S : But most people like looking at the pictures of pretty girls.
(a) RQSP (b) SPRQ (c) RPSQ (d) SQRP

Ques 3. S1 : We are living in an age in which technology has suddenly ‘annihilated distance.’
S6 : In that event, we should be dooming ourselves to wipe each other out.
P : We have never been so conscious of our variety as we are now that we have come to such close quarters.
Q : Physically, we are now all neighbours are, but psychologically, we are still the strangers to each other.
R : Are we going to let this consciousness of our variety make us fear and hate each other ?
S : How are we going to react ?
(a) QPSR (b) QPRS (c) PRQS (d) SRPQ

Ques 4. S1 : Mom was pleased to receive your wishes on her birthday.


S6 : Your Mom has forgotten all the bitterness and sends her blessings to you.
P: Girl ! wishes are more powerful than any other thing in the world.
Q: Both of us had forgotten the day.
R: Your letter holds a proof of it.
S: It was your letter and the card which reminded us of it.
(a) PSQR (b) QRSP (c) RQSP (d) QSPR

Ques 5. S1 : A century ago, the cinema was just a mechanical toy.


S6 : Finally, it has evolved as the century’s most potent and versatile art form.
P : Thus, it gained respectability and acceptance.
Q : It gradually came to be considered as an art form of the new era.
R : By the 1920s, even its worst critics had to take it seriously.
S : Later, it was viewed as an extension of photography.
(a) PQRS (b) SQRP (c) SPQR (d) QRSP

Directions (Ques 6-10) : Rearrange the following four sentences (A), (B), (C) and (D) in the proper sequence to form a meaningful
paragraph then mark the correct sequence as your answer.
Ques 6. A: It also gives rise to a feeling of animosity among the different sections of the society.
B: In a democratic system, frequent use of power is never desirable, be it on the part of government or the people.

PEV 107 Page 117


C: Therefore, citizens should never resort to violent ways and means in democracy, though they have the right to oppose
the government.
D: It destroys the stability and security in public life. The proper sequence should be
(a) DBAC (b) BDAC (c) BDCA (d) DACB

Ques 7. A: He was so busy with them that he did not get time to eat.
B: Thousands of people came to him and asked different types of questions.
C: No one cared to see that he had his food or rest that night.
D:Swami Vivekanand once stayed in a small village. The proper sequence should be
(a) BCDA (b) CBAD (c) DBAC (d) DBCA

Ques 8. A: The facts speak for themselves so, they need exposition only, not demonstration.
B: At the present moment, it is widely recognised that India holds the balance in the world-wide competition between rival
ideologies.
C: It is not, of course, only in geographical sense that India is in a key position.
D: India’s key position simply needs pointing out. The proper sequence should be
(a) DACB (b) CDAB (c) BCDA (d) BDAC

Ques 9. A: This feeling of an extensive group gives rise to a fellow feeling, a feeling of brotherhood amongst the citizens.
B: This feeling takes up beyond the bounds of family, caste, religion as well as region and helps us to develop a broad
perspective that all of us together constitute an extensive group called ‘the nation’.
C: National integration is the feeling among all the citizens of a country that they all are part of one nation.
D: We do not, then, limit our thinking to our own caste or religion but think about all our fellow citizens.
(a) CDAB (b) CABD (c) CDBA (d)CBAD

Ques 10. A: The peasant, the shoemaker, the sweeper and such other lower classes of India have much greater capacity for work
and self-reliance than you.
B: Remember that the nation lives in the cottage.
C: They are producing the entire wealth of the land without a word of complaint.
D: This process of production is going through long ages. The proper sequence should be
(a) BDAC (b) BDCA (c) DCBA (d) BACD

Directions (Ques. 11-15) : In questions 11 to 15, there are six sentences marked as S 1,S6 , P,Q,R,S. The positions of S1 and S6 are fixed
as the first and last sentence of the passage. You are required to choose one of the four alternatives given below every passage,
which would be most logical sequence of the sentences in the passage.
Ques 11. S1 : His wrist watch had gone out of order.
S6 : His estimate appeared reasonable.
P: He took it to a watch repairer.
Q: He gave an idea of the likely cost of the replacement based on the examination of the watch.
R: He found that some parts needed replacement.
S: The repairer opened the outer case and checked the parts.
(a) PQSR (b) RQSP (c) PSRQ (d) RPSQ

Ques 12. S1 : Now-a-days, soap is going almost out of use as a washing agent.
S6 : There are better washing agents than soap,but scientists are not yet sure if their use is harmless to man.
P: They produce lather due to the presence of calcium salts in water.
Q: Its place has been occupied by a new range of chemicals, called detergents.
R: So, they are called soapless soaps.
S: Detergents are not soaps because they are not sodium or potassium derivatives of fatty acids, as a normal soap is.
(a) SQRP (b) QSRP (c) SQPR (d) QPRS

Ques 13. S1 : Our house is high up on the Yorkshire coast and close to the sea.
PEV 107 Page 118
S6 : Between the two, shifting backwards and forwards at certain seasons of the year, lies the most horrible quicksand
on the shores of Yorkshire.
P: One is called the North spit and another the South.
Q: The sand hills here run down to the sea and end in two stretches of rock, sticking out opposite to each other.
R: This one leads through a dark plantation of fir-trees and brings you out between low cliffs to the loneliest and ugliest little bay on
all our coasts.
S: There are beautiful walls all around us in every direction except one.
(a) SQRP (b) QSPR (c) QPSR (d) SRQP

Ques 14. S1 : Unhappiness and discontent spring not only from poverty.
S6 : We suffer from sickness of spirit and hence, we should discover our roots in the eternal.
P: Man is a strange creature, fundamentally different from other animals.
Q: If they are undeveloped and unsatisfied, he may have all the comforts of the wealth, but still feel that life is not worthwhile.
R: He has far horizons, invariable hopes, spiritual powers.
S: What is missing in our age is the soul, there is nothing wrong with the body.
(a) PRQS (b) SPRQ (c) SPQR (d) PRSQ

Ques 15. S1 : Before we left Bareilly jail, a little incident took place which moved me then and is still fresh in my memory.
S6 : This spontaneous act of courtesy and the kindly thought that prompted; it touched me and I felt very grateful to him.
P: He told me the packet contained old German illustrated magazines.
Q: The Superintendent of Police of Bareilly, an Englishman, was present there, and as I got into the car, he handed to me, rather
shyly, a packet.
R: I had never met him before, nor have I seen him since and I do not even know his name.
S: He said that he had heard that I was learning German and so, he had bought these magazines for me.
(a) RQPS (b) QPSR (c) QPRS (d) RQSP

Exercises : Advanced
Read the given sentences and rearrange them logically to from coherent paragraph:

A. KPDC has been set up by Exim Bank in partnership with AfDB, State Bank of India and IL&FS.
B. Opportunities for infrastructure development in Africa are huge with a World Bank study few years back projecting the annual
investment requirement in infrastructure at $ 90 billion.
C. At their bilateral talks, both sides also discussed the aspect of getting the recently formed Kukuza Project Development Company
(KPDC) “off the ground and run”
D. New Delhi continues to strengthen its relations with Africa, accounting for investments estimated at $ 7.4 billion, mostly lines of
credit channelled through Exim Bank, for the implementation of projects across Africa.
E. It’s main purpose is to essentially look to bring infrastructure projects in Africa to a bankable stage and facilitate exports from
India to Africa.
1. Which is the Fourth sentence after Rearrangement?
a) E b) A c) B d) D

2. Which is the First sentence after Rearrangement?


a) C b) B c) E d) A

3. Which is the Fifth sentence after Rearrangement?


a) B b) D c) C d) E

4. Which is the Second sentence after Rearrangement?


a) E b) A c) C d) D
5. Which is the Third sentence after Rearrangement?
a) A b) C c) E d) B

PEV 107 Page 119


Read the given sentences and rearrange them logically to from coherent paragraph:
A. Business strategies should be established based on how a business ecosystem is likely to evolve, not on what we think we can
excel at.
B. To achieve this, a new approach is needed that will help to navigate the complexity and uncertainties of the business ecosystem
landscape.
C. New players emerge while other business activities are unbundled.
D. With the ongoing transformation of the business landscape, many industry boundaries have drifted, blurred and changed.
E. Mapping and analysing ecosystems is about identifying, testing and selecting options to create and capture value.
F. It is about forming new hypotheses and defining how they can be tested and implemented.

6. Which is the Fifth sentence after Rearrangement?


a) E b) A c) B d) D e) C

7. Which is the Third sentence after Rearrangement?


a) C b) B c) E d) A e) F

8. Which is the First sentence after Rearrangement?


a) B b) D c) C d) E e) A

9. Which is the Second sentence after Rearrangement?


a) E b) F c) C d) D e) B

10. Which is the Fourth sentence after Rearrangement?


a) A b) C c) E d) B e) D

PEV 107 Page 120


Critical Reasoning
Critical thinking is the ability to think clearly and rationally about what to do or what to believe. It includes the ability to engage in
reflective and independent thinking. Someone with critical thinking skills is able to do the following:

 understand the logical connections between ideas


 identify, construct and evaluate arguments
 detect inconsistencies and common mistakes in reasoning
 solve problems systematically
 identify the relevance and importance of ideas
 reflect on the justification of one's own beliefs and values

Example of a Critical Reasoning Text

A CEO of a major company noted a serious decline in worker productivity during the previous five years. According to a report done by
an outside consultant, productivity dropped by 35% by the end of that period. The CEO has therefore initiated a plan to boost
productivity by giving employees shares of the company as part of their pay package.

Conclusion
Most problems have a central idea or thesis. This is almost always located in the sentence at the beginning of the text or in the
sentence at the very end. In this case, it is at the end of the passage: The CEO has therefore initiated a plan to boost productivity by
giving employees shares of the company as part of their pay package. Words like therefore, thus, hence, and so usually tell us that
this is the conclusion.

Premise
Premises are the facts or evidence that support or lead to the conclusion. Unlike assumptions, they are explicit. Here is an example
from the text: A CEO of a major company noted a serious decline in worker productivity during the previous five years. This premise
helps the author lead to the conclusion or main idea of the text.

Assumption
Assumptions are the facts that support the conclusion, like the premise does, but unlike the conclusion and premises they are not
stated in the text, they are implicit. Here is what would be an example of an assumption for this particular Critical Reasoning
problem: Owning something or part of something obliges you work harder to make it succeed.

Supporting Information
Like a premise, this is stated and explicit information embedded in the text, but unlike a premise, it does not support the conclusion. At
best, it supports a premise or provides further detail or information regarding a premise. From the text: According to a report done by
an outside consultant, productivity dropped by 35% by the end of that period.

Arguments in real life can take a number of forms, but arguments on Critical Reasoning questions are relatively formulaic. The typical
argument has three parts:
1) Premise: the starting point of deductions; often, agreement to this is assumed.
2) Conclusion: what the author wants you to believe by the end of the argument
3) Assumption: the unstated link between premise and conclusion. Although unstated, the assumption is the nerve center of the
argument, the linchpin holding the whole thing together.

Premise (s) + Assumption (s) = Conclusion

Argument: Every hockey fan I know is nice. I do not know Judy, but since she is wearing a hockey jersey, she must be nice.
Premise: Every hockey fan I know is nice. I don’t know Judy. Judy is wearing a hockey jersey.
Assumption: Since Judy is wearing hockey jersey, she is a fan.
Conclusion: Judy is nice

PEV 107 Page 121


Types of Critical Reasoning Questions

Critical Reasoning Questions can be categorized into five major types.

1. Weaken the Argument


2. Strengthen the Argument
3. Find the Assumption
4. Find the Conclusion / Draw Inference
5. Paradox Questions

Premise vs. Conclusion

A premise includes the reasons and evidence behind a conclusion. A conclusion is the statement that the premise supports and is
a way of promoting a certain belief or point of view. To help us better identify the premise and conclusion of an argument, we can
take a look at indicator words.

Consider the following argument: Since carrots are full of vitamins, it follows that your body will benefit if you eat them.

In this argument, how do we know which part is considered the premise and which part is the conclusion? The premise here is the
fact that carrots are full of vitamins. The conclusion is that your body will benefit from you eating carrots.

This statement about carrots includes indicator words. Indicator words provide assistance to you when you are trying to identify
an argument and its parts. The phrase Since carrots are full of vitamins uses the indicator word 'since' which is often associated
with premises. The last part of the sentence uses the phrase, 'it follows that' to show that it is a conclusion.

Examples of words or phrases that are typically included in premises:

Because since given that seeing that as shown by assuming that considering that for the reason
that

Examples of words or phrases that are typically included in conclusions:

Therefore thus it follows that which proves/implies that which means that as a result so we may
conclude

If the passage contains no indicator words, try these two strategies:


a. Ask yourself, "What claim is the writer or speaker trying to prove?" That claim will be the conclusion.
b. Try putting the word "therefore" before each of the statements in turn. The statement it fits best will be the conclusion.

Practice: Identify premise and conclusion in the following arguments and label them as (p) and (c) respectively.

1. No brass instruments use reeds, and flutes don't use reeds, so flutes must be brass instruments.
2. Betty will be angry unless someone else brings the music stands. But if Ann doesn't bring them, no one will. So either Ann will bring
the music stands or Betty will be angry.
3. If we don't consolidate city and county school systems, the city school system will continue to deteriorate, producing a large
number of young adults who are not equipped to find work that will keep them out of poverty. We must not allow this disastrou s
social situation to occur, so we must consolidate city and county schools.
4. Many herbs are known to have medicinal properties. So it is not a waste of taxpayer dollars to finance trials of herbal treatments
that appear implausible. Open-mindedness is a friend of scientific investigation, and since herbal treatments are widely used, it is
important to determine whether they work and whether they have any harmful effects.

PEV 107 Page 122


5. A steady movement of people from the city to suburban and rural areas has decreased the city's population, increased the
percentage of its population that are poor, and increased the percentage of its population that are African-American and Hispanic. If
the tax base continues to shrink, then the resulting poor support for education and services, combined with racial and ethnic
polarization, will lead to increasing tension between city and suburban populations. We must stop this trend. The only way to stop it is
by consolidating city and county governments, so that there is a single tax base in support of a unified, high quality system of
education and a uniform level of municipal services.
Answers:
1. [P1]No brass instruments use reeds, and [P2]flutes don't use reeds, so [C1]flutes must be brass instruments.
2. [P1]Betty will be angry unless someone else brings the music stands. But [P2]if Ann doesn't bring them, no one will. [C1]So either
Ann will bring the music stands or Betty will be angry.

3. [P1] If we don't consolidate city and county school systems, the city school system will continue to deteriorate, producing a large
number of young adults who are not equipped to find work that will keep them out of poverty. [P2]We must not allow this disastrous
social situation to occur, so [C1] we must consolidate city and county schools.

4. [P1]Many herbs are known to have medicinal properties. So [C1]it is not a waste of taxpayer dollars to finance trials of herbal
treatments that appear implausible. Open-mindedness is a friend of scientific investigation, and since [P2] herbal treatments are
widely used, [C2] it is important to determine whether they work and whether they have any harmful effects.

5. [P1] A steady movement of people from the city to suburban and rural areas has decreased the city's population, increased the
percentage of its population that are poor, and increased the percentage of its population that are African-American and Hispanic.
[P2]If the tax base continues to shrink, then the resulting poor support for education and services, combined with racial and ethnic
polarization, will lead to increasing tension between city and suburban populations. [P3]We must stop this trend. [P4]The only way to
stop it is by consolidating city and county governments, so that there is a single tax base in support of a unified, high quality system of
education and a uniform level of municipal services.

Weaken the Argument/Find the Flaw

There are two ways in which you can weaken an argument:


I. An answer that weakens the argument can directly disprove the assumption. Any choice that states that the assumption is wrong will
weaken the argument.

II. If different evidence can strengthen the argument; different evidence can weaken the argument as well. Any new information given
in an answer choice that makes the assumption less likely to be correct will weaken the argument as a whole.

A Weaken the Argument question may be worded as:

 Which of the following, if true, most seriously weakens the argument?


 Which of the following, if true, would cast the most doubt on…?
 The objection implied above…is based on doubts about…?
 Which of the following points to the most serious logical flaw in the author’s argument?

Read the sample question given below to get more clarity:

The cellular service quality has dropped significantly in India. Ten years ago, there were no instances of call drop or no connectivity.
There is also a decrease in the speed and reliability of service.

All of the following would tend to strengthen the conclusion of the argument above except:

A. The volume of connections handled by the Mobile operators has increased dramatically over the last ten years.
B. Unprecedented increases in the cost as well as scarce availability of spectrum for mobile services have put severe pressures on the
Mobile companies.
C. Mobile services have diversified from carrying mere voice data to a whole range of internet data such as downloading, video-calling,
data sharing, etc.

PEV 107 Page 123


D. The opposition to negative externality of cellular radiation has obstructed increase of network services in response to the increasing
subscriber base.
Understand the question carefully. The question is asking you look for an option that will weaken our conclusion. For that you need to
find the conclusion. The conclusion in the given argument is 'The cellular service quality has dropped significantly in India.'

Now if we look at the option A, it is uprightly giving us a reason as why there is a decline in the services. If the volume of the mobile
connection subscriber increase in a dramatic way, and the service provider is not in a capacity to handle it perfectly, it will surely lead
to mismanagement.

And similar is the case with option B, as- if there is scarcity of spectrum it will be difficult to give good services.

We are now left with option C and D. Option C says that as compared to the older times when the cellular companies were catering to
only one kind of service whereas, now they have diversified their services. This option is weakening the main argument, which makes it
the correct answer option.

Option D is also strengthening the conclusion as it says that due to the negative externality of the radiation, people are opposing it but
they are not opposing the use of mobile phones, which means there may be less mobile towers in comparison to the increasing
subscriber base.

Strengthen the Argument

If you are asked to look for an answer choice that strengthens the argument; then find the answer choice that corroborates that the
central assumption is correct.

There are two ways of strengthening an argument:


I. If you come across an answer choice that would be correct according to an assumption question, it would also be correct for a
strengthen question. The correct answer can simply be a paraphrasing of the assumption itself.

II. If the correct answer can confirm the assumption by mentioning an experiment, survey or any other proof and helps the assumption
appear to be true, then it will support our main argument.

A Strengthen the Argument question may be worded as:

 Which of the following, if true, most strongly supports…?


 Which of the following, if true, would most strengthen…?
 The statements above, if true, best support which of the following assertions?

Read the sample question given below to get more clarity:

Beasley & Halpert Law Firm has instituted an Employee Wellness Program that will provide attorneys and support staff with free
access to Smokers Anonymous programs, diabetes monitoring, and discounted memberships to a local gym. Similar programs at other
firm have been shown to improve workplace attendance and performance, and reduce the employer’s costs for employee health
insurance. Thus, the Employee Wellness Program will be good for both the employees and the firm.

If true, which of the following would best support the conclusion of the argument above?

a) Many employees take advantage of free diabetes monitoring when it is offered by employers.

b) Smokers Anonymous programs are only effective for 20% of those smokers who use them.

c) Discounted memberships at a local gym will make it easier for employees to improve their cardiovascular health and reduce the
incidence of serious illness.

d) Exercising without the help of a personal trainer can often lead to injury due to incorrect use of weight-training equipment.

e) Beasley & Halpert will give employees taking part in the Smokers Anonymous program one paid hour off each Friday afternoon to
attend the group meetings.

PEV 107 Page 124


Choice A - The monitoring’s popularity might indicate that it is beneficial to the employees, but it might not. This choice doesn’t clearly demonstrate
that the Employee Wellness Program benefits the employees, and is a good example of a wrong answer.

Choice B makes it LESS likely that the programs will benefit either the employees or the firm. This answer choice may catch your eye if you didn’t read
the question closely enough, and are mistakenly looking for a weakener instead of a strengthener.

Choice C is the correct answer. The argument seems to imply that the increased attendance and performance and reduced health insurance costs are
due to improved employee health, which would naturally benefit the employees. This choice makes that unstated implication clear, and fills the gap
in the argument.

Choice D might be tempting, but again, it requires too many assumptions to tie it into the argument as a strengthener. In order for this to strengthen
the argument, one must assume that employees would still exercise without the personal training services, and that they would incorrectly use the
weight-training equipment.

Choice E might benefit the employees, but it would be a burden to Company X. Therefore, it’s not the best choice.

Find the Assumption

Assumption questions will ask you to select the answer choice with the information that must be true (the ‘assumption’) in order for
the given argument to be accurate. To figure out the answer, you’ll first need to figure out the main thrust of the given argument. The
correct answer choice, again, will have to be true in order for that argument to be logical. Incorrect answer choices will often
be possibly true, but won’t be absolutely necessary to the argument’s validity.

A Find the Assumption question may be worded as:

 Which of the following is an assumption made in drawing the conclusion above?


 Which of the following in an assumption on which the argument relies?

Read the sample question given below to get more clarity:

Last year, support for the social and behavioural sciences represented only about three percent of the government’s total budget for
research funds in the United States. Thus the particularly sharp reductions imposed on such programmes this year seem to be dictated
not by financial constraints but by social philosophy.

Which of the following is an assumption on which the conclusion of the above passage is based?

A. Government funding is the primary source for research money in the United States.

B. The social and behavioural sciences are as valuable as physical and biological sciences.

C. Three per cent is an insignificant portion of the government’s total budget for research funds.

D. The government funds allocated for research in the social and behavioural sciences are not sufficient for the work that needs to
be done.

It’s a simple assumption question. The conclusion, quite clearly, is that the reduction is dictated not by financial concerns, but by social
philosophy. The reasoning here is that since these fields are a mere three percent of the total money spent on research, it would be
foolish to make drastic reductions purely for financial reasons. The basis of the reasoning is obviously that three percent of the money
spent on research isn’t really a significant sum – which is what option C states. C is the correct answer.

While answering a question, you often find that many options seem extremely relevant to the given situation. Remember then, your
job is not to find what is relevant, but to deduce the basis of the argument.

The Negation Test

This test always works, and it always a good way to verify an assumption. Here’s the rule:

If you negate a statement, and it’s still possible to imagine that the conclusion is still true even with this negated statement, then
that original statement is definitely not an assumption of the argument.

PEV 107 Page 125


If you negate a statement, and this negated statement is a devastating objection which shatters the argument and makes the
conclusion untenable, then that original statement is an assumption of the argument.

Here’s a super-simple argument, with only three answer choices:

3) Alex likes this movie. Therefore, Betty will like it.

Find the assumption of the argument

A. Both Alex & Betty liked the same movie last year

B. Carla didn’t like this movie, and last year, she & Betty liked the same movie.

C. Betty likes the movies that Alex likes.

Find the Conclusion/Draw Inference

Inference questions ask you to make inferences—draw logical conclusions—based on the evidence in the given passage. Key words
you might see are imply or infer.

An Inference question may be worded as:

 Which of the following can properly be inferred from the statements above?
 Which of the following, if true, best supports as a conclusion??

For inference questions, you must draw conclusions only from the information you’re directly given in the passage. Beware of
answer choices that contain words like ‘any,’ ‘best,’ ‘worst,’ ‘only,’ ‘all,’ or ‘none,’ as they are often overly general and can’t be verified
by the limited information in the passage. Incorrect answer choices will often be overly extreme; make leaps in logic that can’t be
verified based on the given info, or contain tangential/unrelated information.

Read the sample question given below to get more clarity:

Randall: Many of the productions of my plays by amateur theater groups are poorly done, and such interpretations do not provide a
true measure of my skills as a dramatist.

Which one of the following can be properly inferred from Randall’s statement?

(A) Some amateur theater groups’ productions of Randall’s plays provide a true measure of his skills as a dramatist.
(B) All amateur theater group productions of Randall’s plays that are not poorly done provide a true measure of his skills as a
dramatist.
(C) All of the productions of Randall’s plays by amateur theater groups that do not provide a true measure of his skills as a dramatist
are poorly done.
(D) If a production of a dramatist’s play is well done, then it provides a true measure of his or her skills as a dramatist.
(E) At least some amateur theatrical groups’ productions of Randall’s plays fail to provide a true measure of his skills as a dramatist.

(A) seriously distorts Randall’s statement. Just because some amateur productions don’t do him justice doesn’t mean that there are
other productions that do. If the GMAT tells you that some marbles are red, you can’t automatically infer that some are not red.

(B) is another sort of distortion. Randall’s statement about certain poorly done productions in no way guarantees anything about
productions that aren’t poorly done.

(C) is far too extreme. Randall does establish a correlation between poor production quality and failure to provide a true measure of
his skills, but that correlation has only been established for a certain set of productions and can’t be extended to all productions.

(D) attempts to extract a broad principle from Randall’s statement, but his statement is too particular to allow this kind of
extrapolation.

PEV 107 Page 126


The answer is (E).

Paradox Questions

Paradox/Discrepancy questions will ask you to choose the answer choice that explains the paradox in the given argument. A paradox
refers to the coexistence of two seemingly contradictory pieces of information. The correct answer choice will logically explain why
those pieces of information are not actually contradictory. Key words you might see are explain, paradox, or discrepancy.

A Paradox question may be worded as:

 Which of the statements below provides the most likely explanation for the two seemingly contradictory statements above?
 Which of the following, if true, most helps to resolve the paradox outlined above?
 Which of the following, if true, best explains the reason for the apparent discrepancy described above?

Read the sample question below to get the clarity:

French cuisine is famous for its frequent and liberal use of cream and cheese, both high in saturated fat. For years, medical studies
have shown the strong correlation between diets high in saturated fat and coronary heart disease, and yet, France has a much lower
incidence of such disease than found in comparable countries like the United States. This is the so-called French Paradox.

Which of the following, if true, helps to explain the French Paradox?

(A) Certain kinds of cheese can have as much as five times the amount of saturated fat that cream has.

(B) People in the United States, per capita, eat almost the same amount of saturated fat on average as do people in France.

(C) The United States imports more cheese from France than from any other country.

(D) Red wine, typically served with French food, helps to clean the buildup of fats in the arteries, reducing the risk of heart disease.

(E) It is typically for a French person to have either cream or cheese at each of the three meals in a day.

Both (B) and (E) do the opposite: they make the paradox harder to explain. With (B), if folks in the US and France eat about the same
amount of saturated fat, then why do Americans get heart disease but not the French? With (E), if French are eating high fat foods all
the time, why aren’t they getting heart disease? In other words, neither of these answers the question, and in fact, both of them
simply would make it even harder to understand.

Answers (A) and (C) are off-the-wall irrelevant. Choice (A) says that cheese has more fat then cream, but the French are eating both of
those, so it doesn’t matter: either way, the French are eating high fat food. (C) changes the topic to imports, which is completely
unrelated to the direct relationship of diet and epidemiology.

Only (D) resolves the paradox. Since the French drink red wine, which in moderation cleans the arteries, this explains how they could
eat high fat foods and have a much lower risk of heart disease.

Tutorial
Exercises: Weakening, Strengthening, Assumption

1. During the past 20 years, computer scientists focused increasingly on starting and running successful businesses. However, since
businesses must be profitable, computer scientists must focus on developing products that generate profit. Consequently, computer
science has lost its creative aspect.

Which of the following assumptions is most necessary in order for the conclusion above to be drawn from the argument above?

PEV 107 Page 127


A) All computer programs must lack creativity in order to be well received.
B) Some computer scientists entirely disregarded creativity and chose instead to pursue profit.
C) A program cannot be both creative and profitable.
D) Computer scientists are obsessed with the profitability of their work.
E) Non-profit institutions use large amounts of software.

2. For years, a considerable number of students on West County High School's track team complained about shin splints (medial tibial
syndrome). However, during the most recent season, the number of students who complained about shin splints dropped significantly.
School officials assert that this reduction in complaints occurred entirely as a result of the school's decision to build a new running
track that provided a softer running surface, which absorbed much of the shock on the knees and shins that occurs when running and
causes shin splints.

Which of the following, if true, most severely weakens the school officials' explanation for the decrease in complaints about shin
splints?

A) As a result of West County High School's adoption of better medical staff and new medical scanning devices, many
students whose complaints would have been diagnosed in years past as an instance of shin splints are now diagnosed
with a different condition.
B) West County High School built its track after a number of neighboring schools with similar track teams built new tracks
and each school saw the number of complaints about shin splints drop.
C) This past season, members of West County High School's track team received and wore new and highly acclaimed shoes
designed to soften the impact of running on the shin and knee.
D) This past season, the total number of students who complained of pain while running rose.
E) The maker of the new track claims that on average, complaints about shin splints fall 25% when its tracks are
implemented.

3. If the county continues to collect residential trash at current levels, landfills will soon be overflowing and parkland will need to be
used in order to create more space. Charging each household a fee for each pound of trash it puts out for collection will induce
residents to reduce the amount of trash they create; this charge will therefore protect the remaining county parkland.

Which of the following is an assumption made in drawing the conclusion above?

A) The collection fee will not significantly affect the purchasing power of most residents, even if their households do not reduce
the amount of trash t
B) The collection fee will not induce residents to dump their trash in the parklands illegally.
C) The beauty of county parkland is an important issue for most of the county's residents.
D) Landfills outside the county's borders could be used as dumping sites for the county's trash.

4. Last year the rate of inflation was 1.2 percent, but for the current year it has been 4 percent. We can conclude that inflation is on an
upward trend and the rate will be still higher next year.

Which of the following, if true, most seriously weakens the conclusion above?

A) The inflation figures were computed on the basis of a representative sample of economic data rather than all of the available
data.
B) Last year a dip in oil prices brought inflation temporarily below its recent stable annual level of 4 percent.
C) lncreases in the pay of some workers are tied to the level of inflation, and at an inflation rate of 4 percent or above, these pay
raises constitute a
D) Government intervention cannot affect the rate of inflation to any significant degree.

5. Studies last year concluded coffee drinking had little, if any, effect on the risk of pancreatic or kidney cancer. A study of 59,000
women in Sweden found no link between caffeine and kidney cancer. Another review suggested that those who drink coffee have half
the risk of pancreatic cancer.

Which of the following statements, if true, would weaken the conclusion drawn above?

PEV 107 Page 128


A. The study is done in cold countries only, countries in arid and semi-arid regions are not considered under this study.
B. Indians drink more coffee than the Swedish do.
C. The caffeine content is different in different varieties of coffee.
D. Intake of 15 ml to 30 mi of milk per day offsets the negative effects of caffeine. The study did not consider the milk intake.

6. We, at Comfort Stationers, have always been striving to provide stationery items that would make your work more enjoyable and
less strenuous. Our latest innovations are a smooth-flow pen and gradual-friction paper. A combination of these two reduces strain on
your fingers and allows faster writing the causes lesser fatigue. Therefore, replacement of your pen and paper with our innovative
products reduces cost of clerical jobs.

Which of the following, if true, would weaken the conclusion drawn in the above argument?

A. Those who are already using the above new products report greater difficulty in transition from new products to regular ones
than from the regular ones to the new ones.
B. The cost of manufacturing these new products is not more than the cost of manufacturing the regular ones and the new
products last longer than the regular ones.
C. The number of offices using the new products is increasing month by month.
D. These products need to be purchased in huge lots and need to be stored in special conditions. The cost of procurement and
strong is quite high.
E. It is found that children get adapted to the new products faster than grown ups.

7. The Supreme Court in its land mark judgement gave women the right to terminate their pregnancy as a constitutional right.

Which of the following, if true, can go against the Supreme Court's judgement?

A. In several other countries abortion is illegal.


B. An unborn child should be treated as a person and every person has a fundamental right to live.
C. The elected representatives of one of the states rejected the proposal to make abortion illegal in a 30-70 vote.
D. If a foetus can be treated as a person, an enbryo from which the foetus is formed should also be treated as a person.

8. A salesman of cable TV, while trying to sell his product, was explaining to a perspective customer. "Free TV channels are not really
free. You end up paying heavily for the costly advertisements that support free TV channels."

Which of the following, if true, goes against the statement of the salesman?

A. Consumers who do not own a TV are the ones who spend less on purchase as compared to those who own a TV.
B. At places where the reception of signals is very weak, cable TV will be able to provide a better picture quality.
C. Cable TV channels also have as many advertisements as free TV channels.
D. Those who subscribe to cable TV channels also get access to some free TV channels.

9. The board of directors of C & C, a commercial training institute, are split over whether to continue with NDF course because the
government has instructed all the government approved colleges to take admissions based on the scores obtained by the candidates in
the new KJB test.

Which of the following, if true, would strengthen the case for continuing with NDF course?

A. KJB test is on the similar lines as NDF.


B. NDF is tougher than KDF, hence those who prepare for KJB will be able to face NDF easily.
C. There are many private colleges which are preferred by students to government colleges.Which still want to take admissions
based on the some obtained in NDF.
D. The government colleges have better infrastructure than the private colleges.

10. Many people are of the opinion that physical exercise should be made mandatory for employees to improve their efficiency at
workplace.

Which of the following supports the opinion expressed in the above statement?

PEV 107 Page 129


A. It is found that work environment is a better motivating factor than monetory benefits.
B. Even for the most able sportsperson, physical exercise is an essential part of practice session.
C. Children who do physical exercise show better academic performance than the ones who do not.
D. The chances of occurrences of mistakes increases if an employee is subjected to the same work for more than six months.

11. Electricity would revolutionize agriculture according to a prediction of the not-so-distant future published in 1940. Electrodes
would be inserted into the soil, and the current between them would kill bugs and weeds and make crop plants stronger.

Which of the following, if true, most strongly indicates that the logic of the prediction above is flawed?

A. In order for farmers to avoid electric shock while working in the fields, the current could be turned off at such times without
diminishing the intended effects
B. If the proposed plan for using electricity were put into practice, farmers would save on chemicals now being added to the soil.
C. It cannot be taken for granted that the use of electricity is always beneficial.
D. Since weeds are plants, electricity would affect weeds in the same way as it would affect crop plants.
E. Because a planting machine would need to avoid coming into contact with the electrodes, new parts for planting machines
would need to be designed.

12. In anticipation of the coming year, Tecumseh Autos, a national auto manufacturer, is anticipating sales of its vehicles. Tecumseh
manufactures compact cars, sedans, minivans, trucks, SUVs, and sports cars. In all categories of vehicles, Tecumseh sets prices so that
the profit per vehicle is, on average, about the same. Since the best indicator for sales in each category are the sales last year,
Tecumseh’s marketing analysts’ prediction of the three most profitable categories of vehicles in the coming year will be compact cars,
minivans, and SUVs respectively.

Tecumseh’s marketing analysts’ prediction relies on which one of the following assumptions?

(A) Across all manufacturers, the most popular cars on the road in America are compact cars, minivans, and SUVs.
(B) The models of Tecumseh’s compact cars to be sold in the upcoming year are identical to or similar to those of last year.
(C) Last year, no other category of Tecumseh’s vehicles generated more profits than SUVs and less than minivans.
(D) The prediction will be refined after an analysis of the sales in the first quarter of this year.
(E) The number of models of compacts cars that Tecumseh produces is greater than the number of models of either minivans or
SUVs.

13. Most people can gain vitamin C from fruits such as oranges and cantaloupes. People with Laestrygonian Disease have weakened
digestive systems that cannot digest fruit or vitamin supplements. The easiest foods for these people to digest are grains such as rice
and barley. Regular intake of vitamin C would be extremely beneficial to those who suffer from Laestrygonian Disease, so scientists
have figured out a way to create “fortified rice” by infusing rice with high doses of vitamin C. This fortified rice will provide great
benefit to those with Laestrygonian Disease.
Which one of the following is an assumption on which the conclusion depends?

(A) Eventually, this fortified rice will be the optimal way for most people to have a regular intake of vitamin C.
(B) The problems that folks with Laestrygonian Disease have digesting fruit are different from their problems digesting vitamin
supplements.
(C) People with Laestrygonian Disease will not be unable to assimilate the form of vitamin C that is present in the fortified rice.
(D) Only people whose diets consist largely of grains would be able to derive benefit from the vitamin C in the fortified rice.
(E) Vitamin C is the only nutrient which can be infused into rice in such high quantities without compromising the nutritional
integrity of the vitamin.

14. Sale Analyst: When polled, all consumers consistently say that, for household staples, they would buy the lowest cost items. Even
when other factors, such as inherent product quality, are introduce, all consumers still argue that low cost should be the highest
priority in buying household staples. Therefore, these responses demonstrate how little most people are aware of the actual priorities
that drive their purchasing decisions.
The conclusion above is properly drawn if which of the following is assumed?

(A) Many people on restricted incomes are not able to afford any brands of household staples more expensive than the lowest
cost items.

PEV 107 Page 130


(B) Consumers do not always have the motivation to be truthful about their real motivations when responding to a stranger
giving a poll.
(C) Often, higher priced brands of household staples, especially organic versions, are more nutritious and are rated better-tasting
in double-blind tests.
(D) People often give unreliable and self-contradictory explanations after the impulse purchase of a snack or dessert.
(E) For many household staples, famous name brands with recognizable ads consistently outsell less expensive brands of the
same products.

15. Urban planner: The mayor of Dismaston supports a new tax code that would assess local businesses on the property value of their
site rather than on their income or profits; the mayor argues that this change will not contribute to any loss in tax revenue for the city.
Several city council members disagree, citing similar changes to tax code that were unsuccessful in cities similar to Dismaston. The
council members’ argument is without merit, though, because property values rise steadily each year, while business incomes
fluctuate wildly with the national economy.
Which of the following is an assumption on which the argument depends?

(A) The new tax code would not apply to the significant number of online business associated with the city that have no physical
presence on a piece of property in the city.
(B) In previous years, the successes and failures of many previous Dismaston tax codes to generate income have been mirrored
by similar tax codes in the the same cities similar to Dismaston in other respects.
(C) A small number of store fronts in the downtown neighborhood are vacant: under the new code, these businesses are likely to
owe higher taxes than they have paid, unless some loophole is written into the law.
(D) The annual percentage rise in real estate values in Dismaston has been consistently more than the average annual growth
rate percentages across all businesses with properties in Dismaston.
(E) In any year, some unsuccessful businesses will close and other businesses, some quite promising, will open, but total amount
of property in the city is fixed and unchanging, providing greater stability.

16. In order to combat Carville’s rampant homeless problem, Mayor Bloomfield recently proposed a ban on sleeping outdoors in the
city’s many parks. He claims that such a measure will force the homeless to either leave Carville or to find means other than sleeping in
public parks.

Which of the following, if true, suggests that Mayor Bloomfield’s plan will be successful?

(A) Until the ban, the city’s many homeless shelters were at less than fifty percent occupancy.
(B) Many homeless tend to congregate underneath Carville’s numerous overpasses.
(C) Adjacent cities have even tougher measures on the homeless sleeping outdoors.
(D) The percent of Carville’s population that has been homeless has been slowly decreasing in the last five years.
(E) Mayor Jonesmith, Mayor Bloomfield’s predecessor, had been far more tolerant towards the city’s homeless population.

17. Megalimpet is a nationwide owner of office space. They have major office buildings in the downtowns of several cities in the 48
lower states, and rent this space to individual companies. Megalimpet office spaces vary from small office to large suites, and every
space has custom-designed wall-to-wall carpeting. The carpet in several Megalimpet facilities needed replacing. The winning bid for
the nationwide carpet replacement was submitted by Bathyderm Carpet Company (BCC). The bid contract involves all delivery costs,
all installation, and any ongoing maintenance and upkeep while the carpet is under the three-year warranty. Both BCC executives and
independent consultants they hired felt BCC would be able to perform all these services for far less than their bid price; these
circumstances would allow BCC to reap a considerable profit.

Which of the following, if true, most calls in question the argument that BCC will make a large profit from this contract with
Megalimpet?

(A) All the carpets will have to be transported by train from BCC factory in Louisville, KY, to Megalimpet’s locations from coast to
coast.
(B) BCC has already supplied carpets to a number of restaurant chains, and some of those spaces are as large as Megalimpet’s
largest office spaces.
(C) The carpet installation teams will have to cut different sizes of the carpets for the different size office suites in the
Megalimpet buildings.

PEV 107 Page 131


(D) The material in BCC carpets degrades rapidly when it comes into contact with standard toner, found in most laser printers and
photocopiers; the degraded sections are unsightly and smell bad, so they often need to be replaced.
(E) The next competing bid after BCC’s was 50% higher than BCC’s bid

18. A minor league baseball franchise experienced a drop in attendance this week after they suffered three losses by margins of ten
runs or more last week. Many spectators of those games wrote letters to the editors of the local sporting news, complaining of the
poor play of the team in those three losses. Nevertheless, the front office of this baseball franchise maintains that the team’s poor
play in those three losses has nothing to do with this week’s decline in attendance.

Which of the following, if true, most strongly supports the position held by the front office of the baseball franchise?

(A) The spectators who wrote letters to the local sporting news were long-standing fans of this minor league baseball team.
(B) Many minor league baseball franchises attribute a drop in attendance to the quality of play of the team only after a string of
losses.
(C) Other minor league teams in that region of the state reported a similar drop in attendance this week.
(D) This was not the first time this team suffered multiple lopsided losses in a single week, prompting similar letters to the local
sporting news.
(E) This minor league team is over four hours from the closest major league team, so many of the minor league team’s fans do
not often attend major league games.

19. In the twentieth century, the visual arts have embarked on major experimentation, from cubism to expressionism. While tastes
always vary, there are certainly some people who find beautiful objects of each of the art movements of the first half of the twentieth
century. In the latter half of the twentieth century, though, most works are so abstract or shocking that neither the critic nor the
general public uses the word “beautiful” to describe them: indeed, sometimes late twentieth-century artists have, as one of their
expressed goals, the creation of a work that no one could find beautiful. Whatever these artists are creating may be intellectually
engaging at some level, but it is no longer art.
Which of the following is an assumption that supports drawing the conclusion above from the reasons given for that conclusion?

(A) Art critics generally have a different appraisal of a work of art than does the general public.
(B) The meaning of any work of art is defined entirely by the ideas of the artist who created it.
(C) Beauty is a defining quality of art.
(D) All art movements of the latter half of the twentieth century are responses to the movements of the first half of the century.
(E) It is not possible for any work to be simultaneously beautiful and intellectually engaging.

20. The National Farm Administration (NFA) has been concerned over the last decade with the struggles of barley growers.
Plan: In an effort to support these barley growers, two years ago, the NFA began a program of sending them, each autumn, a free
special mix of fertilizer and enzymes designed to multiply barley yield, to be applied the following spring during first growth. This mix
had been stunningly successful in multiplying the yield of barley in laboratory conditions.
Results: Most barley growers reported little change in their economic status over this two year period.
Further information: All barley growers received the shipments, and all used them. Weather conditions have been fair to optimal for
barley growth over the past two years.
In light of the further information, which of the following, if true, does most to explain the result that followed the implementation of
the plan?

(A) During these two years, most of the barley growers reported using no other fertilizer besides the special mix sent by the
government.

PEV 107 Page 132


(B) The trucks that drove the special mix from the depot in Wisconsin to the individual farms sometime took as much as 4 or 5
days.
(C) Some of the enzymes in the special mix multiply the growth of a bacteria that feeds on the young barley plants.
(D) This program was implemented at a time when more than half of barley growers nationwide were reported barely breaking
even in their yearly expenses.
(E) This was the second such NFA program to aid barley growers; the first one, 14 years ago, was started with high hopes, but did
little to change their situation.

Exercises: Find conclusion/Draw inference, Paradox

1. Many scientific studies have found links between genius and mental illness, particularly bipolar disorder in which patients have
violent mood swings between elation and depression. In one interesting Swedish study, 700,000 Swedes had intelligence tests at age
16 and again 10 years later. Those who scored well were four times more likely than the others to have developed bipolar disorder.
The US neurologist James Fallon came up with a convincing argument based on his own findings in the field: the brain area involved in
mood swings is the same area where creativity is born. This may explain why some people can draw previously unseen connections
among ideas, images, shapes and the like.

Which of the following is inferred from the above?

A. Those with bipolar disorder are likely to be highly intelligent.


B. Most people who excel in creativity are likely suffering from a mental problem.
C. Often there is a correlation between mental illness and genius.
D. Mental disorders give birth to genius.

2. India’s baffling array of state and national labor laws date to the 1940s: one provides for the type and number of spittoons in a
factory. Another says an enterprise with more than 100 workers needs government permission to scale back or close. Many Indian
businesses stay small in order to remain beyond the reach of the laws. Big firms use temporary workers to avoid them. Less than 15%
of Indian workers have legal job security. The new government can sidestep the difficult politics of curbing privileges by establishing a
new, simpler labor contract that gives basic protection to workers but makes lay-offs less costly to firms. It would apply only to new
hires; the small proportion of existing workers with gold-star protections would keep them.

Which of the following options best summarizes the main idea of the paragraph?
(A) More Indian workers can get permanent jobs and legal job security if existing labor laws are reformed.
(B) Effective labor law reform can encourage many Indian businesses to grow to more than 100 workers.
(C) Outdated Indian labor laws need to be simplified to provide basic protection to workers and curb privileges.
(D) The difficult politics of curbing privileges can be avoided if the changes in the labor law only apply to the new hires.

3. A study published in 2006 by Friedrich Schneider on the world’s shadow economies dealt briefly with the “tax morality” of Germans.
According to the study, two-thirds of the Germans surveyed regarded tax evasion as a “trivial offence,” while only one-third judged
stealing a newspaper this way. Indian tax morality is similar, but it makes a distinction between expatriate illicit money, which is
viewed as a serious crime perpetrated by the very corrupt, and money held within India, which is perceived as a practical measure.

Which of the following conclusions can be drawn from the above?


(i) Two-thirds of the Germans evade tax and consider it only a trivial offence.
(ii) Stealing a newspaper is a bigger crime in Germany than tax evasion.
(iii) As long as the money is held in India, illicit money is accepted as practical by Indians.
(iv) Indians regard tax evasion, especially holding illicit money abroad, as a serious crime.
(A) i, ii, iv
(B) i and iv
(C) only iii
(D) iii and iv

4. Data on planes returning from bombing missions was used to study of the vulnerability of airplanes to enemy fire. Analyzing the
pattern and frequency of hits from enemy gunfire, it was seen that some parts of planes were hit disproportionately more often than
PEV 107 Page 133
other parts. How could these planes be optimally reinforced with armor plating? There were tradeoffs to consider. Every addition of
plating added to the weight of the plane, decreasing its performance. Therefore, reinforcements needed to be added only to the most
vulnerable areas of the planes.

Which of the following can be concluded from the above?


(A) The parts hit disproportionately more than the others have to be reinforced as those received the maximum amount of
damage.
(B) No conclusion can be drawn as the data set is incomplete. There is no data on the planes shot down.
(C) The parts with the least damage have to be reinforced, as the returning planes have survived attacks to the most damaged
areas.
(D) Reinforcements have to be added to all areas of the plane.

5. Oklahoma is not perceived as overpopulated because, in spite of a horrendous drought, it is not facing famine. Famine in Oklahoma
is inconceivable because it receives a fair price for its exports, it has not leased its land to foreign countries, the poorest of the poor
receive a helping hand from the government, and farmers and ranchers receive federal assistance in times of droughts. It is a lack of
these factors in Horn of Africa, plus political insecurity in Somalia, which explain the famine – not overpopulation.

Which of the following options best summarizes the main idea of the paragraph?
(A) Hunger is caused by too many people pressing against finite resources.
(B) In spite of drought and overpopulation, there is no famine in Oklahoma.
(C) Overpopulation and famine are not causally related in the Horn of Africa.
(D) Famine in the Horn of Africa is not only due to overpopulation but, more importantly, due to the lack of government
assistance and political insecurity.

6. In Dec 2014, the Pew Research Center conducted a survey, asking 1507 people spread among all 50 states and the District of
Columbia , “What do you think is more important – to protect the right of Americans to own guns or to control gun ownership?” For
the first time in more than two decades, a higher percentage (52%) said it was more important to protect the right of Americans to
own guns than to control gun ownership (46%). The researchers used 1993 as a reference point because it's seen as the height of gun
violence in America; they also noted that gun-related violence fell sharply in the 1990s and more gradually in later years. But Pew also
noted that many Americans didn't seem to agree with the numbers. A survey found that only 12 percent of respondents thought the
gun crime rate was lower than it was in 1993 — and 56 percent thought it was higher.

It can be concluded from this survey result that:


(A) Though gun crime is thought to be rising, a majority of people are against stricter background checks on gun purchases.
(B) Most people are not in favor of proposals to restrict gun sales as gun-related violence is falling.
(C) More Americans believe the right to own a gun should be protected, despite perceived increase in gun crime.
(D) With falling gun crime, fewer Americans support controls on gun ownership.

7. Contrary to popular belief, the idea of evolution didn’t originate with Darwin, but was around for decades before he came along. His
accomplishment was to come up with a workable scheme by which it likely occurred. Darwin, it must be said, had unusual exposure to
the enormous diversity of life on earth for a man living in his time, through his voyage on the Beagle. However, his eureka moment
came not through studying biology, but by reading the paper of an economist, Thomas Malthus, which showed that populations grow
faster than the resources to sustain them. It was then that Darwin realized that only those best adapted to their environment would
survive and pass on their traits to offspring.

What is the main idea of this paragraph?


(A) It is impossible to come up with big insights without crossing domains.
(B) The idea of evolution did not originate with Darwin.
(C) It is Malthus, not Darwin, who should be given credit for the theory of evolution.
(D) It was the connections that Darwin uncovered more than the facts themselves that made his work important.

8. It has been argued by behavioral economists in all earnestness that poverty shapes mindsets. From here, it is a hop, skip, and jump
to holding that the poor are poor because their poverty prevents them from thinking and acting in ways that can take them out of
poverty. When behaviorist economics speaks of poverty as a “cognitive tax”, it writes ‘action’ — the political agency of the poor — out
of the equation. In such a case, the focus as well as the onus of poverty-alleviation would shift from the state — from macroeconomic
policy, from having to provide employment, health and education — to changing the behavior of the poor. The structural causes of

PEV 107 Page 134


poverty — rising inequality and unemployment — as well as the behavior of the owners of capital are evicted from the poverty debate,
and no longer need be the focus of public policy.

Which of these options best summarizes the given paragraph?


(A) Where decisions of the poor tend to be flawed from an economic point of view, behavioral economists believe that
governments can intervene with policies aimed at nudging the poor towards the right decision.
(B) By shifting the burden of poverty alleviation from the state onto the poor themselves, behavioral economists are ignoring
both the structural causes of poverty as well as the behavior of the wealthy.
(C) Given that poverty diminishes political agency and shapes mindsets, insights into how poverty affects behavior could have
implications in public policy.
(D) The focus of public policy ought to be in providing employment, health and education, rather than addressing rising inequality
and changing the behavior of the poor.

9. In an attempt to encourage “livelier” writing, some teachers want children to stop using words like ‘said’, which doesn’t have any
emotion. The assumption here is that emotion is a desirable quality in every word of a sentence, and that a rich word is always more
appropriate than a plain one. You don’t have to invoke Hemingway, who made a fetish of plain words, to recognize that successful
writing modulates the lavishness of its diction for effect, rather than cranking the dial all the way to maximum floridity and leaving it
there.

What is the main idea of this paragraph?


(A) Successful writers use only plain, unemotional words.
(B) Emotion is not a desirable quality in every sentence.
(C) Avoiding words like “said” helps children improve their writing.
(D) Good writers use rich words in moderation, for effect.

10. Euphemisms in use seem to change every generation or so – a tendency towards which we often roll our eyes. But the fact is that a
word is always redolent of various associations and metaphorical extensions beyond its core meaning. Indeed, a word is like a bell
tone, with a central pitch seasoned by overtones. As the tone fades away, the overtones can hang in the air. Words then bias as
equivalents to the overtones. As we move on the euphemism treadmill then, from ‘crippled’ to ‘handicapped’ to ‘disabled’ to
‘differently abled’, we acknowledge the eternal gulf between language and opinion. In a linguistically mature society, we should expect
that the terms we introduce to help us kick off new ways of thinking will require periodic replacement, like tyres.

What is the main idea of this paragraph?


(A) All words wear out with use, like tyres, and need to be replaced periodically with more meaningful equivalents.
(B) The eternal quest for euphemisms to bridge the gap between language and opinion is tedious.
(C) We must accept the euphemism treadmill as an essential part of linguistic life in a civilized society.
(D) Euphemisms help us avoid the trap of thinking too much into the meanings of words.

11. Many students, especially those who are poor, intuitively know what the schools do for them. They school them to confuse process
and substance. Once these become blurred, a new logic is assumed: the more treatment there is, the better are the results; or,
escalation leads to success. The pupil is thereby schooled to confuse teaching with learning, grade advancement with education, a
diploma with competence, and fluency with the ability to say something new. His imagination is schooled to accept service in place of
value. Medical treatment is mistaken for health care, social work for the improvement of community life, police protection for safety,
military poise for national security, the rat race for productive work. Health, learning, dignity, independence, and creative endeavor
are defined as little more than the performance of the institutions which claim to serve these ends, and their improvement is made to
depend on allocating more resources to the management of hospitals, schools, and other agencies in question.

What is the main idea of this paragraph?


(A) Allocating more resources to health, learning, dignity, independence, and creative endeavor is not likely to increase their
quality.
(B) Schooling doctors thinking by teaching students to identify the process with the results.
(C) As a society we often mistake grades and diplomas for competence, medical treatment for healthcare, police protection for
safety and the rat race for productive work.
(D) When it comes to schooling, less is more: the less the treatment there is, better the results

PEV 107 Page 135


12. In times when expectations can change at the drop of a hat, flexibility is as important as speed. However, in their zeal for
efficiency, some organizations have engineered out all the slack they can from their businesses at the expense of agility. As Tom
DeMarco puts it in his book Slack, ‘An organization that can accelerate but not change direction is like a car that can speed up but not
steer. In the short run, it makes lots of progress in whatever direction it happened to be going. In the long run, it’s just another road
wreck.’

Which of the options below conveys the main idea of the paragraph best?
(A) Only organizations that are not very efficient can adapt to change well.
(B) Too little slack in organizations is as bad for innovation as too much slack.
(C) Reducing slack to zero can cost businesses their adaptability to change.
(D) Business processes without slack slow down an organization’s progress.

13. The image of an oral telling may be caught on paper, film or in digital format, but recordings are not the word shared live. The
presence of teller and audience, and the immediacy of the moment are not fully captured by any form of technology. Unlike the insect
frozen in amber, a told story is alive. It always changes from one telling to the next depending on the voice and mood of the
storyteller, the place of its telling and the response of the audience. The story breathes with the teller’s breath.

Which of the options below conveys the main idea of the paragraph best?
(A) Unlike stories in paper, film and digital formats, a told story is alive, and hence more potent.
(B) The immediacy and intimacy of live storytelling cannot be captured by recordings.
(C) Technology and oral storytelling are separated by a deep divide and one cannot take the place of the other.
(D) The living human presence experienced through oral tradition brings out the true power of stories.
14. The median house value in Butlerville has fallen significantly in the last few months. Nonetheless, the number of homes sold has
been at its lowest level in seven years.
Which of the following best explains the discrepancy?
(A) The value of homes is not the only determinant of whether a house will be sold.
(B) The neighboring city of Jacksonburg has seen a recent surge in home sales.
(C) The Butlerville economy is struggling and the unemployment rate has reached historic rates.
(D) Analysts predict that the number of homes sold will increasing in the coming year.
(E) Homes priced over a million dollars have seen the sharpest decline in sales.
15. TV networks believe that football broadcasts provide greater opportunity for profit than any other sport. This belief persists
despite the fact that professional baseball leagues have increased their revenue by fifty percent over the last ten years while
professional football leagues have increased revenue by only fifteen percent over that time.

Which of the following statements, if true, would most help to reconcile the TV networks' beliefs with the statistics cited?

(A) Many TV networks purchase broadcasting rights to only one sport


(B) Baseball has been popular for many years while football's increase in popularity has come primarily in the last thirty years
(C) Baseball leagues have significantly longer seasons than football leagues
(D) In the last ten years, football has gained more TV revenue than baseball despite baseball leagues gaining more revenue for
their respective league.
(E) Football and baseball leagues attract different types of advertisers

16. Under the modern model for music distribution, musicians sell the right to broadcast their music to companies who give listeners
affordable access to large quantities of music. Some musicians who have been selling their music for many years are displeased with
the new model and have seen decreased revenue. Yet, musicians as a whole are making more money under the new model than they
were previously.

Which of the following, if true, would most help to reconcile the apparent conflict?

(A) Some musicians want to make more money than they made under the old model
(B) The old model could be used now and would generate more revenue than the new model
(C) The new model is not generating as much revenue as it could be generating
(D) A greater number of musicians sell music now than under the old model
(E) Music producers charge higher fees now, leaving less revenue for musicians

PEV 107 Page 136


17. Recent evidence has conclusively shown that cholesterol levels do not correlate with human lifespan. Despite this new evidence,
doctors still advise patients with high cholesterol to take medication and engage in physical activity to reduce cholesterol levels.

Which of the following, if true, would most help resolve this discrepancy?

(A) Most medications that doctors prescribe to reduce cholesterol levels are clinically proven to be effective.
(B) High cholesterol levels are proven be correlated with hypertension disorders.
(C) Many doctors receive large gifts from pharmaceutical companies.
(D) Engaging in extensive physical exercise can stress the human heart.
(E) Low levels of cholesterol are proven to increase quality of life.

18. Over the past ten years, insurance premiums have increased, resulting in a large decrease in insurance enrollment across the
country. Insurance company revenues, however, have progressively increased in each of the ten years during this period, and industry
analysts predict further increases in years to come.

Which one of the following, if true, offers the best explanation for the situation described above?

(A) Insurance companies donate substantial sums of money to political campaigns.


(B) The rise of concierge medicine exceeds the number of those enrolling in traditional insurance in most high-income areas.
(C) More individuals are seeking a subsidy for their health care.
(D) Most insurance companies raise their premiums every three years.
(E) The decrease in the number of people enrolling for health insurance over the past ten years has been more than offset by the
increases in insurance premiums.

19. During his second semester of college, Tom enrolled in tutoring sessions to help him improve his grades. However, his GPA for his
second semester was worse than his GPA for his first semester.

Each of the following, if true, helps to explain why Tom's GPA worsened, EXCEPT:

(A) The tutoring sessions that Tom enrolled in were counterproductive because the tutor taught him incorrect information.
(B) Tom took much more difficult courses his second semester than he did during his first semester.
(C) Tom experienced a personal crisis during his second semester that distracted him from school.
(D) Tom's school initiated a new academic policy his second semester, which required all professors to base their grading on a
curve.
(E) The tutoring sessions took more time than Tom had realized they would take.

20. Eye surgery has become a viable alternative to wearing glasses. There have been questions about long term effectiveness for many
years, but the doubt seems to be disappearing. Few people have reported negative effects even years after surgery and many people
say their improved vision has lasted. We should see the demand for eye surgery increasing drastically as a result of doubts being
dispelled. Yet, the number of people obtaining eye surgeries remains relatively constant.

Which of the following most helps to explain the paradox?

(A) Most people who decide to not have eye surgery, do so because of the high cost
(B) Eye surgery is more popular in some locations than in others
(C) Some people have experienced negative effects from eye surgery
(D) There are many negative effects from eye surgery even though most people now believe that there are none
(E) Previous doubts about eye surgery effectiveness were unreasonable

Exercises: Mixed

1. The average life expectancy for the United States population as a whole is 73.9 years, but children born in Hawaii will live an average
of 77 years, and those born in Louisiana, 71.7 years. If a newlywed couple from Louisiana were to begin their family in Hawaii,
therefore, their children would be expected to live longer than would be the case if the family remained in Louisiana. Which of the
following, if true, would most seriously weaken the conclusion drawn in the passage?

PEV 107 Page 137


A. Insurance company statisticians do not believe that moving to Hawaii will significantly lengthen the average Louisianan's life.
B. The governor of Louisiana has falsely alleged that statistics for his state are inaccurate.
C. The longevity ascribed to Hawaii's current population is attributable mostly to genetically determined factors.
D. Thirty percent of all Louisianans can expect to live longer than 77 years.
E. Most of the Hawaiian Islands have levels of air pollution well below the national average for the United States.

2. The average life expectancy for the United States population as a whole is 73.9 years, but children born in Hawaii will live an average
of 77 years, and those born in Louisiana, 71.7 years. If a newlywed couple from Louisiana were to begin their family in Hawaii,
therefore, their children would be expected to live longer than would be the case if the family remained in Louisiana. Which of the
following statements, if true, would most significantly strengthen the conclusion drawn in the passage?
A. As population density increases in Hawaii, life expectancy figures for that state are likely to be revised downward.
B. Environmental factors tending to favor longevity are abundant in Hawaii and less numerous in Louisiana.
C. Twenty-five percent of all Louisianans who move to Hawaii live longer than 77 years.
D. Over the last decade, average life expectancy has risen at a higher rate for Louisianans than for Hawaiians.
E. Studies show that the average life expectancy for Hawaiians who move permanently to Louisiana is roughly equal to that of
Hawaiians who remain in Hawaii.

3. A program instituted in a particular state allows parents to prepay their children's future college tuition at current rates. The
program then pays the tuition annually for the child at any of the state's public colleges in which the child enrolls. Parents should
participate in the program as a means of decreasing the cost for their children's college education. Which of the following, if true, is
the most appropriate reason for parents NOT to participate in the program?
A. the parents are unsure about which public college in the state the child will attend.
B. The amount of money accumulated by putting the prepayment funds in an interest-bearing account today will be greater
than the total cost of tuition for any of the public colleges when the child enrolls.
C. The annual cost of tuition at the state's public colleges is expected to increase at a faster rate than the annual increase in the
cost of living.
D. Some of the state's public colleges are contemplating large increases in tuition next year.
E. The prepayment plan would not cover the cost of room and board at any of the state's public colleges.
4. Company Alpha buys free-travel coupons from people who are awarded the coupons by Bravo Airlines for flying frequently on Bravo
airplanes. The coupons are sold to people who pay les for the coupons than they would pay by purchasing tickets from Bravo. This
making of coupons results in lost revenue for Bravo. To discourage the buying and selling of free-travel coupons, it would be best for
Bravo Airlines to restrict the
A. number of coupons that a person can be awarded in a particular year
B. use of the coupons to those who were awarded the coupons and members of their immediate families
C. days that the coupons can be used to Monday through Friday
D. amount of time that the coupons can be used after they are issued
E. number of routes on which travelers can use the coupons
5. To prevent some conflicts of interest, Congress could prohibit high-level government officials from accepting positions as lobbyists
for three years after such officials leave government service. One such official concluded, however, that such a prohibition would be
unfortunate because it would prevent high-level government officials from earning a livelihood for three years. The official's
conclusion logically depends on which of the following assumptions?
A. Laws should not restrict the behavior of former government officials.
B. Lobbyists are typically people who have previously been high-level government officials.
C. Low-level government officials do not often become lobbyists when they leave government service.
D. High-level government officials who leave government service are capable of earning a livelihood only as lobbyists.
E. High-level government officials who leave government service are currently permitted to act as lobbyists for only three years.

6. A conservation group in the United States is trying to change the long-standing image of bats as frightening creatures. The group
contends that bats are feared and persecuted solely because they are shy animals that are active only at night. Which of the following,
if true, would cast the most serious doubt on the accuracy of the group's contention?
A. Bats are steadily losing natural roosting places such as caves and hollow trees and are thus turning to more developed areas
for roosting.
B. Bats are the chief consumers of nocturnal insects and thus can help make their hunting territory more pleasant for humans.
C. Bats are regarded as frightening creatures not only in the United States but also in Europe, Africa, and South America.
D. Raccoons and owls are shy and active only at night; yet they are not generally feared and persecuted.

PEV 107 Page 138


E. People know more about the behavior of other greatly feared animal species, such as lions, alligators, and greatly feared
animal species, such as lions, alligators, and snakes, than they do about the behavior of bats.

7. Meteorite explosions in the Earth's atmosphere as large as the one that destroyed forests in Siberia, with approximately the force of
a twelve-megaton nuclear blast, occur about once a century. The response of highly automated systems controlled by complex
computer programs to unexpected circumstances is unpredictable. Which of the following conclusions can most properly be drawn, if
the statements above are true, about a highly automated nuclear-missile defense system controlled by a complex computer program?
A. Within a century after its construction, the system would react inappropriately and might accidentally start a nuclear war.
B. The system would be destroyed if an explosion of a large meteorite occurred in the Earth's atmosphere.
C. It would be impossible for the system to distinguish the explosion of a large meteorite from the explosion of a nuclear
weapon.
D. Whether the system would respond inappropriately to the explosion of a large meteorite would depend on the location of the
blast.
E. It is not certain what the system's response to the explosion of a large meteorite would be, if its designers did not plan for
such a contingency.

8. Opponents of laws that require automobile drivers and passengers to wear seat belts argue that in a free society people have the
right to take risks as long as the people do not harm other as a result of taking the risks. As a result, they conclude that it should be
each person's decision whether or not to wear a seat belt. Which of the following, if true, most seriously weakens the conclusion
drawn above?
A. Many new cars are built with seat belts that automatically fasten when someone sits in the front seat.
B. Automobile insurance rates for all automobile owners are higher because of the need to pay for the increased injuries or
deaths of people not wearing seat belts.
C. Passengers in airplanes are required to wear seat belts during takeoffs and landings.
D. The rate of automobile fatalities in states that do not have mandatory seat belt laws is greater than the rate of fatalities in
states that do have such laws.
E. In automobile accidents, a greater number of passengers who do not wear seat belts are injured than are passengers who do
wear seat belts.

9. The cost of producing radios in Country Q is ten percent less than the cost of producing radios in Country Y. even after
transportation fees and tariff charges are added, it is still cheaper for a company to import radios from Country Q to Country Y than to
produce radios in Country Y. The statements above, if true, best support which of the following assertions?
A. labor costs in Country Q are ten percent below those in Country Y.
B. importing radios from Country Q to Country Y will eliminate ten percent of the manufacturing jobs in Country Y.
C. the tariff on a radio imported from Country Q to Country Y is less than ten percent of the cost of manufacturing the radio in
Country Y.
D. the fee for transporting a radio from Country Q to Country Y is more than ten percent of the cost of manufacturing the radio
in Country Q.
E. it takes ten percent less time to manufacture a radios in Country Q than it does in Country Y.

10. Toughened hiring standards have not been the primary cause of the present staffing shortage in public schools. The shortage of
teachers is primarily caused by the fact that in 11 recent years teachers have not experienced any improvements in working conditions
and their salaries have not kept pace with salaries in other professions. Which of the following, if true, would most support the claims
above?
A. Many teachers already in the profession would not have been hired under the new hiring standards.
B. Today more teachers are entering the profession with a higher educational level than in the past.
C. Some teachers have cited higher standards for hiring as a reason for the current staffing shortage.
D. Many teachers have cited low pay and lack of professional freedom as reasons for their leaving the profession.
E. Many prospective teachers have cited the new hiring standards as a reason for not entering the profession.

11. Increase in the level of high-density lipoprotein (HDL) in the human bloodstream lower bloodstream-cholesterol levels by
increasing the body's capacity to rid itself of excess cholesterol. Levels of HDL in the bloodstream of some individuals are significantly
increased by a program of regular exercise and weight reduction. Which of the following can be correctly inferred from the statements
above?
A. Individuals who are underweight do not run any risk of developing high levels of cholesterol in the bloodstream.

PEV 107 Page 139


B. Individuals who do not exercise regularly have a high risk of developing high levels of cholesterol in the bloodstream late in
life.
C. Exercise and weight reduction are the most effective methods of lowering bloodstream cholesterol levels in humans.
D. A program of regular exercise and weight reduction lowers cholesterol levels in the bloodstream of some individuals.
E. Only regular exercise is necessary to decrease cholesterol levels in the bloodstream of individuals of average weight.

12. When limitations were in effect on nuclear-arms testing, people tended to save more of their money, but when nuclear-arms
testing increased, people tended to spend more of their money. The perceived threat of nuclear catastrophe, therefore, decreases the
willingness of people to postpone consumption for the sake of saving money. The argument above assumes that
A. the perceived threat of nuclear catastrophe has increased over the years.
B. most people supported the development of nuclear arms
C. people's perception of the threat of nuclear catastrophe depends on the amount of nuclear-arms testing being done
D. the people who saved the most money when nuclear-arms testing was limited were the ones who supported such limitations
E. there are more consumer goods available when nuclear-arms testing increases.

13. A discount retailer of basic household necessities employs thousands of people and pays most of them at the minimum wage rate.
Yet following a federally mandated increase of the minimum wage rate that increased the retailer's operating costs considerably, the
retailer's profits increased markedly. Which of the following, if true, most helps to resolve the apparent paradox?
(A) Over half of the retailer's operating costs consist of payroll expenditures; yet only a small percentage of those expenditures go
to pay management salaries.
(B) The retailer's customer base is made up primarily of people who earn, or who depend on the earnings of others who earn, the
minimum wage.
(C) The retailer's operating costs, other than wages, increased substantially after the increase in the minimum wage rate went
into effect.
(D) When the increase in the minimum wage rate went into effect, the retailer also raised the age rate for employees who had
been earning just above minimum wage.
(E) The majority of the retailer's employees work as cashiers, and most cashiers are paid the minimum wage.

14. Products sold under a brand name used to command premium prices because, in general, they were superior to nonbrand rival
products. Technical expertise in product development has become so widespread, however, that special quality advantages are very
hard to obtain these days and even harder to maintain. As a consequence, brand-name products generally neither offer higher quality
nor sell at higher prices. Paradoxically, brand names are a bigger marketing advantage than ever. Which of the following, if true, most
helps to resolve the paradox outlined above?
(A) Brand names are taken by consumers as a guarantee of getting a product as good as the best rival products.
(B) Consumers recognize that the quality of products sold under invariant brand names can drift over time.
(C) In many acquisitions of one corporation by another, the acquiring corporation is interested more in acquiring the right to use
certain brand names than in acquiring existing production facilities.
(D) In the days when special quality advantages were easier to obtain than they are now, it was also easier to get new brand
names established.
(E) The advertising of a company's brand-name products is at times transferred to a new advertising agency, especially when
sales are declining.

15. High levels of fertilizer and pesticides, needed when farmers try to produce high yield of the same crop year after year, pollute
water supplies. Experts therefore urge farmers to diversify their crops and to rotate their plantings yearly. To receive governmental
price-support benefits for a crop, farmers must have produced that same crop for the past several years. The statements above, if
true, best support which of the following conclusions?
(A) The rules for governmental support of farm prices work against efforts to reduce water pollution.
(B) The only solution to the problem of water pollution from fertilizers and pesticides is to take farmland out of production.
(C) Farmers can continue to make a profit by rotating diverse crops, thus reducing costs for chemicals, but not by planting the
same crop each year.
(D) New farming techniques will be developed to make it possible for farmers to reduce the application of fertilizers and
pesticides.
(E) Governmental price supports for farm products are set at levels that are not high enough to allow farmers to get out of debt.

16. A famous singer recently won a lawsuit against an advertising firm for using another singer in a commercial to evoke the famous
singer's well-known rendition of a certain song. As a result of the lawsuit, advertising firms will stop using imitators in commercials.
PEV 107 Page 140
Therefore, advertising costs will rise, since famous singers' services cost more than those of their imitators. The conclusion above is
based on which of the following assumptions?
(A) Most people are unable to distinguish a famous singer's rendition of a song from a good imitator's rendition of the same song.
(B) Commercials using famous singers are usually more effective than commercials using imitators of famous singers.
(C) The original versions of some well-known songs are unavailable for use in commercials.
(D) Advertising firms will continue to use imitators to mimic the physical mannerisms of famous singers.
(E) The advertising industry will use well-known renditions of songs in commercials.

17. The interview is an essential part of a successful hiring program because, with it, job applicants who have personalities that are
unsuited to the requirements of the job will be eliminated from consideration. The argument above logically depends on which of the
following assumptions?
(A) A hiring program will be successful if it includes interviews.
(B) The interview is a more important part of a successful hiring program than is the development of a job description.
(C) Interviewers can accurately identify applicants whose personalities are unsuited to the requirements of the job.
(D) The only purpose of an interview is to evaluate whether job applicants' personalities are suited to the requirements of the
job.
(E) the fit of job applicants' personalities to the requirements of the job was once the most important factor in making hiring
decisions.
18. Because no employee wants to be associated with bad news in the eyes of a superior, information about serious problems at lower
levels is progressively softened and distorted as it goes up each step in the management hierarchy. The chief executive is, therefore,
less well informed about problems at lower levels than are his or her subordinates at those levels. The conclusion drawn above is
based on the assumption that
(A) problems should be solved at the level in the management hierarchy at which they occur
(B) employees should be rewarded for accurately reporting problems to their superiors
(C) problem-solving ability is more important at higher levels than it is at lower levels of the management hierarchy
(D) chief executives obtain information about problems at lower levels from no source other than their subordinates
(E) some employees are more concerned about truth than about the way they are perceived by their superiors

19. Chaos Inc., a company in the fast moving consumer goods sector had sales output rising by nearly 12% per year. A new
partner, Kim, joined the company and introduced many innovations such as Kaizen groups to solve manufacturing problems, and
e-business applications involving vendors and distributors. But after Kim’s advent, sales output increased by only 6% per year. It
appears that the innovations implemented by Kim caused the reduction in the annual sales growth rate.
Which of the following if true would weaken the above conclusion?
A) Kim’s innovations were focused on long-term improvement and ignored short-term profitability and growth.
B) Chaos Inc. does not price its products based upon the actual cost of production.
C) General demand for the company’s product has declined over time.
D) Managers of the company were busy in creating innovations, rather than focusing on sales promotions.
E) None of these

20. The extent to which a society is really free can be gauged by its attitude toward artistic expression. Freedom of expression can
easily be violated in even the most outwardly democratic of societies. When a government arts council withholds funding from a
dance performance that its members deem “obscene,” the voices of a few bureaucrats have in fact censored the work of the
choreographer, thereby committing the real obscenity of repression.
Which of the following, if true, would most seriously weaken the argument above?
A) Members of government arts councils are screened to ensure that their beliefs reflect those of the majority
B) The term obscenity has several different definitions that should not be used interchangeably for rhetorical effect
C) Failing to provide financial support for a performance is not the same as actively preventing or inhibiting it
D) The council’s decision could be reversed if the performance were altered to conform to public standards of appropriateness.
E) The definition of obscenity is something on which most members of society can agree.

21. It is unfortunate for a country, specially for the poor and women, that whenever any kind of disincentive – even of the mildest
nature such as debarring a person with more than two children from contesting any local body election – is introduced to
promote family planning, there are always some activists and organizations who unnecessarily oppose the measures without
realizing that the voluntary family planning programmes area miserable flop as clearly highlighted by the national family health
surveys.
Which of the following if true would strengthen the author’s contention?

PEV 107 Page 141


A) Percentage of rural women voluntarily seeking family planning services in government health centre facilities has drastically
come down over the last decade.
B) One child policy on China, where employees with more than one child are discriminated against, has been very effective in
controlling their population.
C) Activists and organization which seem to advocate civil rights issues typically have their own vested interests and hidden
agendas.
D) Punitive measures are the best deterrents for refraining people from committing deeds which adversely affect the society at
large.
E) None of these

22. In recent study published in The Journal of Family Practice, no significant benefit over a placebo was found from using the
antibiotic Amoxicillin among 135 patients with typical indications of a sinus infection. All the patients complained of sinusitis, with
pus in the nasal cavity, facial pressure of nasal discharge lasting longer than seven days. A small subgroup of patients receiving the
antibiotic became better faster than the others. But the researchers were unable to discern anything about those patients prior
to administering Amoxicillin that indicated a bacterial infection, as opposed to a viral one.
From the above passage it may be inferred that
A) Use of an antibiotic is beneficial for sinusitis patients.
B) Antibiotics should be administered to only those sinusitis patients, who have been diagnosed to have a bacterial infection.
C) Antibiotics do not have any effect on sinusitis patients.
D) Use of an antibiotic may be beneficial for a small percentage of sinusitis patients with bacterial infection, but in general
antibiotics are useless for sinusitis patients.
E) None of these

23. Governments have only one response to public criticism of socially necessary services: regulation of the activity of providing
those services. But governments inevitably make the activity more expensive by regulating it, and that is particularly troublesome
in these times of strained financial resources. However, since public criticism of child-care services has undermined all confidence
in such services, and since such services are socially necessary, the government is certain to respond.
Which one of the following statements can be inferred from the passage?
A) The quality of child care will improve.
B) The cost of providing child-care services will increase.
C) The government will use funding to foster advances in child care.
D) If public criticism of policy is strongly voiced, the government is certain to respond.
E) If child-care services are not regulated, the cost of providing child care will not increase.

24. A survey recently revealed a high correlation between a household’s level of education and its library. Specifically, the more
years of college and graduate school education received by the household’s members, the more books in the household’s library.
The survey also indicated that the higher the education level of the household, the greater the percentage of books that are not
works of fiction in its library.
Which of the following can be properly inferred from the survey results cited above?
A) People with a higher level of education prefer reading nonfiction to works of fiction.
B) Households with low education levels generally own more works of fiction than do households with high education levels.
C) Households with lower levels of education generally own more works of fiction than nonfiction.
D) The higher the education level of a household, the fewer works of fiction owned by the household.
E) Households with high education levels generally own more nonfiction books than do households with low education levels.

25. Columnist: The winner of this year’s national spelling bee won by correctly spelling the spoken word Ursprache, which means
“fame” in German. Given the richness of our language, why must we resort to words taken from modern foreign languages to
challenge our best spellers? Ursprache is listed in our dictionary, as are words from many other foreign languages, but future
spelling bees should limit themselves to words in our dictionary that have been anglicized in all aspects because spelling English
words, not knowledge of linguistics and international phonetics, is the point of these contests.
Which of the following can most reasonably be inferred from the argument above?
A) The spelling contest winner knew how to spell most of the anglicized words in the dictionary.
B) Foreign words are more difficult than anglicized words for all contestants to spell.
C) Spelling contestant winners should be determined by their facility with all aspects of language.
D) To spell foreign words, contestants must recognize the language and know its pronunciation.
E) The English language contains more borrowed words than most other languages.

PEV 107 Page 142

You might also like